Вы находитесь на странице: 1из 48

Original jx canned answer:

Whether congress/ senator’s bill prohibiting [. ] validly precluded the supreme court from hearing [. ] ‘s case by restricting the court’s appellate and original jurisdiction.
Article III section 2 specifies that the supreme court’s orig jx extends only to those cases involving ambassadors or other public ministers and consults and (2) those in
which a state is a party and that its appellate jx extends to all other cases “ with such exceptions and under such regulations as congress shall make.” In Marbury v.
Madison, the judiciary act of 1791 gave the supreme court original jx to issue a writ of mandamus. However, chief justice john marshall observed that article III of the
constitution conferred original jx only in a narrow set of cases and none of which were involved in marbury’s case. Chief Marshall stated, where a statute and the
constitution are in conflict under the supremacy clause “ the constitution is paramount.” Thus, the judiciary act of 1791 that expanded jx over marbury’s case could not
be given the effect of law and a writ of mandamus could not be issued by the supreme court. Whether the section of the hypothetical bill is constitutionally proper in
[preventing the case from going to the supreme court] depends on if this case falls within the cases outlined in the text of Article III section 2. The language of [. ] would
suggest [party]’s case is/is not allowed to go directly to the supreme court. While the language from [the federal/senator’s act] purports to limit/ expand the court’s
original jx so that [. ] would be/would not be allowed to bring this case to the court. In Marbury the facts involved an attempt by congress to expand the supreme court’s
original jx and here congress is [. ] orginal jx . The principle in Marbury v. Madison establishes that any statute attempting to adjust the court’s original jx must fail
because the constitution is paramount. Thus, the [party] is correct that secton[ ] is unconstitutional/constitutional and thus [party] can bring case directly to the
supreme court.
Supreme court appellate review canned answer
To determine whether the [party] is correct that section [ ] of the bill is/ is not constitutionally proper in preventing/ allowing [party] to bring the case on appeal to the
supreme court depends on an application of article III section 2. On these facts, it seems that congress may make such “exceptions and regulations” to the courts
appellate jx. [party] may argue the text of the constitution in Article III section 2. 1 states “ the judicial power shall be vested in one supreme court ….[and] shall extend to
all Cases… and Controversies; (Article III section 2) and that in all the cases other than those where it has orig jx the supreme court shall have appellate jx and limiting
[party’s single case] would be contradictory to the structure of the constitution by limiting the supreme court’s power to hear a category of case and controversy.
However, the [other party] would argue even tho the constitution provides the supreme court with appellate jx, the supreme court found in Mccardle the appellate jx is
conferred “ with such exceptions and under regulations as congress shall make.” ( article III section 2). Therefore [other party] is likely correct the section[ ] is
constitutional and the supreme court may not hear case on appeal.
Justiciability argument
Whether [ ] lawsuit is justiciable depends on whether the case meets the requirements set forth in Article III section 1 “ Case or Controversy” provision. The term
justiciability refers to whether the federal judiciary has jx to decide a particular dispute. The Supreme Court has developed five justiciability doctrines including the
prohibition against advisory opinions, standing, ripeness, mootness, and political question doctrine. The first inquiry that will be discussed is whether [party] has
standing. Under Lujan v. Defenders of Wildlife , the supreme court determined that the Article III case or controversy requires that a litigant must have an injury that is
fairly traceable to D’s conduct. To meet the constitutional standing requirements to bring a case to federal court the P must demonstrate they suffer (1) an injury in fact
;(2) that was caused by the D and (3) that is redressable by the court. For the injury in fact element, [party] must show he/she suffered an invasion of a legally protected
interest that is concrete and particularized and actual or imminent rather than hypothetical. Party can allege suffered a concrete injury because of harm to ( ex. Health
and safety of community, harm to financial well being from decreased employment, harm to education. Second, the causation element is met here because fairly
traceable to D’s conduct and party’s harm suffered here/ is not met here because the injury is too speculative and there are too many contingencies/ factors that could
cause this injury. whether the harm occurs depends on ( 2 many factors) assuming the feared harm occurs it is not clear it would lead to [this fact alleged]. There are too
many independent causal actors that are not involved in the litigation; therefore, the causal link is too attenuated. Third, the redressability element is met here because
the court could issue declaratory relief that would [fix injury]
Ripeness element : to determine whether a case is ripe a ct will look to whether a finding of on-ripeness will cause substantial hardship to the p and to the fitness of the
issue/ record for judicial review. The court has helf the more the case involves a purely legal question the more a ct will find ripeness satisfied Abbott Labs v. Gardner.
The p can argue that statue being enforced or even if has not then faces an undue hardship from the possible enforcement of the statute. ( ex: main business is thing
banned). Having to give up business seems inevitable bc of the heightened enforcement. Faced with the decision to contune work and face liability under the act by doing
activiety or forego the income by ceasing conduct. The gov could argue that the prosecution is not imminent or even likely & may suggest that the issue of whether gov
should enforce the is not fit for judicial review because the ( ex: AG) is tasked with enforcing all laws so it not appropriate for judiciary to instruct AG on discretionary
matters like enforcement. However the economic incentive to litigate the case is clear and thus the enforcement is sufficiently imminent and the sig of the hardship
would make the lawsuit ripe for judicial consideration. The government’s argument of judicial fitness for review brings up the issue of whether this case presents a
political question. Should the court find the matter involved in this case is more appropriate for resolution among the political branches of government ( executive &
legislature then this case is a political question and the ct will not consider the case. Baker v. Carr provides six factors for the fed ct to consider when deciding if a case is
a political question. The first three are considered more important. First Baker factor whether the text of the constitution expressly commits a particular issue to
another branch
PQ examples:
The first two factors are whether the Constitution seems to have committed the power to another branch, or whether the question presents difficulties in the creation of
standards by the courts.

 example, the Constitution does give Congress the power to declare war, but it also makes the President the commander-in-chief of the military. The document
says nothing about resolving conflicts when, for example, the President commits military forces without a formal declaration of war. In addition, the decision
whether and when to commit military forces involves substantial issues of policy that are probably better left to the political branches.
 Similarly, in the President and the Senate have shared responsibilities for nominating and confirming judicial nominees, but what happens when there is a
stalemate between the branches? Can the judiciary order the Senate to hold hearings? Force a vote on nominees? It’s likely that a court would simply let the
branches work it out for themselves.
 In D, not only does the Constitution assign the sole power of impeachment and the sole power to try impeachments to the House and Senate, respectively,
but the Constitution contains no criteria for defining ‘‘high crimes and misdemeanors.’’

Impeachment Example:
If senate advise and consent of treaty power the constitution does not define how the senate can advise the president. In powell the ct found no PC bc the question was
whether congress had a specific power – power to exclude a member based on a criteria other than the qualifications listed in the constitution. The question of best way
for senate to advice is implicitly delegated to the senate bc the constitution is silent.

Gerrymandering PQ/ Justiciability Argument:


….purpose of the controlling party is to totally get rid of representation of approximately 40% of the voters in the state- which would seem under most any definition to
constitute “ vote dilution.”
Congress power to declare war political question analysis
11th amendment

I.
Under the federal Age Discrimination in Employment Act (ADEA), Congress has specified that no employer, public or private, may discriminate on the basis
of age against an employee, no matter how old. The ADEA by its terms applies to all employers, public or private. The Act includes a provision allowing the
person discriminated against to recover money damages in “any state or federal court of competent jurisdiction.” Methuselah, who is 80, worked as an actuary
for the State of Efficiency until last year, when he was fired solely on account of what his boss called “an age so advanced that no one that old could possibly
be a competent worker.” Meth has sued the state in the Efficiency state courts, seeking money damages under the federal statute for the lost job. Is there any
constitutional barrier to Meth's recovery?
A. Yes. Under Alden v. Maine, 527 U.S. 706 (1999), a constitutionally based “sovereign immunity” prevents the states from being required to entertain,
in their own courts, a private damage suit based on violations of federal law. This immunity means that Congress can't do what it purported to do
here—require Efficiency to allow itself to be sued for money damages in its own courts by a private party for a violation of the federal ADEA.

B. NOTE: Alden says that this sovereign immunity is not specifically mentioned in the Constitution, but is tacitly recognized by the Tenth Amendment (see
LSP/GEN
C. NOTE:Observe that when you combine Alden with the Court's recent decisions saying that Congress can't generally abrogate the Eleventh Amendment
(e.g., Seminole Tribe), the states are largely immune from private damage suits for violations of federally created rights. So here, Meth can neither sue
Efficiency in state court for age-discrimination damages despite the “protections” he supposedly has under the ADEA, nor sue the state in federal court
for these damages.
II. Under the federal Family and Medical Leave Act (FMLA), Congress authorized employees to take up to 12 weeks of unpaid family leave to care for family
members who suffer from serious health conditions. Congress passed the FMLA because it had extensive evidence of gender discrimination by the states in
the administration of employee leave benefits and desired to remedy that discrimination with the requirements of the FMLA. Dibbs, an employee of the State
Department of Human Resources, sued the State after it denied his request for family leave. Is the lawsuit barred by the Eleventh Amendment?
A. No. In passing the FMLA, Congress relied on its §5 of the Fourteenth Amendment power to enforce the Equal Protection Clause. In Nevada Dept. of
Human Resources v. Hibbs, 538 U.S. 721 (2003), the Court ruled that Congress abrogated the states' sovereign immunity in the FMLA because the
legislation was a congruent and proportional solution to the problem of gender discrimination.
B. The Court has also held that a state does not have immunity in a lawsuit, brought under the Americans with Disabilities Act (ADA), where the
disabled plaintiff alleged that he was denied access to the courts. Because access to the courts is a fundamental right protected by the Fourteenth
Amendment, the Court upheld Congress' §5 power to abrogate state sovereign immunity. In general lawsuits under the ADA, however, where
employees claim that they were fired because of their disabilities, Congress lacks the power to abrogate state sovereign immunity. CH pp. 229-231.
III. State Prisoner Tony is paraplegic and uses a wheelchair. State prison authorities confine him to a small cell that is so narrow that he cannot move his
wheelchair around. They also refuse his requests for trips to the restroom so that he frequently has to sit in his own waste. Can Tony sue the State for violations
of the Americans with Disabilities Act?
A. Yes. Although in earlier cases the Court had held that Congress could not use its §5 of the Fourteenth Amendment powers to abrogate state
sovereign immunity in the Americans with Disabilities Act (ADA), the Court held unanimously in U.S. v. Georgia, 546 U.S. 151 (2006), that a
paraplegic prisoner could sue the state under the ADA in these circumstances. The reasoning was that the prison's conduct violated the Eighth
Amendment's ban on cruel and unusual punishment, which is applied to the states through the Fourteenth Amendment. Because treating a prisoner
with cruel and unusual punishment is a constitutional violation under the Fourteenth Amendment, Congress could validly abrogate the states'
immunity.
IV.
Congress passes the Gambling Act, which provides that any state that allows non–Native American gambling must negotiate with any Native American tribes
in the state to give them an opportunity to conduct comparable gaming operations. The Gambling Act authorizes lawsuits by the Native American tribes in
federal courts if the states fail to negotiate with the tribes in good faith. Native American Tribe sues the State of Chance in federal court alleging that the State
did not negotiate in good faith. Can the federal court hear the lawsuit?
A. No. These facts are based on the case of Seminole Tribe of Florida v. Florida, 517 U.S. 44 (1996), in which the Court held that Congress could not use
its Article I powers to abrogate the state's Eleventh Amendment immunity. Seminole Tribe was a 5-4 decision that started the Court's line of cases
limiting Congress' ability to abrogate state sovereign immunity. E Ch. 16-VI(A)(5).

B. NOTE Art. I, §8, cl. 3 gives Congress the authority to regulate commerce with the Native American tribes. This is part of the Commerce Clause of Article
I, §8.
V.
Bard's Bookstores did a lot of business with Shakespeare State University, whose students always need books. Unfortunately Bard's was not so good at
business and went bankrupt. Mr. Cats was appointed supervisor of the bankrupt estate. He sued Shakespeare in Bankruptcy Court, alleging that Bard's had
illegally transferred money to Shakespeare before it entered bankruptcy. Shakespeare argued that the case should be dismissed because of state sovereign
immunity. Should the Bankruptcy Court take jurisdiction of the lawsuit?
A. Yes. In Central Virginia Community College v. Katz, 546 U.S. 356 (2006), a 5-4 decision by Justice Stevens, the Court ruled that Congress has the
power to abrogate state sovereign immunity under the Bankruptcy Clause of Article I, §8, cl. 4, which provides that Congress shall have the power
to establish “uniform Laws on the subject of Bankruptcies throughout the United States.”

B. NOTE:The Bankruptcy Clause is the only Article I power that the Court has allowed to abrogate state sovereign immunity.
C. NOTE:Congress may abrogate state sovereign immunity using its powers under §5 of the Fourteenth Amendment.
VI. The State of New Jersey imposes a special income tax on commuters from other states that results in out-of-state commuters paying a higher income tax to
New Jersey than is paid by New Jersey’s own citizens. The State of Pennsylvania, some of whose citizens pay the commuter tax to New Jersey, has brought
an action against New Jersey in federal court, alleging that the discriminatory tax violates the Privileges and Immunities Clause of Article IV. May New Jersey
prevent the federal court from hearing this case?
A. The Eleventh Amendment probably bars the suit. While the action is between two states, the exception for suits brought against a state by another
state does not apply because Pennsylvania is not suing to redress an injury directly to itself. Even if Pennsylvania may suffer an indirect economic
injury in the form of reduced tax revenues if its citizens pay excessive taxes to New Jersey, this is too indirect an injury to permit the state to sue
New Jersey in federal court.
B. Nor is Pennsylvania likely to be able to sue on behalf of its injured citizens. The New Jersey tax probably does not affect a substantial proportion of
Pennsylvania’s population but rather only those who work in New Jersey. In addition, those Pennsylvania citizens who are injured by the tax may
challenge it before the New Jersey tax authorities. See Pennsylvania v. New Jersey, 426 U.S. 660 (1976).

VII. South Central Bell Telephone Company sued the State of Alabama in an Alabama state court seeking a refund of franchise taxes, claiming that the tax
discriminated against out-of-state corporations in violation of the Commerce Clause (Art. I, §8, cl. 3). Alabama consented to allowing such actions to be filed
against it in state court. The Alabama trial court dismissed the plaintiff’s claim and the Alabama Supreme Court affirmed. The company now seeks review in
the U.S. Supreme Court under 28 U.S.C. §1257. The State of Alabama has objected that the literal language of the Eleventh Amendment bars appellate
jurisdiction because “this case began in state court as a suit brought against one State . . . by citizens of another. . . . ” May the Supreme Court hear the case?
A. The Eleventh Amendment poses no obstacle to the Supreme Court’s hearing the case. Even though the suit could not have been filed in a state
court or in a lower federal court without Alabama’s consent, the Eleventh Amendment does not apply to the Supreme Court’s exercise of
appellate jurisdiction over suits from the state courts. See South Central Bell Telephone Co. v. Alabama, 526 U.S. 160, 165-166 (1999) (refusing to
overturn the Court’s “long-established and uniform practice of reviewing state-court decisions on federal matters, regardless of whether the State
was the plaintiff or the defendant in the trial court”).
VIII. Verizon and MCI entered into a reciprocal compensation agreement for carrying the calls of each other’s Maryland customers. The agreement was approved
by a state agency—the Maryland Public Service Commission (“the Commission”). After the approval, Verizon informed MCI that it would not pay reciprocal
compensation for calls made by Verizon’s customers to the local access numbers of Internet service providers. In response to a complaint filed by MCI, the
Commission ordered Verizon to make the contested payments. Verizon, claiming that the Commission’s order violates federal law, has filed a suit against
the individual commissioners in federal district court, seeking injunctive and declaratory relief. On the assumption that the State of Maryland has not
waived sovereign immunity, is Verizon’s suit barred by the Eleventh Amendment?
A. No. This is a classic example of a case to which the stripping doctrine applies. Verizon’s suit, naming the commissioners individually as
defendants, will be treated as one against the commissioners and not as one against the state. Furthermore, as the Court stated in Verizon
Maryland Inc. v. Public Service Commission of Maryland, 535 U.S. 635 (2002), “In determining whether the doctrine of Ex parte Young avoids an
Eleventh Amendment bar to suit, a court need only conduct a straightforward inquiry into whether [the] complaint alleges an ongoing violation
of federal law and seeks relief properly characterized as prospective.” Id. at 645. Consistent with this statement of principle, Verizon alleges that
the Commission’s order is an ongoing violation of federal law to the extent that it requires Verizon to continue making the contested payments.
As a remedy Verizon seeks the prospective relief of an injunction.
IX. Sally, a lawyer, has sued the justices of her state supreme court under 42 U.S.C. §1983, alleging that the Rules of Professional Conduct that the court
promulgates and enforces to regulate attorney advertising violate the First Amendment. Her complaint seeks damages for lost business, and an injunction
barring enforcement of the rules against her. The justices have moved to dismiss the suit on the basis of common law immunity. How should the court rule
on the motion?
A. The motion should be granted in part and denied in part. Though the defendants are judges, the acts for which they are being sued do not involve
judicial or adjudicative conduct. Instead, to the extent they promulgated rules governing attorney advertising, the judges were performing a
legislative function. As such, they are absolutely immune from civil suit, whether the claim is for damages or for prospective injunctive relief. To
the extent that the justices are responsible for enforcing the rules by initiating proceedings against attorneys, they function in a prosecutorial
capacity, giving them absolute immunity from damages claims but no immunity from injunctive relief. Thus Sally’s claim for damages must be
dismissed, but her request to enjoin future enforcement of the rules against her is not barred by common law immunity. See Supreme Court of
Virginia v. Consumers Union of the United States, 446 U.S. 719 (1980).
Florence was fired from her job with the University of California after the university discovered that she was a member of a radical underground group.
Florence had been working on a project funded heavily by the federal government. In its contract with the federal government, the university agreed that
no one posing a security risk would be allowed to work on the project. Florence has sued the university in federal court under 42 U.S.C. §1983 claiming that
the university violated her First Amendment right of free association. She seeks damages of $250,000. May the federal court hear her suit?

B. The suit is barred by the Eleventh Amendment. Courts have found the university to be part of the state for a number of reasons: The university is
funded by the State of California; it is controlled by a board of regents appointed by the governor; higher education is a function traditionally
assumed by the state rather than by local government; and the state has treated the university as being an arm of the state. See, e.g., Mascheroni v.
Board of Regents, 28 F.3d 1554, 1559 (10th Cir. 1994).
C. Once the university is deemed to be part of the state for Eleventh Amendment purposes, Florence’s suit will also fail on the ground that she has no
cause of action, for neither the state nor a state entity is a “person” who can be sued under §1983. See Feied v. Regents of the University of California,
188 Fed. Appx. 559, 561 (9th Cir. 2006); Thompson v. City of Los Angeles, 885 F.2d 1439, 1442-1443 (9th Cir. 1989).

D.
E. =A

Judicial Review of state court decisions

 Abbey Jones was convicted by a state court of second-degree murder and was sentenced to death under the state’s death penalty statute. On appeal Jones
argued that imposing the death penalty constitutes “cruel and unusual punishment,” in violation of the state constitution and the Eighth and Fourteenth
Amendments to the U.S. Constitution. The state supreme court ruled in Jones’s favor, holding that imposing the death penalty violated both the state and
federal constitutions. The state has petitioned the Supreme Court for review under 28 U.S.C. §1257. Is the state ground of decision adequate?
o First, the state ground of decision fully sustains the result. Regardless of what is said about the Eighth and Fourteenth Amendments, under the
state constitution Jones cannot be executed. And, since the state court has the last word as to the meaning of its own constitution, the Supreme
Court cannot review the correctness of that ruling. Instead, the Supreme Court could reverse the state constitutional ruling only if it
were invalid because it conflicted with the Constitution, laws, or treaties of the United States. That is not the case here. States are free to give
people more rights under state law than they enjoy under federal law. While second-degree murderers may possess no federal right not to be
put to death, the state may give them this right under its own constitution or laws. The state ground of decision is therefore adequate.
 George and Al were the Republican and Democratic candidates for President of the United States in the 2000 election. That election was a very close one. Its
outcome hinged on which candidate was deemed to have won the popular vote in Florida so as to be entitled to that state’s 25 electoral college votes. The
initial Florida vote count showed Al trailing George by fewer than 1,800 votes out of 6 million votes cast. As a result, Al was entitled to a recount under
Florida law. Al raised several challenges to the recount process, but Florida’s secretary of state rejected each of them and, after a partial recount, certified
George as having won the popular vote in Florida. Al sued the secretary in state court. Florida’s high court ruled in Al’s favor and held that under state law,
counties had an extended but limited amount of time to submit their recount returns. The deadline set by the court was such that Florida would still be able
to benefit from a federal “safe harbor” provision, 3 U.S.C. §5, under which Congress would accept the state’s designation of its presidential electors. George
then sought review in the U.S. Supreme Court, even though the Florida decision was based solely on state law. His petition argued that the Florida court’s
decision rested on principles derived from the state constitution, whereas Article II, §1 of the U.S. Constitution provides that the manner for choosing
presidential electors must be determined by a state’s “Legislature.” George also claimed that the Florida court had relied on a novel interpretation of state
law, thus altering the procedure for choosing presidential electors after the election occurred, in violation of 3 U.S.C. §5, which requires Congress to accept a
state’s timely designation of its presidential electors if they were chosen in accord with laws enacted prior to election day. Al urged the Court to deny the
petition, arguing that the state court decision simply reflected “a narrow reading and clarification of state statutes that were enacted long before the present
election took place.” May the U.S. Supreme Court review this case if it wishes to do so?
o Unlike cases in which there are both federal and state grounds for a decision, the Florida court’s decision rests entirely on state grounds. Yet
such a decision is insulated from Supreme Court review only if those state grounds are adequate and independent. As to adequacy, George
contends that the state grounds are contrary to two federal provisions. Even if his arguments may appear strained—e.g., 3 U.S.C. §5 seems only
to address when Congress will accept a state’s designation of its presidential electors, not when a designation is invalid as a matter of law—each
is such that, if accepted by the Court, it would render the state ground contrary to federal law and hence inadequate. The possibility of such a
ruling is itself enough to defeat the adequacy of a state ground.
o As to independence, the state court’s grounds of decision satisfy this requirement if they were based solely on the court’s interpretation of state
law. However, if the state court’s reading of state law was influenced by its understanding of what Florida needed to do to satisfy the federal
“safe harbor” provision, that ground would not be independent of federal law, since a misreading of the federal statute might have affected the
court’s reading of state law. Thus, the state court’s decision arguably did not rest on adequate state grounds; nor is it clear that those grounds
were independent. See Bush v. Palm Beach County Canvassing Board, 531 U.S. 70 (2000).
o In the actual case, the Supreme Court did not address the adequate and independent state ground doctrine as such. However, its opinion noted
that the Florida court’s decision may have violated Article II, §1 (i.e., no adequate state ground) and may have been shaped by that court’s reading
of 3 U.S.C. §5 (i.e., no independent state ground). Yet, it was unclear from the state court’s opinion whether either of these federal questions was
present. The Florida court may have relied solely on state statutes rather than on the state constitution, and while 3 U.S.C. §5 was cited in a
footnote, it may have played no role in the decision. Given this uncertainty, Michigan v. Long would have allowed the Supreme Court to review
the case. Instead, the Court remanded the case so the Florida high court could clarify the bases for its decision. As the Court explained, “After
reviewing the opinion of the Florida Supreme Court, we find ‘that there is considerable uncertainty as to the precise grounds for the decision.’
This is sufficient reason for us to decline at this time to review the federal questions asserted to be present.” 531 U.S. at 78 (emphasis supplied).
By reversing and remanding on this basis, the Supreme Court was able to halt the vote recount in Florida without having to address the merits
of the case. A day after Florida’s high court clarified its decision, the Supreme Court in another case held Florida’s recount procedure to be
unconstitutional and barred the state from continuing with the process. See Bush v. Gore, 531 U.S. 98 (2000).

 Congress, in an effort to streamline the federal judiciary, passes a statute eliminating diversity jurisdiction (i.e., jurisdiction over cases brought by a citizen
of one state against a citizen of another state). The legislation would prevent diversity suits from being brought in federal district court, so that they would
have to be brought in state court. (The legislation does not directly change the Supreme Court’s appellate jurisdiction, so that the Supreme Court can
continue to review the judgments of state courts, including state court suits that turn on a federal question.) Is the congressional legislation constitutional?
__________
o Yes. The Constitution gives Congress full control over the jurisdiction of the lower federal courts. In fact, these lower federal courts do not
even exist until Congress creates them; Article III, Section 1 grants the federal judicial power to the Supreme Court and to “such inferior courts
as Congress may from time to time ordain and establish.” This language has been interpreted to mean that Congress may also define the cases
that may be heard by the lower federal courts, and that Congress may do this by refusing to let the lower federal courts hear cases that fall
within the general federal judicial power (e.g., cases between citizens of different states).
 Presidential power and privilege in domestic affairs
o Frustrated by a lack of cooperation from the Senate, which seems intent on denying him most of his nominees for important cabinet and
judicial posts by not holding hearings and not scheduling votes on them, the President has decided to take matters into his own hands.
Claiming authority under his constitutional duty to ‘‘take care that the laws be faithfully executed,’’ the President begins handing out
‘‘temporary’’ commissions to scores of persons whose offices require the Senate to advise and consent to their nominations. The President had
considered making numerous ‘‘recess appointments’’ under the authority given to him in Article II, § 2, clause 3, but decided against it because
(1) he would have to wait until the Senate went into recess, which it is unlikely to do in anticipation of such a move on the part of the
President; and (2) he is afraid that if the Senate does recess and he makes a massive number of recess appointments, the Senate will retaliate
by not considering any of the President’s pending legislation. There is an act on the books (the Vacancies Act) that limits the tenure of
temporary appointees who have not been confirmed by the Senate. The President, however, makes it clear that his acting appointees will
occupy their positions in excess of the limit set in the Vacancies Act, and will remain until the Senate takes action on the nominees he has sent
to it. If challenged, what should a reviewing court do?

 Invalidate his appointments, because he is acting contrary to the Vacancies Act and because the Senate must otherwise confirm all nominees.
 This question called for the application of Justice Jackson’s test for presidential action from Youngstown Sheet & Tube v. Sawyer. The Vacancies Act represents
Congress’s express opinion that temporary appointees should serve for a limited amount of time; the President’s stated intention to violate that puts him at
odds with Congress. Ascertaining whether he can or cannot make these appointments requires you, under Youngstown, to assess the President’s
power minus that of Congress over the matter. The Constitution gives the President power to appoint, but only with Senate ‘‘advice and consent.’’ Thus the
Constitution requires the branches to share power. Thus, A misstates the role of the President. The ‘‘take care’’ clause would be of no help, because he is
violating an act of Congress, not executing it, by having temporary appointees stay in excess of the time permitted under the Vacancies Act. The President’s
only ability to appoint unilaterally occurs with recess appointments, which the facts indicate the President was not inclined to make. It hardly matters that
the President’s actions were ‘‘like’’ those provided for in the Constitution. Therefore, D does not help much; the fact that there were seizure provisions on the
books that were ‘‘like’’ the one President Truman used did not cut any ice with the Youngstown Court.
 Although B correctly notes the President is opposing Congress here, it is incomplete. Acting contrary to the will of Congress doesn’t automatically result in
the presidential action’s invalidation. Rather, a reviewing Court has to consider the President’s independent power minus that power Congress possesses
over the same subject matter. Here, the President is not only acting contrary to the Vacancies Act, but his unilateral appointments violate Article II’s
assignment of a confirmation role to the Senate as well. Therefore, Cis the best answer.

 Concerned that the ‘‘advice’’ part of the Senate’s advice and consent power has atrophied, Congress passes a law requiring the President to notify the Senate
of all vacancies requiring Senate confirmation and prohibiting the President from naming any person to one of those vacancies until the President, or the
President’s representative, has consulted with the Senate majority leader and the Chair of the Senate committee with jurisdiction over the nominee’s
department. When the President acts to fill a recent vacancy on the Supreme Court without the consultation required by the Act, Senators sue, seeking an
injunction requiring presidential compliance. A reviewing court would probably do what?

o Deny the injunction, because Congress has usurped the President’s constitutional power to appoint, granted in Article II, § 2.
o At first, this might look a lot like the facts in Youngstown. So you might have immediately been drawn to B. But remember that Jackson’s
opinion actually says that when the President bucks Congress, and his power is at its lowest ebb, he can rely only on his independent
power minus constitutional powers of Congress over the matter. Clearly the President has the power to nominate various officials. U.S. Const.,
art. II, § 2. The Senate has the power to offer ‘‘advice and consent’’ to such nominees, but Congress as a whole doesn’t have any role to play
in appointments, which is a presidential prerogative. Its role (actually, the Senate’s role) is limited to offering ‘‘advice,’’ which by definition is
precatory not mandatory, and consenting — or not — to those nominees. Therefore C does not furnish a sufficient reason for a court to grant
an injunction. If Congress has little or no constitutional power, and the President is assigned that role by the Constitution, then the President
has a good argument that Congress is attempting to usurp his authority. As between A and D, then, A is the better answer. Congressional
acquiescence becomes relevant only in the ‘‘zone of twilight’’ where the challenged act does not fall solely within the presidential authority and
where Congress is silent.

The President receives a subpoena to produce certain evidence needed by the prosecution in an upcoming criminal trial. The evidence consists of notes taken
by the President during meetings with his advisers, concerning various issues of domestic politics. It's to be used by the prosecution in a prosecution of one
of those advisers for violating citizens' rights under color of law. Can the President successfully claim that the doctrine of executive privilege prevents him
from being compelled to disclose this information?
o No. Presidents do indeed possess an executive privilege—that is, a right to maintain the confidentiality of communications to which they become
a party during their performance of their official duties. However, this privilege is “qualified,” not absolute. It must yield to the need to develop
all the facts needed for a criminal trial. U.S. v. Nixon, 418 U.S. 683 (1974). That is the situation here, so the President must comply with the
subpoena.

o NOTE:If the communications involved military, diplomatic, or national security secrets, then the executive privilege might be absolute, and outweigh
law enforcement or judicial needs. U.S. v. Nixon. But that's not the situation here. E Ch.8-IV(D).

Does the President enjoy absolute immunity from civil damages in carrying out his official duties?
o Yes. The President has absolute immunity from civil liability for his official acts. This extends to the “outer perimeter” of his authority under the
Constitution. Nixon v. Fitzgerald, 457 U.S. 731 (1982); E Ch.8-IV(C)(2); CH p. 361.

o NOTE:
o But the President does not have immunity—even “qualified” immunity—for acts that are completely unrelated to the carrying out of his job.
See Clinton v. Jones, 520 U.S. 681 (1997) (no immunity for acts taken before President took office, such as the sexual harassment claimed here).

o NOTE:
o Presidential aides generally have only a qualified immunity, such that they can be subject to civil damages if their actions violate statutory or
constitutional rights, which are clearly established, and of which a reasonable person would know. Mitchell v. Forsyth, 472 U.S. 511 (1985); E
Ch.8-IV(C)(3), (4).
o
Suppose that a study has shown that the average American worker puts in four hours of overtime per week, often at the employer’s insistence. One effect of this
practice is to reduce the number of available jobs and to thus increase the nation’s unemployment rate. To combat this practice, the President issues an executive
order barring employers from forcing workers to put in any overtime. The executive order authorizes the Justice Department to enforce the order by obtaining a
court injunction against any employer who violates the order. Is the executive order valid?

o Assuming there is no federal statute imposing such a ban on employers, the President’s action constitutes a clear exercise of lawmaking power.
The President has in effect made a law governing overtime work. Had such a measure been enacted by Congress, the executive’s Article II, §3
duty to “take Care that the Laws be faithfully executed” would have fully justified the President’s instructions to the Justice Department. In the
absence of a congressional statute, however, the President has assumed the dual role of lawmaker and law enforcer, thereby violating separation
of powers.
o One could analyze this problem in either textual or structural/functional terms. A textual argument would urge that by vesting “all” legislative
power in the Congress, Article I precludes the exercise of the lawmaking power by the executive. A structural or functional approach would
argue that the President has sought to aggrandize the executive branch by usurping power that should more appropriately be exercised by the
legislature. One of the fundamental principles of separation of powers is that “[t]here can be no liberty where the legislative and executive
powers are united in the same person, or body of magistrates. . . . ” The Federalist No. 47, at 302 (Madison, quoting Montesquieu) (Clinton
Rossiter ed., 1961).

Suppose that a federal statute authorizes the Interstate Commerce Commission (ICC), in the interest of safety, to issue regulations restricting the use of billboards along
interstate highways. According to the statute, Congress may by concurrent resolution disapprove any such regulations. A concurrent resolution requires the approval of
the House and Senate but does not need to be approved by the President. Pursuant to this statute, Congress adopts a concurrent resolution blocking an ICC regulation
that would have banned all billboards along Interstate Highway 80. Is the concurrent resolution valid under the principles announced in Chadha?

o The concurrent resolution is unconstitutional. It involves legislative action since its purpose and effect are to alter the rights and duties of persons outside
the legislative branch, including advertisers and ICC officials. Bicameralism does not pose a problem. In contrast to the one-House veto struck down
in Chadha, the concurrent resolution involves a two-House legislative veto since it had to be approved by both the House and the Senate. Presentment,
however, was not met because a concurrent resolution is not submitted for approval to the President.

Congress passes the Personal Communications Services Act of 1999. The PCSA authorizes the FCC to award to private applicants, based on competitive bid, various pieces
of radio spectrum for use in a new method of providing cellular phone service. The PCSA provides that any spectrum award by the FCC shall become permanently effective
90 days after it is first made, unless Congress has within the 90-day period enacted a Joint Resolution cancelling the particular award. According to the statute, such a Joint
Resolution is to take effect immediately, without further action by any government official. The FCC then awards a particular spectrum license to Comm Co. Sixty days after
this award, Congress passes a Joint Resolution purporting to cancel the award to Comm Co.
(a) If Comm Co. wishes to attack the constitutionality of the Joint Resolution stripping it of its license, what is the best argument it can make? _________________
(b) Will this constitutional attack succeed? _________________
o
(a) That the Resolution violates the Presentment Clause, by which the President is given the opportunity to veto any bill.
o (b) Yes. The scheme described here is a classic two-house “legislative veto,” under which Congress attempts to keep oversight over administrative action by
reserving the power to cancel that administrative action by means of a Resolution. The scheme here, insofar as it calls for the Joint Resolution to take effect
immediately, deprives the President of his opportunity to veto any bill. Therefore, the scheme is invalid. See INS v. Chadha (The theory behind the invalidity
of the legislative veto is that the Resolution is itself the exercise of legislative power, so it must be carried out by the same pro cedures as for any other
legislative act, i.e., passage by a majority of each house and presentment to the President for his signature.)

Jessica was the White House Press Secretary. After she had served for one year, the President summarily fired her, without giving any reasons. Jessica sued the President
for damages in federal court, alleging that he fired her in violation of a federal statute, Title VII. Jessica’s claim is that the President made sexual advances to her, and
fired her in retaliation for her refusal to entertain those advances. (Title VII does indeed prohibit sexual harassment of the sort charged here.) The President has moved
to dismiss the suit on the broad grounds that he has absolute immunity from civil liability for any official act done by him during his term of office, and that the court
must dismiss the case without considering whether the allegations are true or false. Should the President’s motion be granted? _________________
o Yes. The President does indeed have absolute immunity from civil liability for his official acts. See Nixon v. Fitzgerald. This immunity applies
even if the official act was done illegally or maliciously. If the President did in fact fire Jessica for the reasons she states, the President has
clearly violated Title VII, which bans various forms of sexual discrimination, including sexual harassment. However, since the President was
clearly acting within the “outer perimeter” of his duties as President (that is, when he fired his Press Secretary he was clearly acting within the
bounds of his presidential authority), the illegality of his actions is irrelevant for civil damage purposes. (Observe that if the conduct charged
here had been carried out by any other federal government official, the immunity would have merely been a “qualified” one, since the conduct
charged here violated a “clearly established” right. See Harlow v. Fitzgerald.)

Rise of the administrative state

Congress recently passed an Emergency Economic Stabilization Act that permits the President, through the Secretary of the Treasury, to offer loan guarantees to
businesses facing shortfalls due to economic distress. The Act also instructs the President to suspend such guarantees if, in his judgment, the executives of the business
are possessed of ‘‘unreasonable’’ salary and benefits packages relative to other employees and executives in the industry. When the President suspends loan guarantees
to BigCorp because its executives refused to relinquish their corporate jet, the BigCorp executives sued, claiming that the provision of the Act delegating power to the
President to suspend the guarantees violated Article I, § 7. Which of the following statements is true about the Act?
o The Act is likely constitutional, because Congress may delegate power to the President to exercise according to statutory conditions.
o As we saw in Chadha and in New York v. Clinton, just because an act giving the President (or one house of Congress) the power to do X passes
both houses and is signed by the President doesn’t always mean that the act passes constitutional muster. Both the legislative veto and the line-
item veto initially complied with the requirements of Article I, § 7. There-fore, B is not going to be the correct answer.
o On the other hand, we saw in the line-item veto case that the Court had, in the past, approved of delegation of certain power to the President,
which he could exercise or not following guidelines laid down by Congress. Therefore, C is not correct either. D is simply incorrect: Granting the
President the ability to suspend the loan guarantees — like the ability to suspend the tariff exemption in Field v. Clark, discussed in the line-item
veto case — was a policy choice made by Congress. In the facts presented here, the President was simply exercising the discretion given him by
Congress.
o That leaves A, which is the best answer. As long as delegations to the executive branch include intelligible principles to guide their exercise, they
will be permitted. Even if the guidelines are general or subjective, as they are here, the Court will not disturb those delegations.

‘‘Earmarks’’ are appropriations that members of Congress insert into the annual budget directing dollars into their district for designated uses. Earmarks can take
different forms, including so-called unrequested spending, whereby Congress funds a program at a level higher than that requested in the President’s budget. As part of
earmark reform, Congress has passed the Earmark Accountability Reform (EAR) Act. Henceforth, the President may reduce unrequested spending to the level originally
requested by the President, if, in the President’s opinion, the additional spending is ‘‘wasteful’’ or ‘‘unnecessary’’ and that finding is reported to Congress. Unless
Congress restores the earmarks by majority vote in a resolution that is sent to the President for a veto or signature, the earmarks do not become law. Ames City, the
capital of the State of Ames, is due to receive some unrequested spending for infrastructure development, but the President deems that extra money ‘‘wasteful’’ and so
indicates in a message to Congress. A resolution restoring the money fails and the Mayor of Ames City sues, claiming that the EAR Act is unconstitutional. How would a
reviewing court likely find?

o The Act is unconstitutional because it enables the President to substitute his policy judgment for that of Congress.

o This was a little more difficult, but if you read the description of Clinton v. New York closely, you should have been able to pick the correct answer. Let’s start
with answers easily discarded. Again, B is incorrect, because the question isn’t whether the initial Act complied with Article I, § 7, but whether the Act further
empowered someone to take action in the future that did not comply with Article I, § 7, but should have. Similarly, C is wrong because Presidents exercise
delegated power all the time; only if the delegation is standardless in the literal sense of the word would the Court consider invalidating the Act.
o Between the remaining answers, A is incorrect, because the President isn’t simply declining to spend funds that are appropriated; rather the Act purports to
empower the President to reduce funding in a bill that has been appropriated by Congress. While subtle, the difference is an important one. Had the Act simply
given the President the power to decline to spend all unrequested funding over the administration-requested amount, it would likely pass muster, as the
ability to suspend the tariff exemptions in Field v. Clark did. But the Act did something different: It empowered the President to reduce the actual amount in
the bill. This meant that — unless Congress reversed the President’s alteration through a resolution that was itself subject to presidential veto — the bill that
became law following the reduction would be different than that which was passed by Congress. The President’s judgment would replace Congress’s and, as
in Clinton v. New York, that is unconstitutional. Therefore D is the best answer.

President Cheap Skate receives a detailed bill passed by both Houses of Congress. The bill funds numerous programs, sending money for flood relief to some parts of the
country and funding for farmers to other states. President Skate reads through the bill and decides that farm programs in the Northeast are overfunded compared to the
rest of the nation. He takes out a big black Magic Marker and crosses out the lines of the bill that give money to vegetable farms in Noo Yawk and Road Isle. Then he signs
the bill so that the rest of the bill's provisions become law. Is the President's action constitutional?

o No. The President has attempted a line-item veto. The Court has ruled that line-item vetoes violate Separation of Powers even when they are authorized by
congressional legislation. Clinton v. New York City, 524 U.S. 417 (1998).

What clause of the Constitution does the line-item veto violate?

o The Presentment Clause, Article I, §7, cl. 2, which states that after a bill is approved by both Houses it shall be presented to the President; “if he approve he
shall sign it, but if not he shall return it.” The line-item veto violates this procedure because the veto occurs after the bill is signed into law and the cancellation
applies to only part of the bill. The Presentment Clause requires veto of the entire bill. E Ch.8-I(C)(2)(b).

What is a legislative veto?


o A legislative veto occurs when Congress delegates power to an executive agency or the President, but retains the right to veto actions of the agency or the
President. A legislative veto undermines the executive right to veto, and thus violates the Separation of Powers doctrine. Imconversion & Naturalization Service
v. Chadha, 462 U.S. 919 (1983); E Ch.8-I(C)(3).

o EXAMPLE:Congress grants the President the power to promulgate regulations concerning imconversion that are to have the force of law. However, Congress
requires the President to give it the proposed regulations before they take effect, so Congress can approve or disapprove them. This is an impermissible legislative
veto.

Congress has established the Federal Truck Safety Board (FTSB). In the statute setting up the FTSB, Congress has provided that the Director of the FTSB shall be appointed
by the President, without the need for House or Senate confirmation. The enabling legislation also provides that at any time, without cause, the House and Senate, acting
together in the form of a Joint Resolution, may remove the Director. The Director is given certain powers, including the ability to issue an order suspending certain types
of truck traffic if the Director concludes that the suspension is needed to protect the safety of interstate commerce.
In the 30 years since the FTSB was set up, Congress has played no role in the appointment or removal of the FTSB’s Director. The incumbent, Derrick, was appointed 10
years ago. Recently, there have been a rash of accidents on the highway involving trucks of more than six feet in width. Derrick has decided that such wide trucks are
dangerous, and has, therefore, issued an order suspending any truck of that width or greater from moving on the federal highways across a state border. Truck Co., owner
of many extra-wide trucks, wishes to challenge the constitutionality of Derrick’s action.
(a) What is the best grounds on which Truck Co. can challenge the constitutionality of Derrick’s order? _________________
(b) Will this attack succeed? _________________

o Answer
o That by retaining the right to remove Derrick, Congress has improperly vested executive functions in its own agent.
o Yes, probably. Congress and its agents can only exercise “legislative power,” not “executive power.” The FTSB Director’s powers are clearly executive — the
Director is carrying out the laws (by determining what is required for safety) rather than “formulating” the laws. Congress, by retaining the right to remove
the Director without cause, converted the post of Director into an agent of Congress, thus in effect taking executive powers onto itself. Therefore, Derrick is
not permitted to exercise his statutory powers, including issuance of the order. See Bowsher v. Synar, striking down certain powers of the Comptroller General
on the similar analysis that Congress improperly converted him into an agent of Congress by retaining the power to remove him.

Executive powers over foreign affairs

The President has entered into an executive agreement with the Government of Venelivia, settling claims between that government and several companies whose assets
were seized by Venelivia when their operations were nationalized. As part of the agreement, the President agrees to order the transfer of property held in U.S. banks,
which had previously been frozen. The State of Ames, in which some Venelivian property is held, refuses, citing a state law prohibiting the transfer of foreign assets to
countries who seize property of Ames corporations without paying fair market value. (One of the businesses nationalized by the Venelivian government was based in
Ames.) The Ames Attorney General sues to prevent the transfer of assets in federal court, alleging that the executive agreement is unconstitutional. What should the
judge do?

o Dismiss the case, because the President may enter into agreements settling foreign claims that trump state law.
o The facts here recall those in Pink and Belmont, where the U.S. government assisted the Soviet Union in recovering property held in the United States
pursuant to an executive agreement extending diplomatic recognition to the Soviets. In the face of the conflict between the executive agreement and the
Ames statute — at least where the President is attempting to settle claims between private citizens and a foreign state — the executive agreement will
trump state law. The President may do this via an executive agreement, as opposed to a formal treaty, so C is incorrect. Moreover, the President need not be
specifically authorized by Congress to enter into such agreements; therefore D is incorrect as well. And although the President does indeed have broad
powers in foreign affairs matters, A is not the best answer, at least as compared to B, which correctly states the rule of Belmont and Pink.

As part of the United States’ settlement of outstanding claims with Venelivia, the President enters into another executive agreement establishing an arbitral panel for the
resolution of future claims between American citizens and the Venelivian government. The agreement recites that it is the policy of the U.S. government that all claims be
settled through the panel, as opposed to ordinary litigation in state or federal courts. After Venelivia nationalizes BigOilCo, a corporation with its principal place of business
in the State of Ames, BigOilCo files suit, taking advantage of a new Ames statute enabling citizens who have property nationalized overseas to receive treble damages if
they can prove they received less than fair market value for their property. The United States intervenes, arguing that the executive agreement preempts the Ames law as
applied to disputes with Venelivia. Which of the following statements is true?

o the statement of policy is sufficient to preempt state law


o
The twist here is that although the executive agreement itself does not expressly preempt contrary state law, an accompanying ‘‘policy statement’’ included
in the agreement does. Again, though, at least where claims settlement is involved, the Garamendi case teaches that such a statement will suffice to preempt
state law. Therefore C is the correct answer.
o A is an incorrect statement of the law — the agreement doesn’t have to specifically preempt state law. The executive agreement
in Garamendi explicitly disclaimed preemptive effect. B, too, is incorrect because the President can enter into executive agreements without congressional
authorization. D, however, goes too far: It is clear after Medellin that not all state laws can be preempted as a matter of presidential prerogative, even where
foreign affairs are involved.

Concerned about growing violence among drug gangs in Mexico, the United States and the government of Mexico enter into a series of executive agreements intended to
reduce violence along the border, including measures to prevent guns from making their way from the United States across the border into Mexico. In connection with
the executive agreement, the President sends a memorandum to the Governor of Texas ordering him to confiscate weapons in certain border towns where violence has
escalated. Those who are lawfully possessing arms will have the opportunity to demonstrate that and will have their weapons returned. Assume for purposes of this
question that Texas generally permits lawabiding citizens to possess guns and even carry them concealed on their person if they have a license to do so, which is
available as a matter of right to citizens 21 or older who have passed a safety course and have not been convicted of certain violent crimes. The Governor sues to prevent
enforcement of this executive policy. What should a reviewing court do?

o Side with the Governor because the policy embodied in the memorandum cannot preempt state law.

o
This question furnishes a good illustration of the point made earlier — not every state law is vulnerable to preemption by a mere statement of executive
policy. Although some broad language in Garamendi suggested otherwise, the Court made clear in Medellinthat executive branch power, even in foreign affairs,
did not extend that far. Medellin seemed to limit Garamendi to the settlement of private claims between countries and American citizens. Here, however, the
executive branch is attempting to override Texas’s gun laws on the strength of an executive policy statement. Thus, A doesn’t state the law here accurately.
o Likewise, B is inaccurate. The Court has never held that federalism is absolutely irrelevant in foreign affairs disputes. But C is incorrect, too: Presidents don’t
need the explicit approval of Congress to enter into executive agreements. That leaves D, which is the correct answer after Medellin.
o
What are the President's powers in the area of foreign affairs?
To:
1. Recognize (and withdraw recognition of) foreign governments;
2. Receive ambassadors and other public ministers; and
3. Act on the nation's behalf in day-to-day dealings with foreign governments.

NOTE:
These are exclusive powers. The President shares with Congress the power to determine foreign policy (although the President has a distinct advantage, in that he controls
information and can respond quickly to developments abroad). Remember that the federal government enjoys exclusive control over foreign affairs; the states have none.
E Ch.8-II(A); CNR §6.1.

What powers does the President enjoy as Commander in Chief of the armed forces?
It's not entirely clear, but such powers do include:
1. Committing armed forces to repel a sudden attack on the United States itself (or any internal insurrection), even before Congress declares war;
2. Legislative power in “theaters of war” (e.g., establishing military governments in occupied territories); and
3. Controlling the disposition (placement) of armed forces.

NOTE:
The President does not have the power to declare war; that power belongs to Congress (although, in his role as Commander in Chief of the armed forces, the President
may conduct military activity in actual hostilities against the United States without such a declaration, under Article II, §2). E Ch.8-II(B); NR pp. 261-264; CNR §6.8.
o
In response to the terrorist attacks of September 11, 2001, President George W. Bush issued an executive order providing that non-U.S. citizens who were
suspected of aiding the war on terrorism would be tried by special military commissions. The president's tribunals offer less protection to defendants than
the procedures established by a congressional statute called the Uniform Code of Military Justice. Hamdan, a citizen of Yemen, was arrested in Afghanistan
and charged with assisting Al Qaeda with planning the 9/11 attacks. Can Hamdan be tried for war crimes before one of President Bush's military tribunals?
o No. Justice Stevens' opinion for the Court in Hamdan v. Rumsfeld, 548 U.S. 557 (2006), concluded that Congress had not authorized the tribunals and that the
President did not have independent authority to create them. According to Justice Kennedy's concurrence, because Congress had already passed legislation
setting the scope of military tribunals, the President acted in opposition to existing law—i.e., in Category 3 of the Steel Seizure categories, where the President's
authority is at its lowest ebb. E Ch.8-I(B)(3)(b), Ch.8-II(B)(2).

o NOTE:
o Once Congress authorizes military commissions, the President acts within his powers to enforce them, as long as they do not violate other constitutional
provisions.

In response to the Second World War, Congress passed the Non-Detention Act, which states “no citizen shall be imprisoned or otherwise detained by the United States
except pursuant to an Act of Congress.” Harry, a U.S. citizen, was captured in Afghanistan and detained by the executive branch as an “enemy combatant.” Does the President
have the authority to detain Harry?

o No. According to Justice Jackson's test, the President's authority is “at its lowest ebb” when he acts in contradiction of the express or implied will of Congress.
Here the Congress has prohibited the detention of citizens without congressional authorization. E Ch.8-I(B)(2)(b).

In response to the Second World War, Congress passed the Non-Detention Act, which states that “no citizen shall be imprisoned or otherwise detained by the United States
except pursuant to an Act of Congress.” In response to the terrorist attacks of September 11, 2001, Congress passed the Authorization for Use of Military Force (AUMF).
Hamdi, a U.S. citizen, was captured in Afghanistan and detained by the executive branch as an “enemy combatant.” Does the President have the authority to detain Hamdi?
o Yes. A plurality of the Court in Hamdi v. Rumsfeld, 542 U.S. 507 (2004), ruled that the AUMF provided congressional authorization for the President's detention
of Hamdi, and that the President had not violated the Non-Detention Act.
o NOTE:Justice Thomas, the fifth vote to uphold the president's power, wrote that the President had inherent power as Commander in Chief to detain U.S. citizens
as enemy combatants.
o NOTE:Justice Scalia, joined by Justice Stevens, concluded that a U.S. citizen cannot be held by the government without charges or trial unless Congress suspends
the writ of habeas corpus.
o NOTE:The Court also ruled that Hamdi must be accorded some form of due process. Hamdi v. Rumsfeld, 542 U.S. 507 (2004); CH pp. 377-378.

What is a writ of habeas corpus?

o Habeas corpus is Latin for “you have the body.” A writ of habeas corpus is used by a prisoner to challenge the government's imprisonment of the prisoner's
body. The writ forces the government into federal court to explain or justify why the prisoner is being held. If the judge decides the prisoner is being held in
violation of the Constitution or federal law, the judge may order release so that the government no longer has the body. E Ch.8-II(B)(2).

Congress passed the Military Commissions Act (MCA), eliminating the jurisdiction of federal courts to hear habeas petitions from detainees at Guantanamo Bay in Cuba.
The government claims that noncitizens detained as enemy combatants in territory outside U.S. borders have no constitutional rights or privilege to habeas corpus. Boum,
an Algerian national who has been declared an enemy combatant and held at Guantanamo Bay, challenged his denial of habeas. Who should win?
o Boum. The Court in Boumediene v. Bush, 128 S. Ct. 2229 (2008), held by a 5-4 vote that even foreign nationals detained in Guantanamo Bay have a
constitutional right to habeas corpus review. The Suspension Clause of Article I, §9, cl. 2 has full effect at Guantanamo. Justice Kennedy's opinion concluded
that the jurisdiction-stripping of the MCA violated the Suspension Clause. Boumediene v. Bush, 128 S. Ct. 2229 (2008); SCW pp. 22-39; E Ch.8-II(B)(2).

o NOTE:Congress must act in accordance with the formal requirements of the Suspension Clause to deny the writ, which the Military Commissions Act
in Boumediene failed to do.
o NOTE:Chief Justice Roberts' dissent argued that Congress had provided an adequate substitute for habeas with the tribunals of the MCA and therefore had not
suspended habeas corpus in violation of the Suspension Clause

Why did the writ of habeas corpus apply in Guantanamo Bay, which is not part of the United States?

o According to Justice Kennedy's opinion in Boumediene, the base at Guantanamo Bay was “within the constant jurisdiction of the United States,” which
“maintains de facto sovereignty over this territory.” Boumediene v. Bush, 128 S. Ct. 2229 (2008); E Ch.8(II)(B).

Relations have been strained between the United States and the country of Iguana ever since a former U.S. President insulted Iguanians by appointing a former child movie
star as ambassador to Iguana. Iguana has, for some time, severely restricted the econversion of Iguanians to the United States. To ease the tension, the President, with the
advice and consent of the Senate, enters into a treaty with Iguana, permitting the citizens of each country unrestricted travel between the two. This causes a mass exodus
from Iguana to the United States. In response, Iguana clamps down and forbids an Iguanian family from emigrating to the United States. The President terminates the
treaty because of this breach, without consulting the Senate. Is the termination valid?

o Yes. Although treaties may be created only with the advice and consent of the Senate, the President may, himself, continue or terminate a treaty without the
Senate's consent. In fact, the President's termination of a treaty would be considered a non-justiciable political question, thus removing any judicial roadblocks
from a President's power to nullify treaties. Goldwater v. Carter, 444 U.S. 996 (1979); E Ch.5-III(B), Ch.16-VII(E)(1); CH pp. 372-373.

On March 1, at the request of the prime minister of France, the President of the United States sent armed forces to a French island in the Pacific that was under attack by
Cuban military forces. The President gave Congress no advance warning of his decision to employ U.S. troops. The President later defended the decision on the ground
that an 1882 treaty between the United States and France bound each nation to assist the other in repelling an attack upon its territory or possessions. Was the
President’s action constitutional?
o
Answer
o The President’s unilateral decision to initiate hostilities with Cuban military forces in the Pacific would appear to have usurped Congress’s authority to decide
whether or not the United States should engage in war. Even if the Cuban attack on the French island was sudden and unexpected, it did not constitute a
sudden attack on the United States, its territories, or armed forces. The situation therefore does not fall within the emergency exception contemplated by the
Founders and recognized by the Court in The Prize Cases.
o The President’s reliance on the treaty with France is misplaced. If a treaty purported to give the President the authority unilaterally to commit the United
States to war, the treaty would be unconstitutional. Article I, §8, cl. 11 assigns the power to declare war to Congress—i.e., to the House and the Senate. A treaty
confirmed by the Senate alone cannot take the place of a congressional declaration of war. Many U.S. mutual defense treaties respect this fact. For example,
the Southeast Asia Collective Defense Treaty, September 8, 1954, Art. 4, 6 U.S.T. 81, 83, committed the United States to respond to an attack upon one of the
signatories “in accordance with its constitutional processes.”
o Under the War Powers Resolution, the President was required to notify Congress by March 3 that he had sent U.S. troops into hostilities. Moreover, whether
or not he gave such notice, the 60-day clock for withdrawal began to run on that date. The President, therefore, had to remove the troops by May 2, unless
“unavoidable military necessity” required an extra 30 days to effect a safe withdrawal, in which case U.S. armed forces would have had to be removed within
90 days—i.e., by June 1—unless Congress in the interim had authorized the President’s action or extended the period during which troops could remain there.

Suppose that federal law imposes a quota on the number of Canadian immigrants who may apply for United States citizenship each year. If the President wishes to
increase this quota, she could seek legislation to this effect from Congress. However, if a majority of the House of Representatives is opposed to the President’s plan, a bill
to increase the Canadian quota would fail. Could the President instead accomplish her objective by invoking the treaty power?
o
If the President has the support of two thirds of the Senate, a treaty with Canada increasing the quota would win Senate ratification. Yet should the President
decide to proceed by treaty, rather than by seeking new legislation from Congress, the House of Representatives will have been completely shut out of the
process. This is troubling in light of the fact that Article I, §8, cl. 4 gives Congress the power “[to] establish an uniform Rule of Naturalization”; by depriving
the House of any role whatsoever in changing this law, the President has arguably violated the principle of separation of powers.
o As intuitively appealing as this argument may be, it will likely fail. If the argument were sound, a great many treaties would be invalid, for they deal with
matters that also fall within the scope of Congress’s legislative powers under Article I. The fact is that treaties frequently operate as a substitute for
congressional legislation. This overlap in subject matter is suggested by the fact that, on occasion, after a President has failed to win ratification of a treaty
from the requisite two thirds of the Senate, the executive has achieved the same goal through a statute or joint resolution that needs only a simple majority
in each House. The potential overlap is also evident from the fact that treaties and statutes sometimes conflict with one another, a phenomenon that could
not occur if the two operated in mutually exclusive realms. The Court has never held that a treaty is invalid simply because it governs an area that Congress
could have regulated under one of its enumerated powers. See Edwards v. Carter, 580 F.2d 1055 (D.C. Cir.), cert. denied, 436 U.S. 907 (1978) (treaty
transferring property to Panama was valid even though Article IV, §3 expressly gives Congress the power to dispose of property belonging to the United
States).
o Yet a good argument can be made that some functions that the Constitution has assigned to Congress cannot be performed by treaty, at least in the absence
of congressional authorization. For example, Article I, §9, cl. 7 states that “[n]o money shall be drawn from the Treasury, but in Consequence of Appropriations
made by Law”; a treaty that sought to appropriate funds without an appropriation by Congress would likely violate this textual limitation. Similarly, a treaty
that purported to make certain conduct a crime would probably violate the Due Process Clause of the Fifth Amendment; that clause was intended to
incorporate the Magna Carta’s prohibition against the Crown’s depriving a person of life, liberty, or property except according to the common law or pursuant
to an act of Parliament. See Davidson v. New Orleans, 96 U.S. 97, 101-102 (1878). It has likewise been argued that a treaty may not substitute for a congressional
declaration of war, for this particular power—unlike the power over naturalization considered in Example 7-Q—was given to Congress in order to prevent
the President from taking the nation into war without the people’s consent. To permit the President and the Senate alone to declare war would circumvent a
vital check on the war power.

In 1978, when President Carter recognized the People’s Republic of China (Peking) as the sole government of China, he announced that he was terminating the 1954
Mutual Defense Treaty between the United States and the Republic of China (Taiwan). Members of Congress filed suit in federal court challenging the President’s action,
claiming that a President may abrogate a treaty only with the consent of two thirds of the Senate or a majority of both Houses of Congress. Was the President’s action
constitutional?

o Answer
o Though Article II, §2, cl. 2 defines the procedure for making treaties, the Constitution is silent on the question of how a treaty may be terminated. One might
contend that the procedure employed to abrogate a treaty should mirror that used to enter into a treaty. On the other hand, it is arguable that the Constitution’s
failure to require any Senate or congressional approval for the abrogation of a treaty means that this authority rests with the President, as part of the executive
power. Historically, the United States has terminated treaties in a variety of ways, including by statute directing the President to send notice of termination;
by statute without separate presidential notice; by the President with the approval of both Houses of Congress; by the President with the consent of the
Senate; and by the President alone.
o In Goldwater v. Carter, 444 U.S. 996 (1979), the Supreme Court dismissed a challenge to President Carter’s unilateral termination of the 1954 treaty with
Taiwan, ruling that the case was not justiciable. Of the six Justices who voted to dismiss the case, four concluded that the issue of how a treaty may be abrogated
poses a political question, another Justice found that the challenge was not ripe, and the sixth Justice did not disclose his reasoning. The issue thus remains
an open one, to be resolved by the political branches themselves until such time as the Court may choose to resolve it.

May the President enter into an executive agreement with France to conduct joint naval exercises in the Pacific or to designate the location of a U.S. consulate in France?
o
Since Article II, §2 designates the President as “Commander in Chief of the Army and Navy,” she possesses the independent power to arrange for naval
exercises. The President is accordingly free to exercise this power through an executive agreement rather than through a treaty that would give the Senate
the ability to block her action. The President has neither usurped the Senate’s authority nor aggrandized the executive sphere.

The Caribbean island of Grenoble is a small but strategically important ally of the United States. Grenoble holds a popular election, but the military refuses to allow the
democratically elected President to take office. The military then begins to seize the property of American businesses located in Grenoble. The President of the the U.S.,
concerned that U.S. strategic interests are endangered, and invoking his powers as Commander in Chief, sends 20,000 U.S. troops onto the island. This action is taken after
the President confers with congressional leaders, but with no other congressional action. With Congress still inactive, fighting continues for six months, due to the Grenoble
military’s well-entrenched status. A member of Congress, Senator Piper, then brings suit for a declaratory judgment that the President has acted unlawfully in using U.S.
forces in this manner.
(a) What is the best argument that Senator Piper can make against the constitutionality of the President’s actions? _________________
(b) Will Piper’s attack succeed? Assume there’s no existing relevant federal legislation. ____________

o Answer
o That it violates Congress’ sole power to declare war.
o Yes, probably. It is true that Article II, Section 2, explicitly grants the President the power of Commander in Chief of the U.S. Armed Forces. However, the
President must use this power subject to oversight by Congress. In particular, the power to “declare war” is given solely to Congress, in Art. I, Section 8. While
the President may, without a declaration of war, probably commit our troops to repel an immediate emergency, it is very unlikely that the President may
wage a prolonged ground war, without a declaration of war, especially where the United States has not been directly attacked.
o Also, the 2006 decision in Hamdan v. Rumsfeld indicates that in cases where the President asserts broad power to act in wartime, and it is not clear that
Congress has acquiesced to what the President is doing, the Court will favor Congress over the President. So here, probably the Court will say that the
President’s action violates Congress’ sole power to declare war.

Chapter 4: Limits on State Regulatory and Taxing Power


Preemption doctrines

During the late 1990s, a number of states and municipalities passed laws and ordinances aimed at putting economic pressure on certain rogue regimes around the
world. In many cases, Congress, too, had imposed sanctions on these same countries. In 1998 both Congress and the State of Massachusetts passed laws relating to the
country of Burma. Congress authorized the President to take certain actions to ensure a peaceful transition to democracy in Burma, but it stopped short of prohibiting
U.S. companies from doing business in the country or with that country’s military government. The U.S. law was otherwise silent on the extent to which states could
impose sanctions as well. Massachusetts, on the other hand, barred companies that did business in that country from bidding on contracts with state or local
governments. The Supreme Court struck down the Massachusetts law. Based on what you read above, what would you guess was the basis for its conclusion?

o Massachusetts’s law posed an obstacle to Congress’s legislative aims.


o Rationale for Correct Answer
o I wanted to see how well you’d been paying attention. First, the facts given help you eliminate three answers right off the bat, leaving you with the correct
one. Because Congress has acted, the DCCD is not at issue here (although, as an alternative ground for its decisions, lower courts found that the Massachusetts
law did violate the DCCD as well as being preempted). Therefore A can be eliminated.
o Moreover, the facts tell you that the federal law was silent as to its preemptive intent. Because express preemption will involve explicit language in the statute
barring complementary state laws, the lack of such language here means you can strike B as well. Moreover, nothing in the facts indicates that companies
subject to both laws couldn’t comply. Federal law prescribed some sanctions; Massachusetts’s law went further. But it was not impossible to comply with
both regimes according to the facts, so C can be eliminated, too.
o Congress had stopped short of banning companies from doing business in Burma because it didn’t want to disadvantage existing U.S. companies who had
contracts and investments in place in the country. Moreover, Congress hoped to provide the President with a combination of diplomatic carrots and sticks
that he could employ as the facts warranted. Because the state law here prohibited business or investment if one wished to do business with the state, the
Court concluded that Massachusetts had chosen obduracy and inflexibility where Congress had opted for more room to maneuver. The state’s law, the Court
concluded, posed an obstacle to congressional aims and was thus preempted. See Crosby v. National Foreign Trade Council, 520 U.S. 363, 372 (2000). D,
therefore, would be the correct answer.

Arizona passed a controversial immigration bill. One of its provisions, section 5(C), made it illegal for an illegal immigrant to apply for work, solicit work in a public
place, or perform work as an employee or an independent contractor. Federal law had no comparable federal penalties, owing to Congress’s decision not to impose
criminal penalties on those — even those in the country illegally — who were looking for work. Which of the following statements is true?

o This question is based on the law invalidated in part in Arizona v. United States, 132 S. Ct. 2492 (2012). In the case, the Supreme Court invalidated section
5(C), along with another provision, Section 6, which authorized state officers to arrest, without a warrant, any person the officer has a probable cause to
believe has committed a public offense that would make him removable from the country. Arizona ¸ 132 S. Ct. at 2506.
o Examining the federal immigration law, the Court concluded that § 5(C), which “enacts a state criminal prohibition where no federal counterpart exists,” was
preempted because “Congress made a deliberate choice not to impose criminal penalties on aliens who seek, or engage in, unauthorized employment.” Id. at
2503, 2504. Permitting that provision to stand “would interfere with the careful balance struck by Congress with respect to unauthorized employment of
aliens” therefore the law “is an obstacle to the regulatory system Congress chose.” Id. at 2505.
o But you need not have known about the case, or the Court’s holding, to reason to the correct decision. The facts tell you that the state is regulating where
there is no comparable federal law. Therefore, B could not have been correct, because “field preemption” occurs only when the federal government has
regulated in an area comprehensively, leaving no room for supplementary regulation by states. Nor is C correct: If there is no federal law, it is possible to
comply with one regime without violating another. Between the remaining answers, A is a better answer than D. Recall that the facts state the decision not to
criminalize the act of looking for or securing employment, even by those in the country illegally, reflected a choice on Congress’s part. As the Court pointed
out, to uphold the state law would upset the balance Congress struck; therefore, state law had to give way.
o You might be interested to know that Section 6 was likewise preempted in the Arizona case. It, the Court pointed out, “attempts to provide state officers even
greater authority to arrest aliens on the basis of possible removability than Congress has given to trained federal immigration officers.” Id. at 2506. That it
could be exercised without input from the Federal Government, in the Court’s opinion, “would allow the State to achieve its own immigration policy.” Id.
Allowing it to stand “violates the principle that the removal process is entrusted to the discretion of the Federal Government.” Id.

The Food and Drug Administration establishes standards for the collection of blood plasma. Transylvania County in the State of Silvercross has an ordinance that imposes
additional regulations on collecting blood plasma. Dracula Enterprises, a company that collects blood plasma, claims the federal standard preempts the county standard.
Does it?
o No; the state statute is valid.
o The issue here is whether the federal standard preempts the field of blood plasma collection, as there's no direct conflict between the two statutes. If a subject
area is one that has traditionally been subject to mainly local rather than national regulation, preemption is unlikely. Here, the field is one historically left to
the states—health and safety regulation. Thus, a court would be unlikely to find preemption (absent a clear expression from Congress that it intended to
preempt the entire area). Hillsborough County, Fla. v. Automated Medical Labs, 471 U.S. 707 (1985); E Ch.6-III(C)(1)(a)(ii).

o RELATED ISSUE:
o The converse is also true; a federal statute in an area of historically national control (e.g., patents, trademarks, imconversion, bankruptcy) will generally
preempt a related state statute. E Ch.6-III(C)(1)(a)(iii); CH p. 402.

Congress passed legislation imposing limited sanctions on the foreign Nation of Apathy to encourage it to act more democratically. The U.S. State of Indignation is critical
of the numerous human rights violations committed by Apathy. State of Indignation therefore passes legislation prohibiting the state and its agencies from buying goods
or services from companies that do business with Apathy. Is the state law valid?

o No. Even though Congress and the State of Indignation have similar goals, Congress' action preempts the state from passing sanctions. In the area of foreign
policy, the Court is likely to find state action preempted by Congress, even where, as here, Congress did not expressly preempt the state legislation and the
state and federal goals are consistent. The Court unanimously found preemption on similar facts in Crosby v. National Foreign Trade Council, 530 U.S. 363
(2000); CH p. 405.

State of Justice passes the Holocaust Victim Insurance Relief Act, which requires any European insurance company doing business in state to disclose whether it issued
insurance policies during the Holocaust. If the European insurance companies fail to comply with the law, they will lose their licenses to do business in the State of Justice.
There is no federal statute on this topic. Is the Holocaust Act valid?

o No. In a 5-4 decision, the Court ruled that a similar state statute interfered with the President's ability to negotiate executive agreements in this area of foreign
policy and therefore was preempted. Even though the President had chosen not to pursue a policy in this area, the state was not allowed to enact a contrary
policy because any such policy would interfere with the President's choices. American Insurance Ass'n v. Garamendi, 539 U.S. 396 (2003); CH pp. 405-406.

The State of Warner is one of the few with a population of the rare yellow-bellied sapsucker. A state statute forbids hunting and trapping yellow-bellied sapsuckers and
authorizes game wardens to seize and destroy any traps they find. A federal consumer safety regulation regulates the design of such traps to prevent them from pinching
the fingers of people who set the traps. Does the federal regulation preempt the state statute?
o No, because the regulations have different purposes, a fact that indicates that the two regulations can co-exist.
o When federal and state laws conflict, the federal law takes precedence under the Supremacy Clause. If there's no direct conflict, the state law is only preempted
if the federal law was intended to preempt the entire field. Here, there's no conflict at all. Instead, the laws have two very different purposes—the federal law
was designed to make the traps safe, and the state law was intended to protect the animals. The state law does not impede operation of the federal law, and
there's no indication the federal law was intended to preempt the entire field of yellow-bellied sapsucker traps. As a result, the state law will stand.

o NOTE:
o If the objectives behind the two regulations were incompatible, the state regulation would fail, even if the state and federal regulations didn't conflict on their
face. E Ch.6-III(C)(2)(b); CH §5.2.5.

The Federal Motor Vehicle Safety Act provides, in part, that car companies may use seat belts to meet federal occupant crash protection standards. Furthermore, the Act
provides that “no state shall have any authority to establish, or to continue in effect, any motor vehicle safety standard that is not identical to the federal standard”
applicable to the same aspect of vehicle performance. Quasimodo is disfigured in a car accident. He brings a product liability suit in state court against Field Marshal
Motors, the manufacturer of his car, alleging a design defect in that his car lacked air bags. Under state common law the lack of air bags could constitute a design defect.
Does the federal law preempt the state law?

o Yes, because the federal law expressly covers the entire area of crash protection.
o Here, the state common law would require safety measures in addition to the federal requirements. The federal law is satisfied by seat belts; the state common
law might require air bags. While there's no direct conflict, the state law will nonetheless be invalid if Congress intended to preempt the entire field. That's
the case here, because the federal law expressly requires that state law mirror the federal standard applicable to the same aspect of vehicle performance. As
a result, the federal law will preempt the state law. Cox v. Baltimore County, 646 F. Supp. 761 (D. Md. 1986); Wood v. General Motors, 865 F.2d 395 (1st Cir.
1988); Taylor v. General Motors, 875 F.2d 816 (11th Cir. 1989); E Ch.6-III(C); CH §5.2.2.

Tired of competing with Japanese car companies, car makers in Detroit persuade the State of Michigan to declare war on Japan. The state militia loads Lincoln Continentals
into portable rocket launchers and aims them at Tokyo. The federal government has not declared war on Japan and has no intention of doing so. Is Michigan's declaration
of war “preempted”?

o Yes, because states have no power to declare war.


o Preemption is an issue whenever the federal government and a state regulate the same matter, or a state regulates something it has no authority to regulate.
When the Constitution prohibits state action in an area, any state regulation in that field is void. Such areas include coining money, foreign affairs, and—as
here—declaring war. Because Michigan is constitutionally barred from declaring war, its doing so is preempted. E Ch.6-III(C)(1)(a)(iii); CH pp. 402-403.
Congress passed the Clean Water Act (CWA) to protect “the navigable waters of the United States, adjoining shorelines and other natural resources of the United States”
against harm from environmental accidents. To protect those waters and resources, the CWA assigns a monetary penalty not to exceed $10,000 per day against any person
or corporation that is responsible for an oil spill onto the waters of the United States. Big Oil Company negligently spilled oil into the water surrounding the State of Salmon.
The fishermen of the State of Salmon, whose jobs were affected by the spill, and their fellow citizens, whose health was affected by the spill, sue Big Oil for their economic
loss and personal injury. Are their lawsuits preempted by the Act?

o No. In the real case about the Alaska oil spill, the Court ruled that Congress had not expressed any intent to occupy the field in the Clean Water Act. Nor was
there any reason for the Court to conclude that Congress would have any interest in preempting state tort law, which compensates injuries to persons, in an
act whose goal was to protect natural resources. Exxon Shipping Co. v. Baker, 128 S. Ct. 2605 (2008).

Art Beats suffered a heart attack while running a marathon. At the hospital a balloon catheter, which is a medical device manufactured by Medtrials, Inc., was inserted into
Art's heart by a surgeon. Later the balloon catheter popped and Art died. His family sued in state court under state tort law arguing that the balloon was defectively
designed and labeled. The federal Medical Devices Act (MDA) provides that once a medical device receives premarket approval from the Food and Drug Administration
(FDA), “no state or local government may impose any requirement that either relates to the safety or effectiveness of the device, or that is different from, or in addition to,
any FDA requirement applicable to the device.” Medtrials' balloon catheter received premarket approval from the FDA. Is Art's lawsuit preempted?
o Yes. The Court ruled 8-1 in Riegel v. Medtronic, 128 S. Ct. 999 (2008), that a similar lawsuit was expressly preempted by the MDA. E Ch.6-III(D)(1).

o NOTE:
o Express preemption is present when the law specifically or expressly says that state or local law is preempted. Contrast express with implied preemption,
where Congress' action suggests preemption. The two main types of implied preemption are field preemption and conflicts preemption. E Ch.6-III.

Suzi Queezi was feeling nauseous when she arrived at the hospital, where her physician, Doctor Donna, prescribed Tummis to take care of the problem. Tummis can be
taken orally or administered directly into the vein by the IV-push method. Doctor Donna opted for the latter approach even though it is more dangerous than giving
Tummis orally. Doctor Donna mistakenly put the IV-push into an artery instead of a vein. Queezi contracted severe blood poisoning and lost her leg. Queezi sued in state
court on a state tort law claim that Tummis should have been labeled to warn doctors that the IV-push method is dangerous. Tummis' label was approved by the Food and
Drug Administration (FDA), which is authorized to “make drug labeling decisions that strike a balance between competing objectives.” Is Queezi's lawsuit preempted?
o No. On similar facts, the Court ruled against preemption in Wyeth v. Levine, 129 S. Ct. 1187 (2009). First, the Court concluded there was no express preemption
as in the Riegel case on the prior card. The Court then ruled that there is a presumption against implied preemption in order to protect the states' police
powers. The presumption held in Wyeth because Congress knows how to write an express preemption statute and failed to do so in the labeling context. E
Ch.6-III(D).

o NOTE:
o It is easier to win on express preemption claims because of the presumption against implied preemption claims. E Ch.6-III(D).

Under federal law, an alien who is in this country more than 30 days must apply for federal registration, must be fingerprinted, and must carry proof of federal registration.
Failure to comply with any of these provisions is a misdemeanor punishable by a fine, imprisonment, or a sentence of probation. The federal government has discretion to
refrain from prosecution or enforcement in individual cases where prosecution might frustrate other federal policies, undermine relationships with foreign countries, or
be inequitable to the individuals involved. Federal law also imposes numerous strict requirements on aliens regarding their health, education, integrity, character, and
length of residence here.
In 2010, undocumented aliens composed roughly 6 percent of Arizona’s population. The state responded by enacting S.B. 1070, §3 of which makes failure to comply with
the federal government’s alien registration requirements a state criminal misdemeanor, punishable by fine or imprisonment; probation is not a sentencing option. The
avowed purpose of the Arizona law is to discourage and deter undocumented aliens from living in the state. No provision of federal law expressly preempts such state
alien registration statutes. Is the Arizona law preempted by federal law?

o
As the facts indicate, the federal alien registration scheme is pervasive, providing detailed regulations applicable to resident aliens throughout the country.
This pervasiveness supports an inference that Congress intended to occupy the field of alien registration. The federal interest in immigration and
naturalization under Article I, §8, cl. 4 is paramount within our federal system, further suggesting the preemptive breadth of the federal law. Similarly, the
treaty power, which is at least collaterally implicated by these facts, is exclusively within the domain of the federal government. Conversely, the regulation
and registration of aliens has not been traditionally left to the states. Taking these considerations together, it would seem fair to infer that Congress
intended to occupy this field. As such, §3 of the Arizona law would be preempted on the basis of implied field preemption. In Arizona v. United States, 132 S.
Ct. 2492, 2502 (2012), the Court on these facts held that “the Federal Government has occupied the field of alien registration. . . . Where Congress occupies
an entire field, as it has in the field of alien registration, even complementary state regulation is impermissible. Field preemption reflects a congressional
decision to foreclose any state regulation in the area, even if it is parallel to federal standards.”

The Federal Cigarette Labeling and Advertising Act (FCLA) requires cigarette manufacturers to place a warning label on all packages of cigarettes sold in this country.
FCLA also includes a preemption provision. Subsection (a) of that provision provides that “[n]o statement relating to smoking and health, other than the statement
required by [this statute], shall be required on any cigarette package.” In addition, subsection (b) provides that “[n]o requirement or prohibition based on smoking and
health shall be imposed under State law with respect to the advertising or promotion of any cigarettes the packages of which are labeled in conformity with the
provisions of this chapter.” The state of Massachusetts has adopted a law that prohibits the outdoor display of cigarette advertisements, regardless of content, within
1,000 feet of a school or playground. The purpose of the ban is to prevent children from being enticed by such ads. Is the Massachusetts law preempted by FCLA?
o
The answer depends on how one interprets the preemptive language employed by FCLA. Clearly, the state law does not transgress the ban imposed by
subsection (a), because it does not pertain to the content of the information that must be placed on a cigarette package. The outdoor advertising ban does,
however, impose a “prohibition” on the “advertising” of cigarettes sold in conformity with FCLA. Moreover, the ban appears to be motivated by concerns
pertaining to “smoking and health,” namely, the health implications derived from children’s developing an interest in smoking. As such, the state advertising
ban would seem to fall squarely within the text of subsection (b). See Lorillard Tobacco Co. v. Reilly, 533 U.S. 525 (2001) (so holding on similar facts).
o One could argue, however, that subsection (b) was meant only as a supplement to the preemptive reach of subsection (a) by insuring a unitary, nationwide
“warning” requirement, regardless of where that warning appeared—i.e., whether on cigarette packages or on billboards. In this sense, the preemptive reach
of subsection (b) would be limited to those state laws that required cigarette advertisers to include a particular type of content within their advertisements,
namely, one that conveyed a health message endorsed by the state. It would not, however, preempt a state from banning cigarette advertising in particular
locations. This was the view adopted by the dissent in Lorillard Tobacco Co., supra, 533 U.S. at 590 (Stevens, J., dissenting). The basic thesis of the dissent was
that subsections (a) and (b), when read together, were designed solely to protect cigarette companies from a multiplicity of warning requirements that might
vary from state to state.

Dedra Shanklin’s husband was killed when a train operated by Norfolk Southern collided with his truck at a railroad crossing. At the time of the accident, the crossing
was equipped with advance warning X-shaped signs that read, “RAILROAD CROSSING.” The signs had been installed pursuant to a federal program under which the
federal government provided funds to the state for their purchase and installation. To receive these funds, the state was required to demonstrate that the warning
devices to be installed at each railway crossing within the state met federal railway-crossing warning standards promulgated by the secretary of transportation. Ms.
Shanklin brought a wrongful death action against Norfolk Southern claiming that it had negligently failed to maintain adequate warning devices at the crossing. The
railroad contends that Shanklin’s suit is preempted by the Federal Railroad Safety Act, which provides, “Laws, regulations, and orders related to railroad safety shall be
nationally uniform to the extent practicable. A State may adopt or continue in force a law, regulation, or order related to railroad safety until the Secretary of
Transportation prescribes a regulation or issues an order covering the subject matter of the State requirement.” Is the railroad correct?
o
Yes, this would appear to be a case of express field preemption. Although the Federal Railroad Safety Act grants states the option of adopting railroad safety
regulations in the absence of controlling federal law, that option expires once the secretary of transportation prescribes regulations covering the subject
matter. Here, the secretary has promulgated regulations setting standards for warning devices at railroad crossings, at least when those warning devices
are installed pursuant to a federally funded program, as was the case here. Since plaintiff’s negligence claim regarding the adequacy of the warning devices
arises out of the same subject matter as is addressed by the federal regulations, her claim is preempted. This is essentially the reasoning the Court adopted
under similar facts in Norfolk Southern Railway Co. v. Shanklin, 529 U.S. 344 (2000). Since we are dealing with express field preemption, it does not matter
that the state’s tort law may actually advance the federal interest in safety at highway crossings. Compare Sprietsma v. Mercury Marine, 537 U.S. 51 (2002)
(construing a federal statute that expressly preempts a state “law or regulation” as not preempting state common law actions).

A valid federal law specifically allows federally insured lenders to include a “due-on-sale” clause in their loan instruments. Such clauses empower a lender, at its option,
to declare the entire amount of a loan immediately due if the property securing the loan is transferred without the prior consent of the lender. The purpose of the federal
law is to grant lenders broad flexibility in determining when to call a loan. This flexibility is seen as a means of insuring the overall financial soundness of the “thrift”
industry. State law, however, provides that a lender may exercise a due-on-sale clause only if the transfer of property actually impairs the security for the loan. Is the
state requirement preempted because it imposes an obstacle to the accomplishment of the federal objective?
Answer
The answer depends on what one identifies as the federal objective. If the objective is to vest federally insured lenders with unfettered flexibility in determining when to
exercise their due-on-sale options, then the state law does create an impediment to achieving that goal. Under state law, the complete discretion created by the federal law
becomes a more limited discretion, requiring a showing that the lender’s security interest has been impaired. In other words, state law limits the federally created
flexibility. On the other hand, if one views the federal objective as insuring the financial stability of the lender, one could argue that the state law is consistent with that
objective since it permits a lender to enforce a due-on-sale clause whenever the security for the loan has been impaired.
In Fidelity Federal Savings & Loan Assn. v. De La Cuesta, supra, the Court concluded, under similar facts, that the state law operated as “an obstacle to the accomplishment
and execution of the full purposes and objectives” of the federal law. 458 U.S. at 156. The flexibility granted the lender by federal law was seen as essential to the ultimate
goal of promoting the financial soundness of the thrift industry. Nor was the Court moved by the fact that “real property law is a matter of special concern to the States . .
. .” Id. at 153. Although it may make a difference in a close case that the subject under regulation has been a traditional concern of the states, when a state law’s conflict
with a federal statute’s goals is as pronounced as the Court viewed it here, “[t]he relative importance to the State of its own law is not material. . . . ” Id. See also Franklin
National Bank v. New York, 347 U.S. 373 (1954) (federal law that grants national banks the right to provide savings accounts preempts state law that prohibits such banks
from using the word “savings” in their advertisements).

Directly injecting the drug Phenergan into a patient’s vein (the IV-push method) creates a significant risk of catastrophic consequences. A Vermont jury found that as a
matter of state tort law, Pharmo, the manufacturer of the drug, had failed to provide an adequate warning of that risk and awarded damages to Diana whose arm had to
be amputated as a result of an IV-push administration of the drug. The warnings on Phenergan’s label, which did not mention this danger, had been deemed sufficient by
the federal Food and Drug Administration (FDA) when it approved Pharmo’s new drug application in 1955 and when the FDA later approved changes in the drug’s
labeling. Under FDA regulations, a drug manufacturer may not unilaterally change an approved label. There is, however, an exception to that general rule that permits a
unilateral change if the change adds or strengthens a warning to improve drug safety. Pharmo claims that it could not simultaneously comply with FDA regulations,
which generally prohibit changes to an approved label, and the warning standards imposed by state tort law, which would require the label to have been changed. Is
Pharmo correct that it is impossible to comply with both federal and state law and that the former therefore preempts the latter in this case?

No. There is no “impossibility” of simultaneous compliance. This problem would have presented an example of conflict preemption had the FDA regulations not included
the “adds or strengthens” exception, for then Pharmo would find it impossible to comply with the FDA’s “no change” rule and at the same time adhere to the
requirements of state law. The exception recognized by the FDA, however, eliminates the impossibility, since a warning regarding the IV-push dangers of Phenergan
would seem to fall well within adds-or-strengthens standard of the exception. See Wyeth v. Levine, supra, 555 U.S. at 569-573 (so holding under roughly similar facts);
cf. Mutual Pharmaceutical Co., Inc. v. Bartlett, supra (finding impossibility preemption where state law imposed a duty to alter a drug’s warning label and federal law
prohibited any change in the warning label).

Supremacy Clause
Understand the Supremacy Clause and Federal Immunity from State Regulation.

A state law directly contravenes a statute enacted by Congress. What clause in the Constitution would provide the answer for which of the two will remain valid?
o The Supremacy Clause of Article VI. That clause states that: “This Constitution, and the Laws of the United States . . . shall be the supreme Law of the Land.” In
such cases, the state law is preempted by the federal law.

o NOTE:
o Most problems with the Supremacy Clause come about when there's no direct conflict between state and federal law; and the two merely address the same
subject matter in different (but not explicitly inconsistent) ways. A direct conflict exists where one can't comply with both federal and state regulations, or
the objectives of the two conflict.

State of Warden is concerned that there are too many frivolous lawsuits by prisoners against state prison officials that are clogging the courts. In response to this problem,
Warden sets up a special state Court of Claims that will hear civil rights cases against prison officials only. The Court of Claims is a court of limited jurisdiction where
prisoners will not be allowed punitive damages or attorneys' fees. Cases against prison officials brought under the federal civil rights statutes will be heard exclusively in
the Court of Claims. Is the State Court of Claims constitutional?

No. In a 5-4 vote in Haywood v. Drown, 129 S. Ct. 2108 (2009), Justice Stevens wrote that a similar court system violates the Supremacy Clause, Article VI, cl. 2, because the
state's policy is “contrary to Congress' judgment that all persons who violate federal rights while acting under color of state law shall be held liable for damages . . . a State
may not . . . relieve congestion in its courts by declaring a whole category of federal claims to be frivolous.”

What does the Supremacy Clause do?


It provides that any state or local law conflicting with a valid federal law must be struck down.

NOTE:
The Supremacy Clause does not provide a source of power for the federal government!

Does the Supremacy Clause state that the Supreme Court is supreme over the other branches in interpreting the Constitution?
No. The Supremacy Clause states that the Constitution and Laws of the United States are supreme over state laws.
NOTE:
In Cooper v. Aaron, 358 U.S. 1 (1958), the Court stated “the federal judiciary is supreme in the exposition of the law of the Constitution.” E Ch.2-II(C)(1).

What's the most common application of the Supremacy Clause?


Commerce Clause problems. In fact, the only time you use a commerce-clause analysis to decide the constitutionality of a state statute impacting interstate commerce is
when there's no relevant federal legislation. If there is relevant federal legislation, then you analyze the problem under the Supremacy Clause. You ask three questions:
1. Did Congress expressly authorize or prohibit state regulation (or does the Constitution expressly bar state action—e.g., coining money, conducting foreign
affairs)?
If so, that authorization or prohibition controls. If not, go on to #2.
2. Is there a direct conflict between the federal and state regulation (e.g., joint compliance isn't possible, or the objectives conflict)? If so, federal law automatically
preempts. If there's no direct conflict (i.e., the federal and state statutes merely cover the same subject matter), go on to #3.
3. Was the federal law intended to occupy the entire field?
If so, the federal law preempts the state law. If not, the state law stands.

NOTE:
Supreme Court cases indicate that Congress will be deemed to have preempted an area only where its intent is unmistakable, or where the nature of the regulated subject
matter does not permit any other conclusion. Florida Lime & Avocado Growers v. Paul, 373 U.S. 132 (1963); E Ch.6-III(C).

The State of Sillycon enacts a statute regulating the marketing of high-tech instrumentation. Subsequently, Congress enacts a similar regulation wherein Congress provides
that it intends the regulation to cover the whole field of high-tech instrumentation. Will the non-conflicting state law still be valid?
No. If Congress provides for regulation intended to cover the whole area in question, and the area is one in which both Congress and the states have power to regulate
(e.g., commerce, with federal power coming from the Commerce Clause and state power from police powers), then the federal law supersedes the state law, regardless of
whether the state law directly conflicts with the federal law. E Ch.6-III(C)(1); CH p. 393.

The State of Serenity enacts a statute that requires potato farmers to remove federal labels on potato bags and replace them with state-approved labels, which involves an
additional expense to the farmer. One out-of-state grower, Farmer Johannes, challenges this as an invalid state regulation of interstate commerce. What result?
According to the Supremacy Clause, federal law will control, because the state law here directly conflicts with the federal law. McDermott v. Wisconsin, 228 U.S. 115 (1913).
There is a direct conflict here, because one can't comply with both the federal and state regulations. Actual conflict cases are the clearest application of the Supremacy
Clause.

RELATED ISSUE:
Say the state regulation required potato farmers to add a state label without replacing the federal label. Then there would be no direct conflict with federal law, because a
farmer could comply with both federal and state requirements. Instead, the conflict would be indirect, and the validity of the state requirement would depend on whether
Congress intended to preempt the entire potato-labeling field (and, beyond that, whether it unduly burdened interstate commerce, which it almost certainly wouldn't).

NOTE:
The area would be subject to federal regulation under the Commerce Clause, and subject to state regulation by virtue of state police powers. E Ch.6-III(B)(1); CH p. 409.

State X law requires that all state X candidates for the United States House of Representatives must reside in the congressional district they seek to represent in Congress
at the time they are elected. Is this law constitutional?
Probably not. According to the Court’s decision in U.S. Term Limits, Inc. v. Thornton, supra, Art. I, §2, cl. 2 of the Constitution establishes the exclusive qualifications for
membership in the House of Representatives: “No Person shall be a Representative who shall not have attained to the Age of twenty five Years, and been seven Years a
Citizen of the United States, and who shall not, when elected, be an Inhabitant of that State in which he shall be chosen.” Since Art. I, §2, cl. 2 does not mention a district
residency requirement, a state may not impose one. See Texas Democratic Party v. Benkiser, 459 F.3d 582 (5th Cir. 2006) (under Qualifications Clause, state may not
require candidates for House of Representatives to be residents of state prior to date of the election); Campbell v. Davidson, 233 F.3d 1229 (10th Cir. 2000), cert. denied,
532 U.S. 973 (2001) (state may not require candidates for House of Representatives to reside in the particular district in which they seek election); Schaefer v. Townsend,
215 F.3d 1031 (9th Cir. 2000), cert. denied, 532 U.S. 904 (2001) (under Qualifications Clause state may not require candidates for House of Representatives to be a
resident of the state at the time of filing nomination papers as distinguished from the date of election).

Employees of the United States Forest Service (USFS) are required to live in houses owned by the USFS and located within national forests. The housing is considered a
part of the employees’ compensation and the government deducts a specified amount from the each employee’s salary as “rent” for the housing. The county of Fresno,
consistent with the principles of federal tax immunity, does not impose its general property tax on national forest land within the county since the legal incidence of such
a tax would fall on the United States itself as the property owner. However, the county does impose an annual use tax on possessory interests in tax-exempt land,
including the possessory interests of the USFS employees. Does the use tax as applied to USFS employees violate the principle of federal immunity from state taxation?

No. While a state or a subdivision of a state may not impose a tax on the property of the federal government, it may impose a use tax on private persons who are granted
the possession or use of that property. This is so because unlike a property tax imposed on federally owned property, the legal incidence of the use tax falls on the person
using the property rather than on the federal government. In the immediate case, the legal incidence of the use tax falls on the USFS employees. The principle of federal
immunity, therefore, is not violated.
Nor is this tax discriminatory even though it only applies to renters of tax-exempt property. Since the county does impose its general property tax on nonexempt property,
the practical effect of the use tax is to place all renters of property on an equal economic footing, for owners of nonexempt property can be expected to pass the economic
burden of the tax on to their lessees. See United States v. County of Fresno, 429 U.S. 452, 464-468 (1977) (same conclusion under similar facts).

The Zandra Company, a privately owned enterprise, has a contract with the federal government under which Zandra manages federally owned atomic laboratories in
New Mexico. As part of its management duties, Zandra purchases goods from vendors for use in the laboratories. The purchases are made in Zandra’s name, without
prior approval by the federal government. Nor is the vendor notified that the purchases are for the federal government or that Zandra is a purchasing agent for the
federal government; indeed, the federal government denies that Zandra is its purchasing agent. Although Zandra is liable to the vendor for the purchase price of the
goods, title to the goods passes directly to the federal government. The latter reimburses Zandra for the cost of the goods, including any sales taxes. The state of New
Mexico imposes a sales tax on all goods sold within the state. The legal incidence of the tax falls on the purchaser. May this tax be imposed on sales to Zandra of goods to
be used at the federal laboratory?
Yes. The legal incidence of this nondiscriminatory tax does not fall on the federal government, but rather on Zandra as the purchaser. The fact that the economic burden
of the taxes will be passed on to the United States is of no relevance. Since the legal incidence of the tax does not technically fall on the United States, the tax will be subject
to federal immunity only if it can be said that Zandra and the federal government are so closely tied that they cannot realistically be viewed as separate entities. Given the
independence under which Zandra operates, a court is likely to conclude that Zandra and the federal government are separate entities. That being the case, the state may
impose this sales tax on the purchases made by Zandra. See United States v. New Mexico, 455 U.S. 720, 741-743 (1982) (same conclusion under similar facts).
Suppose, however, that Zandra correctly described itself as a federal procurement agent when it made its purchases; that title to the purchased goods passed di rectly to
the federal government; that the purchase orders declared the purchase to be made by the federal government; and that Zandra was in no manner liable for the purchases.
These changed facts might well lead a court to conclude that the purchaser of the goods was the United States, rather than Zandra. Under such circumstances, the legal
incidence of the tax would fall on the federal government and the tax would, therefore, be invalid. See Kern-Limerick Inc. v. Scurlock, 347 U.S. 110 (1954) (same conclusion
under similar facts).
Driving without a valid driver’s license is a misdemeanor in the state of Montana. Lester, a resident of Montana, delivers mail on a rural route for the United States Postal
Service, an instrumentality of the federal government. He does not have a valid driver’s license. If Lester continues to drive his mail truck as part of his postal service
duties, may he be punished by the state for driving without a license?
No. The state may not require Lester to obtain a driver’s license as a prerequisite to driving for the United States Postal Service. To conclude otherwise would be to
accept the authority of a state to regulate the employment practices of a federal instrumentality. In precisely such a case, the Court explained, “the immunity of the
instruments of the United States from state control in the performance of their duties extends to a requirement that they desist from performance until they satisfy a
state officer upon examination that they are competent for a necessary part of them. . . . ” Johnson v. Maryland, 254 U.S. 51, 57 (1920) (state may not convict postal
worker for driving without a license in the course of his employment); see also Leslie Miller, Inc. v. Arkansas, 352 U.S. 187 (1956) (contractor working for federal
government immune from state licensing requirement).

DORMANT COMMERCE CLAUSE

Alabama is home to an environmentally friendly manufacturer of paper products. To encourage environmentally responsible packaging and to promote the consumption
of alternatives to environmentally harmful materials, Alabama prohibits the importation of all out-of-state paper products. Assume that Alabama’s stated purpose, to
preserve the environment, is sincere. Alabama’s regulation is likely

Unconstitutional, because Alabama could limit the use of all paper products in the state.

These are a close approximation of the facts in Philadelphia v. New Jersey. Recall that the DCCD applies when Congress has not acted; it is an implicit restriction on state
power that does not depend on the existence of affirmative congressional action. Therefore Dis incorrect.
A and C are likewise incorrect, because recall that the DCCD can be violated by ends as well as means. Even if Alabama has a legitimate interest in protecting the
environment (which it does), and that motive, not some illegitimate interest in economic protectionism, was behind the law’s passage, it still treats out-of-state paper
products differently than those produced in-state based on the origins of those products. The law is facially discriminatory and, because, as B suggests, Alabama has
nondiscriminatory alternatives available to it, the law is likely to be invalidated by a court. Thus, B is the correct answer.

State biologists in Maine have learned that parasites have been diminishing fish stocks in other states. To preserve its native fish population, which has so far avoided
contamination, the legislature passes a law banning the importation of baitfish into the state from other states. Biologists testify that the ban is the only way to ensure
that the parasites won’t infect native fish stocks, because there is no efficient way to test incoming baitfish for the parasite. Out-of-state baitfish suppliers sue, claiming
the Maine law violates the DCCD. A reviewing court should

d. Uphold the law, because there is no other effective means for the state to protect its wildlife.

o Rationale for Correct Answer


o There are no facts given suggesting that Maine’s legislative purpose is in question here. Therefore B is incorrect. Moreover, C is true, insofar as Maine is
exercising its police power here, but that is irrelevant for DCCD purposes. A pure heart does not insulate government regulations from review under the
DCCD. As in Question 1, the ends chosen to effectuate that end might be invalid. Based on Question 1, you might have jumped at A, but facts are given that
make D the correct answer.
o Remember that although facially discriminatory laws are presumptively invalid, they may survive scrutiny if there is a legitimate interest and no less
discriminatory means available to the state to effect that interest. According to the facts, Maine wanted to protect its fish from these harmful parasites, clearly
a legitimate state interest; moreover, the facts indicate that there is no efficient way to test for the presence of parasites in any incoming fish, so the state
claims that it must bar their importation completely. If that is true, then such a law would likely overcome the presumptive unconstitutionality of facially
discriminatory measures.

Concerned about the health risks associated with smoking, and the attendant costs to the state providing care to smokers, Connecticut has taken the extraordinary step of
prohibiting the sale of all tobacco products in the state. Connecticut has a tiny cigar industry, but otherwise produces no tobacco products. Out-of-state tobacco companies
sue, claiming the law violates the DCCD. They note that, as a result of the law, they potentially stand to lose ‘‘millions’’ of dollars in revenue each year from tobacco sales.
The state argues that it spends $1.2 million per year on ‘‘tobacco-caused’’ health care. Other costs include losses in productivity and absenteeism caused by smoking and
smoking-related illnesses, which the state estimates at $1.5 million per year. What should a reviewing court do?

d. Sustain the law, because the burden on interstate commerce does not clearly exceed the benefits to the state.

Rationale for Correct Answer


Unlike the laws in the preceding questions, Connecticut’s law does not differentiate between in-state and out-of-state tobacco. Sales of all tobacco products are prohibited.
Moreover, unlike some questions that follow, it is not the case that, although facially neutral, the effects of the law fall entirely or nearly entirely on out-of-state commerce.
There is some in-state cigar-making. Therefore, C is incorrect.
However, B is not a sufficient condition for upholding a state or local law under the DCCD. Even-handed statutes — even those lacking discriminatory purposes or effects
(discussed later) — can be invalidated if they do not satisfy Pike balancing. The facts do indicate the possibility that out-of-state tobacco companies will lose as much or
more than Connecticut might save by avoiding smoking-related costs (assuming that no one will buy tobacco products out of state and use them in Connecticut), so you
might have found yourself attracted to A, but the inquiry is not whether the burdens exceed the local benefits, it is whether the benefits are clearly exceeded by the burden
on interstate commerce.

The North Carolina Department of Agriculture issued a regulation banning the use of state grades on closed containers of apples sold in the state. The regulation specified
that the only grade that could be used was the grade approved by the United States Department of Agriculture. Apple growers in Washington objected to this regulation,
noting that they had spent a considerable amount of money developing a grading system acknowledged by those in the industry as being superior to the federal grading
system. North Carolina, which also produces apples for sale, has no state grading system. In their lawsuit, the Washington apple growers allege the regulation was
motivated by a discriminatory purpose. Which of the following would not be helpful in demonstrating discriminatory purpose?

A preamble to the legislation reciting that the purpose of the bill was to ensure consumers were not confused by a proliferation of grading systems.
To the extent that purpose makes a difference in constitutional doctrine, the Court usually permits evidence from which one might infer an improper purpose, as it is often
difficult to demonstrate directly. In the DCCD, the improper purpose would be anything that smacked of protectionism or discrimination against out-of-state commerce.
Thus, B would be useful information, because it suggests that the regulation was prompted by concerns of local apple growers that they were having difficulty competing
against their out-of-state competitors.
C would be useful as well. Although a facially neutral regulation whose impact is felt by out-of-state commerce only is often regarded as proof of the discriminatory effects
of the law, the effects can also raise questions about the purpose as well. A similar situation might arise if the law was facially neutral, covering in-state and out-of-state
products alike, but containing numerous exemptions that coincidently shielded local products from the impact of the law. Similarly, short-circuiting the ordinary means
of passing laws might raise suspicions that something untoward was motivating the departure from normal procedures. Therefore, D would be helpful as well.
That leaves A, which is not helpful in establishing a discriminatory purpose, because the purpose stated on the face of the law — consumer protection — is a valid exercise
of the state’s police power and does not suggest unconstitutional economic protectionism or discrimination against interstate comm
Texas requires car dealers to be licensed by the state. Under the state licensing statute, ‘‘automobile manufacturers’’ are not eligible to become licensed car dealers in the
state. Texas has no automobile manufacturers located in the state. Car manufacturers located in Michigan have begun to experiment with selling certified used cars over
the Internet. Under this sales model, leased automobiles that have been turned back in, fleet cars, and other quality, low-mileage cars are put through a series of factory-
approved checks and then resold as used, but with extended warranties. Customers could shop online, then have a car they selected delivered to a local dealer in their
area. The dealer would be paid a fee by the manufacturer for service and handling, but the customer would otherwise not have to visit the local dealership until the car
was delivered, because financing, too, would be handled online. Texas law, though, prohibits such sales from being made to Texas residents because the manufacturers
do not — and cannot — possess a car dealer’s license. Car manufacturers sue, alleging that the law has discriminatory effects. Which of the following would be the
strongest argument in support of their claim?

c. That the effect of the law would be to increase market share in used cars for in-state used car dealers.

Exxon seems to hold that discriminatory effects are not present merely because a state law’s regulatory burden falls on out-of-state firms. Therefore, B is not going to be
enough to sustain a discriminatory effects challenge. Moreover, although the fact that the state has no automobile manufacturers of its own might make it seem as if the
intent is to benefit in-state businesses (Texas used car dealers) at the expense of the out-of-state manufacturers, Exxon rejected an analogous argument that Maryland’s
law benefited in-state independent retail gas station operators at the expense of out-of-state petroleum producers and refiners. So A is not going to be helpful to their
claim, either.
Because the DCCD is sometimes described in terms of creating or preserving a ‘‘free trade’’ zone free among the states, you might have been tempted to answer D. But
whether a state’s regulations result in higher prices or reduced competition among its own citizens is of no consequence to the DCCD, unless those come about as a result
of discrimination against interstate commerce. That leaves C, which is the best answer. If the automobile manufacturers could demonstrate that, as a result of their being
excluded from selling cars in the state, the market share for cars claimed by in-state dealers would increase, that would be helpful in establishing their claim because it
appears that the in-state competitors are benefiting to the detriment of the out-of-state manufacturers as a result of the law. On the other hand, the same might have been
said about the independent retail gas stations in Exxon.

Alabama passed a statute mandating that all printing contracts paid for by state money be bid competitively. A year later, the act is amended; the amendment creates an
exception to the prior year’s act. If an in-state printing company’s bid is within 20 percent of the lowest bidder’s bid, and the low bidder is an out-of-state company, the
Alabama printer is automatically awarded the contract, despite not being the low bidder. When the University of Alabama’s yearbook printing contract comes up for a
bid, LaGrange Printing Co., a Georgia company, is the lowest bidder. However, Phenix City Printing, an Alabama company, enters a bid that is only 18 percent higher than
LaGrange Printing Co., thus winning the contract. LaGrange Printing sues, alleging that the Alabama law violates the DCCD. What should a reviewing trial court do?
. Uphold the law, because Alabama is not acting as a market regulator.

First, if you spotted that the amendment to the Alabama law was facially discriminatory, you’re getting the hang of the DCCD. Therefore, you might have been tempted to
apply strict or heightened scrutiny, and choose B. However, remember that the MPE is an exception to the DCCD. Therefore, you have to consider carefully the other
choices.
If you read closely, you would not choose D, because South-Central Timber stands for the proposition that certain conditions on government contracts are not covered by
the MPE. ‘‘Downstream’’ restrictions that attempt to exert control outside the market in which the state is participating, the Court has held, are not covered by the MPE.
Here, however, Alabama has not attempted to impose any restrictions beyond the market — printing services — in which it has chosen to participate. The state is only
requiring that any state money used to buy services in that market go to in-state printing companies when it is not dramatically more expensive to do so. The statute is
discriminatory to be sure, but it does not attempt to impose any restrictions beyond that particular market. Therefore C is not correct. This leaves A, which is the best
answer, because Alabama is simply exercising its discretion in its choice of vendors, and is willing to pay a little more to support local businesses, as any consumer would
be entitled to do.

Delighted with the success of the Oneida-Herkimer Solid Waste Authority, Herkimer County decides to create a number of county-owned hamburger stands called
‘‘Herki-burger Hamburger Heaven.’’ To ensure their success, the county then requires that all hamburgers be purchased at these county-owned stands. No other
hamburger restaurants are permitted in Herkimer County. An out-of-state owner of a competing private hamburger franchise wishes to challenge the ordinance. Which
of the following would be the strongest argument against the constitutionality of the hamburger stand?
Herkimer County is not engaged in a traditional governmental function.
As strange as these facts might seem, they come from a question put by one of the Justices at oral argument! Here we have the same facts as United Haulers, but with
hamburgers instead of garbage collection and disposal. Although A is true, it is not helpful here, because the point is that some exceptions — the market-participant
exception and United Haulers’ ‘‘public entities’’ exception — permit state and local governments to enact discriminatory laws.
The exception created by United Haulers is separate from, and different than the market-participant exception, so C is not correct. (In any event, Herkimer is doing more
than simply participating in the market for hamburgers; it is monopolizing it. The use of governmental power to exclude competitors is not the sort of power ordinarily
available to private market participants.) Further, although D is true, that actually functions here as an argument in favor of the ordinance’s constitutionality, according
to United Haulers, which pointed out that the flow control ordinance in that case made no distinction between in-state and out-of-state private waste haulers. Here, too, no
distinction is made between in-state and out-of-state private hamburger stands.
That leaves B. It’s not clear how much of United Haulers hinges on the Court’s characterization of garbage hauling as a traditional governmental function. And as we saw
in Chapter 4’s discussion of Usery, the Court has in the past abandoned attempts to define such activities. But one might argue that establishing monopolies to supply things
like hamburgers (as opposed to supplying electricity or even garbage collection) is not ‘‘traditional’’ and should therefore be limited. In any event, it is the strongest
argument among the choices.
ANALYZING STATE STATUTES AFFECTING INTERSTATE COMMERCE
1. Is there relevant federal legislation? If so, there's a preemption issue, not a Commerce Clause issue. If not, go on to #2.
2. Does the statute discriminate against interstate commerce—that is, does it either intentionally or unintentionally favor local economic interests over the
economic interests of other states?
If it does, the statute is invalid without Congress' consent to so discriminate, regardless of whether the burden would otherwise be permissible, unless either
(a) it is designed to protect health and safety interests, it is reasonable, and no non-discriminatory alternatives are available; or (b) the state is a “market
participant,” in which case it can discriminate in favor of local businesses as long as it doesn't violate the interstate Privileges & Immunities Clause of Article
IV (watch for this any time a state or city interferes with private sector employment).
3. If the statute is non-discriminatory in both intent and effect, a balancing test is used to determine whether the burden on interstate commerce is unreasonable,
thus invalidating the regulation: the burden on interstate commerce (e.g., cost and difficulty of compliance, inefficiency created, existence of less burdensome
alternatives) versus the strength of the state interest in the regulation.
E Ch.6-I(G).

N.B.:
There is a strong presumption of validity of non-discriminatory statutes where the state interest is health, safety, or social welfare; on the other hand, a court is not likely
to uphold the statute where the state interest is economic (i.e., aiding local business interests).

NOTE:
Non-discriminatory regulation enjoys a presumption of constitutionality that can only be overcome by a clear showing that the state benefit is outweighed by a national
interest in uniformity or the free flow of commerce.

NOTE:
Congress can't authorize state violations of constitutional provisions other than the Commerce Clause (e.g., Due Process and Equal Protection).
Are there any circumstances under which states can regulate interstate commerce?
Yes. Non-discriminatory regulation is allowed as long as it meets this balancing test—the burden on interstate commerce (e.g., cost and difficulty of compliance, inefficiency
created) is weighed against the strength of the state interest in the regulation to determine whether the burden on interstate commerce is “clearly excessive.” Pike v. Bruce
Church, Inc., 397 U.S. 137 (1970). In general, there is a strong presumption of constitutionality of non-discriminatory statutes where the state interest is safety, health, or
social welfare; courts are much less likely to uphold statutes that only protect local economic interests. E Ch.6-I(G)(3); CH §5.3.5.

NOTE:
Congress does, in practice, enjoy exclusive control over foreign commerce—and foreign commerce is interpreted broadly (e.g., a shipment from San Francisco to San Diego
could be considered “foreign commerce” if the shipment was on the “high seas”). Lord v. Steamship Co., 102 U.S. 541 (1880); E Ch.6-I(E); NR pp. 159-160.

What is the “market participant” rule relating to interstate commerce?


When the state is a market participant (i.e., buyer or seller of goods), it may discriminate in favor of local businesses—that is, it could buy from local businesses exclusively
(or preferentially); similarly, it could sell to them exclusively/preferentially.

RATIONALE:
The Commerce Clause only prohibits discriminatory regulation; when a state itself buys or sells, it's a participant, not a regulator.

EXAMPLES:
A state could require that all cement produced by a state-owned factory be sold only to state residents; it could sell state-owned resources to state residents at a lower
price than to out-of-staters; it could offer a company a cash bonus or tax exemption for opening a factory in state. It could not, however, require that a privately
owned factory sell its goods only to people in state; it couldn't pass laws requiring a company to give employment preferences to state residents.

N.B.:
Keep in mind that the fact a state is a market participant doesn't insulate it entirely from violating the Commerce Clause. Situations in which the state bears the cost of
providing economic benefits are permissible, because the state is spending its own tax revenues, as it has a right to do. However, where what the state is doing is really
shifting the cost of the local benefit to out-of-staters (e.g., by requiring privately owned local businesses to sell to people in-state at a discount), the action is a kind of
regulation, and it violates the Commerce Clause.

N.B.:
The state as a market participant would still be subject to the interstate Privileges & Immunities Clause, which limits its leeway as a market participant. Watch for this
whenever a state or city interferes with private sector employment (e.g., by requiring a contractor on a state building project to hire only state residents for the project;
this could violate the interstate P & I Clause). E Ch.6-I(M)(2); CH §5.3.7.2.

ANALYZING STATE STATUTES AFFECTING INTERSTATE COMMERCE


State regulations affecting interstate commerce must satisfy the following three requirements:
1. The regulation must pursue a legitimate state end;
2. The regulation must be rationally related to that legitimate state end; and
3. The state's interest in the regulation must outweigh the regulatory burden.

4.
Do states have any power to regulate foreign commerce?

5. Hide Answer

6. No; regulating foreign commerce is exclusively a federal power.

7. RATIONALE:
8. There must be only one voice in regulating commercial relations with other governments. Michelin v. Wages, 423 U.S. 276 (1976).

9. RELATED ISSUE:
10. A state can only tax foreign commerce if these six requirements are met: (1) The tax is non-discriminatory; (2) The activity being taxed has sufficient
“minimum contacts” with the taxing state; (3) The tax is apportioned fairly; (4) The tax is related to services provided by the state; (5) The tax notwithstanding
apportionment does not create a substantial risk of international multiple taxation; and (6) The tax doesn't prevent the federal government from speaking
“with one voice when regulating commercial relations with foreign governments.” Japan Line, Ltd. v. Los Angeles, 441 U.S. 434 (1979); NR pp. 159-160, 240.

Is interstate commerce immune from state taxation?


11. No; although Congress can forbid states from taxing interstate commerce, if there is no contrary federal legislation, the Court will balance the burden on
commerce against the state's need for revenue, and require interstate commerce to pay a fair share. E Ch.6-II(F); CH §5.4.1.

12. EXAMPLE:
13. The State of Fandango imposes a 5% tax on interstate telecommunications originating or terminating in state, which is charged to an in-state service address.
(There is a credit for any taxpayer proving that he paid a tax in another state on the same call triggering the 5% tax.) Such a tax is constitutional. Goldberg v.
Sweet, 488 U.S. 252 (1989).
14.
Can Congress forbid states from taxing interstate commerce?
15. Yes. Congress can, under its commerce power, forbid states from passing taxes that affect interstate commerce. Prudential Ins. Co. v. Benjamin, 328 U.S. 408
(1946).

16. NOTE:
17. If there isn't any federal legislation, the states are free to place a fair level of tax on interstate commerce, because such commerce must be expected to shoulder
its fair share of a state's expenses. E Ch.6-II(F)(1); CH p. 455.

18. NOTE:
19. State taxes that discriminate against out-of-state residents also violate the Article IV, §2, interstate Privileges & Immunities Clause. CH p. 464.

20. NOTE:
21. State taxes that discriminate against interstate commerce violate the Equal Protection Clause if they are not rationally related to a legitimate state purpose
(e.g., the state cannot deny a tax exemption to a corporation solely due to its being incorporated in another state). WHYY v. Borough of Glassboro, 393 U.S. 117
(1968); CH p. 464.

Under what circumstances may a state tax activities involving interstate commerce?
First, the state may tax such commerce if Congress expressly so consents.
When there is no guidance from Congress, the validity of the tax will depend on whether it discriminates against interstate commerce.
1. If it is discriminatory (e.g., it singles out interstate commerce for taxation, and there's no similar tax on local commerce), it virtually automatically violates the
Commerce Clause. (It would also violate the interstate Privileges & Immunities Clause, Art. IV, §2 and perhaps, the Equal Protection Clause if the discrimination
isn't rationally related to a legitimate state purpose.) Ch.6-II(C).
2. If it's non-discriminatory, the court uses a balancing test to determine the validity of the tax on interstate commerce. The court balances the state's need for
revenue versus the burden on the free flow of commerce. The burden generally will be too great under the Commerce Clause, if multiple burdens are imposed
on an interstate activity by various state taxes. It will be invalid under the Due Process Clause if there are “insufficient contacts” between the subject matter
and the state to justify the state's taxing the subject matter (because the subject matter would receive insufficient benefits from the state to justify the tax). E
Ch.6-II(B)-(F); CH pp. 455-458.
A state statute bars the importation of baitfish from other states. The effect of the law is to shield local baitfish suppliers from the competition posed by out-
of-state suppliers. Is the law economic protectionist and therefore invalid per se?
The answer depends on why the state banned the importation of baitfish. If the law was adopted because of the fact that it would protect the state’s own baitfish
sellers from their out-of-state competitors, then it constitutes simple economic protectionism. If, on the other hand, the measure was enacted to safeguard the
state’s fish population from parasites thought to be carried by imported baitfish, the law is not economic protectionist, for it can no longer be said that it was
adopted because of the fact that it will shield local baitfish suppliers from interstate competition. While local baitfish suppliers no doubt welcomed the ban and
perhaps even lobbied for it, the measure was adopted in spite of—rather than because of—its competition-shielding effects.
3. A new state X law provides that no vegetables may be sold in the state unless they are certified by the state agricultural commissioner as having been grown
without the use of certain pesticides. No other state bans the use of these specific pesticides. Growers who do not comply with the law are barred from
selling their produce in state X. Smith grows vegetables in state Y and sells most of his crop in state X. Because of the state X law, Smith must now cease
using these pesticides on his fields in state Y if he wishes to continue participating in the state X market. May Smith successfully argue that the state X law is
per se invalid because of its proven extraterritorial effects?
4. No. With respect to the first requirement, the state X law does have the effect of legally controlling Smith’s crop-raising activity in state Y. If Smith wishes to
sell his produce in state X, he has no choice but to comply with state X’s pesticide rules when growing his crop in state Y. If he fails to comply with these rules,
he will be sanctioned by being barred from the state X market.Though the first requirement of Brown-Forman is met, Smith’s challenge will fail under the
second requirement. In contrast to the liquor laws in Brown-Forman and Healy, state X is not regulating an activity or transaction that is wholly unconnected
with the state. Instead, state X’s pesticide law applies only to those out-of-state crops that will later be sold in state X. Vegetables to be sold in other states
need not be grown in compliance with the state X rules.
A state Z corporate takeover law regulates stock tender offers by entities that seek to acquire control of corporations having their principal place of business
in state Z. The act regulates such tender offers regardless of where the entity making the tender offer is located. However, it is undisputed that, because of
state Z’s small size, most of the tender offers that are subject to the act will be made by out-of-state entities. Does state Z’s law discriminate against interstate
commerce?
No. The act is facially neutral, applying to all tender offers regardless of the location of the entity that launched the offer. While the burden of the law will fall
more heavily on out-of-state rather than in-state companies, this is merely the result of the fact that there are more entities launching such tender offers
outside the state than there are inside the state. In contrast to our earlier apple problem, the burden that the act places on those in-staters and out-of-staters
to whom it applies is, both facially and as a practical matter, exactly the same. “Because nothing in the . . . Act imposes a greater burden on out-of-state offerors
than it does on similarly situated [state Z] offerors, we reject the contention that the Act discriminates against interstate commerce.” CTS Corporation v.
Dynamics Corporation of America, 481 U.S. 69, 88 (1987).For the same reason, a Montana severance tax on coal mined in the state did not discriminate against
interstate commerce where 90 percent of the coal was shipped to consumers in other states. Though out-of-staters bore 90 percent of the tax burden, the tax
did not distinguish between in-state and out-of-state consumers, all of whom were taxed the same amount on the coal they consumed. Commonwealth Edison
Co. v. Montana, 453 U.S. 609, 618-619 (1981).
A state prohibits out-of-state trucks from exceeding 10 tons in weight; no weight limit is imposed on in-state trucks. The limit was adopted for the purpose
of protecting the state’s highways. Because surrounding states have a 12-ton weight limit, the law places a heavy burden on interstate commerce. On what
bases might the law be challenged under the dormant Commerce Clause?
o This law might be challenged on the ground that it discriminates against interstate commerce and that there is a less discriminatory alternative that would
achieve the state’s goal equally well or better: Apply the 10-ton weight limit to all trucks, including those with in-state plates. This alternative equalizes the
burdens placed on intrastate and interstate commerce. Yet, while it is less discriminatory against interstate commerce, it in no way reduces the burdens
placed on that commerce.
o The 10-ton weight limit might also be attacked on the ground that there are less burdensome ways of achieving the state’s goal equally well. If it could be
shown that the 12-ton weight limit used by surrounding states is as effective as a 10-ton limit in protecting highways, the 10-ton limit would violate the
dormant Commerce Clause requirement that states use the least burdensome means available to accomplish their goals.
o In this hypothetical situation, the less burdensome alternative of raising the limit to 12 tons is quite different from the less discriminatory alternative of
extending the 10-ton limit to in-state trucks. Redressing the discrimination problem by expanding the law to cover in-state trucks does not cure the fact that
the law may be more burdensome than necessary on interstate commerce. Nor, conversely, would solving the burden problem by raising the weight limit for
out-of-state trucks to 12 tons in and of itself cure the fact that the statute is discriminatory.

Acme, Inc., manufactures and sells bowling balls in states X, Y, and Z. Acme’s gross annual income is $10 million. Zephyr, Inc., also earns gross annual income of $10
million from the manufacture and sale of bowling balls, but all of its operations are confined to state X. State X levies a 10 percent tax on the gross annual income of any
corporation that does business in the state. Does Acme face the risk of multiple taxation?
Yes. If states Y and Z were to impose a gross receipts tax similar to the one imposed by state X, Acme would be subject to a total tax bill of $3 million. By contrast, Zephyr,
whose gross income is identical to Acme’s, will pay only $1 million in taxes. Because Acme is an interstate business, it is at a competitive disadvantage with intrastate
businesses such as Zephyr’s. The tax by state X violates the principal of internal consistency and will be subject to the requirement of fair-apportionment.

The City of Fairhaven is located in the state of North Texarkansas. The border with the neighboring state of South Texarkansas is three miles south of the Fairhaven city
limits. After a serious outbreak of food poisoning, traced to improperly butchered meat, the Fairhaven City Council passed an ordinance forbidding the sale within
Fairhaven of any meat not killed at a slaughterhouse inspected by the Fairhaven Department of Sanitation. All evidence suggests that the Fairhaven ordinance was in fact
motivated solely by health and safety objectives, not by any desire to favor local producers. Fairhaven Sanitation inspectors survey all slaughterhouses within a 70-mile
radius of the city, but do not attempt to cross the boundary to inspect South Texarkansas slaughterhouses. South Texarkansas, at the state level, conducts its own
inspections of slaughterhouses for sanitation, using standards that are closely similar to those used by the Fairhaven inspectors.
Chopem, the owner of a butcher shop in Fairhaven, sells meat purchased from a slaughterhouse in South Texarkansas. Chopem was charged with selling meat that had not
been slaughtered in a Fairhaven-inspected slaughterhouse. Chopem would like to get the charges dismissed on the grounds that the statute, as applied here, violates the
U.S. Constitution.
(a) What is the strongest argument that Chopem can make for the unconstitutionality of the Fairhaven ordinance? _________________
(b) Will this constitutional attack succeed? State your reasons. _________________
o That the ordinance unreasonably burdens interstate commerce.
o Yes. The dormant Commerce Clause prevents a state or local government from placing undue burdens on interstate commerce. Most violations of the
dormant Commerce Clause occur when government acts in a “protectionist” manner, i.e., with an intent to favor the economic interests of local residents over
out-of-staters. But even a non-protectionist “neutral” regulation will be found to violate the dormant Commerce Clause if it amounts to an unreasonable
burden on commerce. This can be true, for instance, of regulations that are enacted for the good-faith purpose of protecting the safety or health of local
residents. The Court performs a “balancing test,” weighing the state or local government’s interest in its regulatory scheme against the national interest in
unburdened free-flowing interstate commerce. A major part of this balancing is whether there were less burdensome alternatives which the government
might have adopted.
o Here, since South Texarkansas conducts a similar inspection, Fairhaven could simply have accepted the results of South Texarkansas’s inspection without
materially compromising its own health standards. Therefore, the Fairhaven ordinance will probably be found to have been an “unreasonable” burden on
commerce. See Dean Milk Co. v. Madison. (The fact that Fairhaven is a municipality rather than a state, and its ordinance might have discriminated against out-
of-town but in-state slaughterhouses, i.e., those beyond the 70-mile inspection radius, does not save the ordinance from a dormant Commerce Clause attack.)
The state of New Wales has a single nuclear power plant. Because the power plant has been in operation for over 20 years, it is now time for the plutonium
used in the plant to be disposed of. The safest and cheapest way to do this is to bury it in a lead-enclosed structure 200 feet below the surface. Because
residents of the state are worried that their state will become a “toxic dumping ground” if strict measures are not taken, the New Wales state legislature has
enacted the following statute: “No plutonium imported into this state after 1994 may be buried anywhere within the confines of this state.” The effect of this
statute is to permit the state’s existing utility to make a one-time disposal of its pre-1994 plutonium by the burial method described above. The owners of a
nuclear reactor located in South Brunswick, the state directly east of New Wales, have attacked the new statute on the grounds that it violates the Commerce
Clause because it prevents them from shipping their spent plutonium into New Wales and burying it there. Assume that Congress has not spoken on the issue
of nuclear-waste disposal at all. Should the court hearing this action agree with the plaintiff? _________________
Yes. Any state or local action that is taken for a protectionist purpose — that is, for the purpose of preferring in-state economic interests over out-of-state interests —
will be strictly scrutinized . This is true even where the measure is taken for what are basically environmental or other non-economic motives. Thus in City of Philadelphia
v. New Jersey (a garbage disposal case), the Court held that one state could not ban the importation of another’s garbage, unless there were no less-discriminatory
alternatives. Here, New Wales is clearly preferring local nuclear plant operators to out-of-state operators. New Wales clearly has a less-discriminatory alternative (allow
importation, and simply limit the total amount of plutonium that may be buried in the state), so the measure would almost certainly be struck down.

Privileges and Immunities Clause of Article IV

ANALYZING INTERSTATE PRIVILEGES & IMMUNITIES CLAUSE PROBLEMS


1. Is the person being discriminated against an out-of-state citizen or resident? If so, he's covered by the clause, go on to #2. (Corporations and aliens are not
protected by the clause.)
2. Is the activity regulated a right “fundamental to national unity”? (Essentially, this means the right to earn a living or otherwise pursue a commercial activity,
and probably also means rights like receiving medical care, owning property, and accessing the judicial system.) If it is, go on to #3. If not, it's not protected.
3. Is the discrimination closely related to a substantial state purpose (e.g., protecting the state's natural resources)? If it is, go on to #4. If not, it violates the
clause.

4. Is the discrimination a reasonable means of protecting the substantial state purpose—namely, are there no less-restrictive means reasonably available? If so,
the discrimination is permissible. If not, the discrimination violates the clause.

Does the interstate Privileges & Immunities Clause require a state to permit a foreign corporation (incorporated in some other state) to do business in state?
5. No. The interstate Privileges & Immunities Clause only protects out-of-state citizens and residents, and a corporation (or an alien) is not considered a “citizen”
or “resident” for purposes of the clause. E Ch.7-IV(B)(2)(a).

6. NOTE:
7. Once a state allows a foreign corporation to do business in-state, that corporation will be protected by the Equal Protection and Due Process Clauses, under
which a corporation is a “citizen.” Thus, under the Equal Protection Clause, a state couldn't discriminatorily burden the foreign corporation, unless the
discrimination was rationally related to some legitimate state purpose. (Note that promoting local business at the expense of an out-of-state competitor is
not a legitimate state purpose.) E Ch.7-IV(B)(6); NR pp. 385-386.

The State of Elliott enacts a statute preventing aliens from owning more than five acres of land in Elliott. E.T., a resident alien, wants to buy ten acres of land
in Elliott and challenges the statute on grounds that it violates the interstate Privileges & Immunities Clause. What result?

E.T. loses, because he isn't protected by the interstate Privileges & Immunities Clause (although he would have a valid Equal Protection claim). The interstate Privileges &
Immunities Clause only protects out-of-state citizens and residents, not corporations or aliens. Article IV, §2, cl. 1; E Ch.7-IV(B)(2)(a); CH pp. 466-467.

It's duck season and Elmer Fudd wants to go on a duck hunting vacation in the State of Animation, adjacent to the state in which Fudd lives. When Fudd applies for an
Animation duck hunting license, he finds that the license will cost him $400, while an Animation resident could obtain the same license for $20. He challenges the scheme,
contending that it violates the interstate Privileges & Immunities Clause. What result?
Fudd will lose. The interstate Privileges & Immunities Clause, Article IV, §2, cl. 1, only prevents discrimination against out-of-state citizens and residents as regards rights
fundamental to national unity. Recreational duck hunting is not such a right. Baldwin v. Montana Fish & Game Comm 'n, 436 U.S. 371 (1978); E Ch.7-IV(B)(3)(a)(ii); CH p.
473.

RELATED ISSUE:
Say Fudd's livelihood consisted of the processing of duck meat into duck loaf, duck dogs, duck jerky and the like, and that Fudd hunted ducks to get his meat supply. Then
his duck hunting would probably be such a “fundamental right” (earning your livelihood from your chosen pursuit normally is), in which case it would probably be
protected by the interstate Privileges & Immunities Clause. E Ch.7-IV(B)(3)(a); CH pp. 471-473.

The State of Kangaroo restricts the right to practice law in the State to Kangaroo residents. Les Judicata, a non-resident, has passed the Kangaroo bar. He challenges the
restriction as a violation of the interstate Privileges & Immunities Clause. The State claims the restriction is valid because lawyers are officers of the court who exercise
state power, and making Kangaroo accept non-resident lawyers would threaten its sovereignty. In addition, the State argues, non-resident lawyers would be less competent
due to their lesser familiarity with local rules and customs. What result?
Les will prevail. The interstate Privileges & Immunities Clause, Article IV, §2, cl. 1, generally prevents states from discriminating against out-of-state citizens and residents
with regard to rights fundamental to national unity. Pursuing one's livelihood is considered such a fundamental right. Lawyers aren't considered to exercise actual
governmental power, so the sovereignty argument doesn't help the State; as to lawyer competence, this is a substantial state concern, but it could be accomplished by less
restrictive means (e.g., a licensing exam). As a result, the restriction is invalid. Supreme Court of New Hampshire v. Piper, 470 U.S. 274 (1985); E Ch.7-IV(B)(4); CH p. 471.

RELATED ISSUE:
Les would also have an Equal Protection claim, because the restriction classifies people into residents and non-residents, and it restricts a fundamental right of non-
residents (the right to interstate travel). As a result, the classification would be subject to strict Equal Protection scrutiny, and would almost certainly be overturned.

The State of Medusa requires that all people who are hairdressers must be licensed to practice their craft in the state. To get a license, one must graduate from an in-state
hairdressing academy. Does this requirement violate the Privileges & Immunities Clause?
Probably. The interstate Privileges & Immunities Clause, Article IV, §2, cl. 1, generally prevents states from discriminating against out-of-state citizens and residents with
regard to rights fundamental to national unity. Pursuing one's livelihood is considered such a fundamental right. A hairdresser's license should qualify as affecting a right
to pursue one's livelihood. Supreme Court of New Hampshire v. Piper, 470 U.S. 274 (1985); E Ch.7-IV(B)(4); CH p. 471.

NOTE:
A license for recreational duck hunting is not such a right. Baldwin v. Montana Fish & Game Comm'n, 436 U.S. 371 (1978); E Ch.7-IV(B)(3)(a)(ii); CH p. 473.

The State of Corinthia is building a new state capitol building. A state statute requires that 50% of the workforce on any state-funded construction project must consist of
state residents. The ordinance was enacted in response to a study that showed that many state construction workers were unemployed and forced to emigrate on this
basis. The ordinance is challenged on Article IV Privileges & Immunities grounds by some out-of-state construction workers denied jobs on the capitol project. The state
claims that the interstate Privileges & Immunities Clause doesn't apply to the government as a market participant. Is the government's argument correct?
No—but it may still win the case.
The interstate Privileges & Immunities Clause prevents states from discriminating against out-of-state citizens and residents as regards “rights fundamental to national
unity.” There is no exception for the state as a “market participant.” However, the state may still prevail if it proves that protecting jobs is a substantial state purpose and
that the statute is a reasonable means of attaining this goal. United Building and Constr. Trades Council v. Camden, 465 U.S. 208 (1984); E Ch.7-IV(B)(5); CH pp. 472-474.

RELATED ISSUE:
Say, instead, that the high unemployment rate in Corinthia was not due to a lack of job opportunities, but rather due to the fact that the workforce lacked job skills and
education and was by and large too far away from the job opportunities that did exist in Corinthia. Because the state's preferential hiring would not remedy the problem
but would still adversely impact non-residents, it would violate the interstate Privileges & Immunities Clause. Hicklin v. Orbeck, 437 U.S. 518 (1978).

NOTE:
There is a “market participant” exception for states under the Commerce Clause. The rationale for the distinction is that the Commerce Clause only addresses regulation,
and when the state itself is a market participant (it's buying or selling), it isn't “regulating.” By contrast, the interstate Privileges & Immunities Clause bars all discriminatory
state conduct, whether regulatory or not, if it unfairly restricts essential activities and basic rights of out-of-staters. E Ch.6-I(M).

State B requires anyone who enters a bicycle race in the state to have a bike racing license. The license fee is $25 per year for citizens of state B and $50 per year for all
others. Mack is a citizen of state A who wishes to enter a bike race in state B. He objects to paying a fee that is double that paid by state B residents. Can Mack invoke the
Article IV Privileges and Immunities Clause to challenge the discriminatory fee schedule?

The Privileges and Immunities Clause will come into play only if the state is discriminating against citizens of other states with respect to a fundamental right. The state
will argue that the interest in bike racing does not involve an interest that is protected by the clause because it is purely recreational in nature. In Baldwin v. Fish & Game
Commission of Montana, 436 U.S. 371 (1978), the Court held that Montana’s discriminatory license fees for elk-hunting licenses did not implicate the Privileges and
Immunities Clause since elk hunting was a recreational rather than a commercial pursuit and thus did not involve a fundamental right under Article IV, §2.
However, Mack might reply that even if bike racing, like elk hunting, is purely recreational, the state still violated his fundamental right to “exemption from higher taxes
or impositions than are paid by the other citizens of the state,” Corfield, supra, 6 F. Cas. at 552, an argument that the Baldwin Court did not address. Mack might also be
able to show that he depends for all or part of his livelihood on bike racing, in which case the Privileges and Immunities Clause might be triggered, for “one of the privileges
which the clause guarantees to citizens of State A is that of doing business in State B on terms of substantial equality with the citizens of that State.” Toomer v. Witsell, 334
U.S. 385, 396 (1948).

State X recently passed a law barring anyone from working as a licensed plumber unless he or she passes the state plumbing exam (SPE) and serves three years as an
apprentice to a licensed state X plumber. Paul is a citizen of state Y. He has been a plumber for many years and has done a great deal of plumbing work in state X.
However, Paul can no longer work as a plumber in state X because he has neither passed the SPE nor been apprenticed to a licensed state X plumber. May Paul challenge
the new state X law on the ground that it violates his rights under the Privileges and Immunities Clause of Article IV?
Since pursuing a trade is a fundamental right under Article IV, §2, the law affects an interest that falls within the purview of the clause. State X will argue that its law does
not discriminate against citizens of other states; rather, it requires everyone, including its own citizens, to meet the statutory requirements before they may work as a
licensed plumber. On this basis the state would urge that the Privileges and Immunities Clause is not triggered here.
Paul might respond that while the law is facially neutral, the apprenticeship requirement should be subject to scrutiny under the clause because it has the practical effect
of discriminating against out-of-staters like himself. Citizens of other states are far less likely than state X citizens to have been apprenticed to a plumber licensed by state
X. It is unclear whether the Court would find such non-facial discrimination enough to trigger the Privileges and Immunities Clause. In Hillside Dairy Inc. v. Lyons, 539 U.S.
59, 67 (2003), the Court noted that while “the absence of an express statement . . . identifying out-of-state citizenship as a basis for disparate treatment is not a sufficient
basis for rejecting” a claim made under the Privileges and Immunities Clause, it remains an open question whether the clause applies only to “classifications that are but
proxies for differential treatment of out-of-state residents” or whether it should also be deemed to reach “classifications with the practical effect of discriminating against
such residents. . . . ” At least one court has held that an as-applied Privileges and Immunities Clause challenge may be made to a facially neutral state law. See Daley v.
Grajec, 2007 WL 2286132 (S.D. Ind. Aug. 7, 2007). Thus, depending on the jurisdiction, Paul might be required to prove that the discrimination was intentional—i.e., that
state X adopted the apprenticeship rule because of its discriminatory effect on out-of-staters; otherwise, the law’s unintended discriminatory effect might not suffice to
bring the clause into play. If Paul can persuade the Court that the apprenticeship rule is discriminatory within the meaning of the clause, the rule will be invalidated unless
the state can show that there is a substantial reason for the discrimination. See §9.5 (“The Substantial Reason Test”).

The State of South Dakota operates its own cement plant. The state has a policy of favoring local buyers of cement over out-of-state buyers during periods of cement
shortages. Under this policy, the state refused to sell cement to Reeves, Inc., a Wyoming corporation, on the ground that Reeves was an out-of-state entity. Can Reeves
challenge South Dakota’s policy under the Article IV Privileges and Immunities Clause?
No. As a corporation, Reeves is not protected by the clause. Although South Dakota is discriminating against out-of-staters with respect to the fundamental right to do
business (i.e., buy cement) in the state, Reeves lacks standing to invoke the Privileges and Immunities Clause since it would be invoking the constitutional rights of third
parties rather than its own rights. See §3.4.5 (“The Rule Against Third-Party Standing”). Reeves might challenge the policy under other constitutional provisions, and a
Wyoming individual or partnership could challenge it under Article IV, §2; however, as we will see in §9.5.2, since a state-owned good is involved, the South Dakota
policy would probably not violate the Privileges and Immunities Clause. Cf. Reeves, Inc. v. Stake, 447 U.S. 429 (1980) (Wyoming corporation unsuccessfully challenged
South Dakota cement policy under the dormant Commerce Clause but did not invoke the Privileges and Immunities Clause).
South Carolina imposes a license fee on commercial shrimp boats. The fee is $25 for boats owned by state residents, and $2,500 for boats owned by nonresidents. Moe
operates a commercial shrimp boat in South Carolina waters. Because he is a citizen of Florida, he had to pay South Carolina a fee of $2,500. Can Moe successfully
challenge South Carolina’s fee structure as violating Article IV’s Privileges and Immunities Clause?
The fee statute affects two fundamental rights that are within the purview of the Privileges and Immunities Clause, for it impairs Moe’s right to do business in the state
and his right to “an exemption from higher taxes or impositions than are paid by the other citizens of the state. . . . ” Corfield v. Coryell, supra, 6 F. Cas. at 552. Moreover, the
law clearly discriminates against citizens of other states who must pay a fee 100 times larger than that paid by South Carolina citizens. The fee statute will thus be struck
down unless it passes the substantial reason test.
The state must first prove that nonresidents pose a special problem that warrants charging them a higher fee than the state charges its own citizens. The state might argue
that its goal is to conserve the supply of shrimp by discouraging nonresidents who use larger fishing boats and thus take a disproportionate number of shrimp from state
waters. Or the state might claim it is more difficult to enforce its fishing regulations against nonresidents because they live outside the jurisdiction. For either of these
arguments to succeed, the state must prove that nonresidents in fact pose the unique evils attributed to them. If, for example, large boats are sometimes also used by
residents, the “larger boats” rationale will not provide a valid reason for treating noncitizens differently. Alternatively, the state would have to prove that noncitizens are
actually more likely to ignore its fishing regulations or that there are demonstrable extra costs entailed in enforcing the state’s fishing regulations against them.
Even if South Carolina shows that citizens of other states pose either or both of the peculiar evils ascribed to them, the state must also prove that it chose the least
discriminatory and least restrictive feasible means of achieving its goals. With respect to conservation, a less discriminatory and less burdensome alternative would be to
charge all boat owners (resident and nonresident) a graduated fee based on the size of a particular vessel. As to the enforcement goal, a less restrictive means would be to
charge nonresidents a higher fee designed merely to compensate the state for the added costs of enforcement; it is unlikely that the extra enforcement costs would justify
a fee 100 times greater than that charged locals. See Toomer v. Witsell, 334 U.S. 385 (1948) (invalidating similar South Carolina shrimp boat license fee schedule).

Arms Co. is a government military contractor. Its sole business is to manufacture M-16 rifles, all of which are then sold by it to the U.S. government. Arms Co.’s contract
with the U.S. government is of a “cost plus fixed fee” variety. Okra, the state in which Arms Co. is located, has enacted a generally-applicable sales tax on all wholesale sales
of tangible property made within the state. Okra applies this tax to Arms Co., with the result that Arms Co. must pay a tax of 8% on all goods bought by it for use in the
manufacture of the M-16 rifle. The net effect of this tax is to increase by 4% the price paid by the U.S. government to Arms Co. for each M-16 rifle.
(a) If Arms Co. wishes to attack the constitutionality of the Okra sales tax as applied to its purchase of materials for use in the M-16, what is its best argument?
_________________
(b) Will this argument succeed? _________________
(a) That the tax violates the federal immunity from state taxation.
(b) No. It’s true that the federal government is itself immune from taxation by any state. But this immunity generally does not extend to federal government employees or
to contractors who work for the federal government. And it doesn’t matter that the “economic incidence” of the tax falls on the federal government — as long as it is not
the federal government itself that is being directly obligedto pay the tax, the fact that the economic burden of the tax may fall on the federal government is irrelevant.

Equal Protection Analysis


Understand Equal Protection analysis.
The State of Ames bars opticians from the manufacture of eyeglasses unless pursuant to a prescription from either an optometrist (who has a doctor of optometry degree)
or an ophthalmologist (who has a medical degree). McGoo, an optician (who has neither degree), sues, claiming that the law simply places low-cost providers at a
disadvantage relative to the other eye doctors. A reviewing court would likely:
Uphold the law, because the legislature could have rationally believed that citizens’ health and safety were better served by having licensed eye professionals prescribe
eyeglasses.
The facts here are similar to those in Williamson v. Lee Optical of Okla., Inc. 348 U.S. 483 (1955), in which the Court upheld a law similar to that in the question. As in due
process cases, we know that states may exercise their police power to regulate the health, safety, and welfare of their citizens. The regulation of the manufacture of
eyeglasses does not, as far as we know, constitute a “suspect” classification. The rational basis test applies, therefore. The aims — consumer protection and protection of
citizens’ health and welfare — are undoubtedly legitimate. We also know that the Court no longer reviews legislation for its wisdom or reasonableness. Therefore, D is
not correct. We also know that the Court will uphold a classification if there is any conceivable purpose that the classification serves. Therefore, C is not correct. (In fact,
it is not clear that even if that were the purpose, it would be illegitimate.) Because the Court will not second-guess legislative judgments implicit in its
classifications, B isn’t correct either. Obviously, the legislature thought that ophthalmologists and optometrists were better qualified than opticians to prescribe
eyeglasses. That leaves A, which is the correct answer: The legislature could have rationally believed that licensed professionals, including ophthalmologists and
optometrists, are better suited than opticians to accurately prescribe eyeglasses for their patients

The State of Ames requires that all uniformed police officers retire at age 50 in an effort to ensure their physical fitness and mental. The law does not apply to detectives
or other plainclothes police officers. Phil sues, claiming that the law violates equal protection. He notes that many officers under 50 are neither physically fit nor mentally
alert. He also argues that it is a violation to single out uniformed cops for this treatment while allowing detectives 50 years old and older to remain on the force. A
reviewing court should:

d. Uphold the requirement, because the fit between the means and ends need not be especially precise, as long as it is not completely without foundation.

Rationale for Correct Answer


These facts are taken from another case, Massachusetts Board of Retirement v. Murgia, 427 U.S. 307 (1976). Again the Court upheld the law. Remember that while there
needs to be a rational relationship between means and ends, that fit does not have to be precise. Simply because a classification tends to cover too much or too little relative
to the purpose does not mean that the classification is invalid — at least, not if the classification is a non-suspect one. Therefore, neither A nor B is correct. The Court has
said repeatedly that states may regulate piecemeal. Further, classifications necessarily discriminate between classes; therefore, C is incorrect because it is irrelevant. Only
discrimination against members of protected classes garners strict scrutiny. That leaves D, the correct answer. It is not completely unreasonable to think that, on the
whole, physical prowess and even mental acuity decline in late middle age; even if individual counterexamples are available, the state is not required to make allowances
for those and may instead draw a bright line.

The State of Ames permits same-sex couples to serve as foster parents. Recently, however, the State prohibited homosexual couples from permanently adopting children,
even those for whom they have been longtime foster parents. During floor debates, several members pointed out that data suggest that married, heterosexual
households furnish a more stable home environment for children, and that they opposed adoption by gay couples on that ground. Others expressed hostility toward
homosexuals and homosexuality in general. Rob and Steve, longtime foster parents to Amy, wished to adopt her and sued the state, claiming that the ban on adoption
was unconstitutional. Which of the following would be helpful to them in overturning the ban?

While in most cases, courts applying a rational basis test will assume facts sufficient to create a rational relationship between means and ends, cases like Cleburne have
seen the Court apply a less deferential version in which it looks closely at the fit between the end of a law and the means used to achieve it. If there is a particularly poor
fit, the Court, as in Cleburne, may question whether the purpose given is a pretext for some less legitimate purpose for the legislation. In Cleburne the Court felt the fit
was so poor that only an irrational fear of the disabled could be behind the special-use permit requirement. Here the reason given for Ames’s law is to promote the
interest of children, which allegedly is best furthered by adoption into heterosexual, married families. If evidence existed undermining that premise, one could argue that
there was a poor fit between the purpose and the ban. Therefore, A would be very helpful: If you could show that long-term foster parent relationships (including those
involving gay men and women) were no less stable than adoptive ones, that would call the ban into question. Similarly, B would be helpful in showing that the state, in
practice, did not necessarily believe that only married couples were fit to adopt. In addition, if you could demonstrate C — that animus was really behind the ban — that
would enable you to invoke Moreno and its claim that legislating disabilities against identifiable groups because of animus was not a legitimate governmental interest.
Because each selection would be useful in overturning the ban, the correct answer is D.

The city council adopted an ordinance outlawing door-to-door sales in residential neighborhoods. The law contains an exemption for agents of the Bigelow Insurance
Co., who are permitted to sell door to door in all parts of the city. Sherry, an agent for the Miller Insurance Co., was arrested and criminally charged with violating the
ordinance. She has asked the court to dismiss the criminal action on the ground that the ordinance violates the Equal Protection Clause of the Fourteenth Amendment.
How should the court rule?

The ordinance is probably constitutional. Even if we have no idea why the city council adopted the measure, the classification will be upheld if there is any conceivable
legitimate reason for treating Bigelow more favorably than other door-to-door sellers. The council may have believed that Bigelow agents are more polite than other
door-to-door sellers. Or the council may have thought that Bigelow had been operating in the city longer than any other door-to-door company, giving it an equitable
claim to favored treatment. Even if none of these hypothetical rationales was in fact what moved the city council, it is enough that the classification might have been
adopted for one or more of these reasons. See New Orleans v. Dukes, 427 U.S. 297 (1976) (upholding exemption to ordinance barring most but not all pushcart vendors
from certain parts of city).

For decades, the City funded sewer projects by apportioning a project’s costs equally among all abutting lots. Lot owners could elect to pay the assessment in a lump sum
or over time in installments. The Sewer Project B involved 180 abutting lots. Forty of the lot owners affected by that project paid the assessment in full, while the
remaining 140 opted to pay by installment. The debts were relatively small, and the monthly payments on the installments were in the range of $25 per month. The City
later decided to abandon this funding model and to fund all future sewer projects through the issuance of bonds. As part of the new approach and in an effort to save
administrative costs, the City forgave the remaining amounts owed to it by Project B lot owners who had elected to pay their apportioned fee in installments. The City
did not, however, offer a refund to those Project B lot owners who had paid the assessment in full. The paid-in-full lot owners now claim that the City’s refusal to provide
an equivalent refund to them violated the Equal Protection Clause. How is a court likely to rule on their claim?
Because the paid-in-full lot owners’ claim does not involve a suspect or quasi-suspect classification and does not implicate a fundamental right, the City’s decision not to
offer them a refund will be upheld if there is any rational basis for the City’s decision. The City could justify its choice as a cost-saving measure because the debt
forgiveness will likely save the City the administrative cost of policing these relatively small debts. In other words, the City could rationally conclude that the cost of
collection and administration no longer justified whatever incremental benefit the City might derive from collecting these small debts. As to the failure to offer an
equivalent refund to the paid-in-full lot owners, the City might argue that offering them a refund would actually add more administrative costs to the City’s effort to
transition to a new funding model. Given these arguments, a court would likely conclude that the City acted rationally. See Armour v. City of Indianapolis, 132 S. Ct. 2073
(2012) (so ruling on similar facts).

Norman moved from Illinois to Vermont shortly after buying a car in Illinois. Vermont requires its citizens to pay a use tax upon registering a car that was purchased in
another state. However, Vermont grants an exemption from the use tax for any sales tax paid to another state if, at the time of purchase, the registrant was a Vermont
citizen. When Norman registered the car in Vermont, the state charged him the full use tax, with no credit for the sales tax he had paid to Illinois, because at the time of
purchase he was not a Vermont citizen. Did Vermont violate the Equal Protection Clause by discriminating against people, such as Norman, who were recently citizens of
another state?
Under the traditional rational basis test, Vermont’s tax would easily be upheld. The difference in treatment may have been based on the assumption that those who lived
in another state when they bought a car benefited from that state’s roads. Since they will derive benefits from the roads in two states, it is not unfair that they pay taxes
to two states. By contrast, those who were Vermonters when they bought a car in another state may have derived little benefit from the other state’s roads and thus
should not have to pay taxes to two states. This sort of speculation clearly affords a rational basis for the Vermont law. However, in Williams v. Vermont, 472 U.S. 14, 22
(1985), the Court, ostensibly applying the traditional rational basis test, invalidated this Vermont tax scheme as creating “an arbitrary distinction that violates the Equal
Protection Clause.” One explanation for the result is that the Court applied an enhanced form of rational basis review.

The state of Chartreuse has delegated to the state bar association the job of determining requirements for the practice of law. Until recently, the bar association imposed
no continuing legal education requirements. However, there has been a large rise in the number of malpractice actions against lawyers in the state, some of which appear
to derive from the fact that lawyers have not kept pace with changing legal principles. Also, there has been a rise in the disdain with which the public in Chartreuse holds
lawyers. The bar association, therefore, imposed a mandatory continuing legal education requirement of 12 course-hours per year. The state imposes on doctors a
continuing education requirement of only 4 hours per year. There is strong evidence that, in general, changes in medicine happen faster than changes in law. Amos, a
lawyer in Chartreuse, has challenged the continuing legal education requirement on the grounds that by imposing a much higher requirement on lawyers than on doctors,
the state has violated his equal protection rights. Assume that the state bar association’s conduct constitutes state action.
(a) What standard should the court use to test whether the legal education requirement satisfies the demands of equal protection? _________________
(b) Will Amos’ attack succeed? _________________
(a) The court should ask whether the classification is rationally related to a legitimate state objective.
(b) No. Unless there is a suspect or semi-suspect class involved, or a fundamental right, the Supreme Court will uphold a state legislative classification against equal
protection attack so long as the classification scheme bears a rational relation to some legitimate legislative objective . The desire to have lawyers be better trained,
and to increase public confidence and reduce suits, certainly seem to constitute legitimate governmental objectives. The link between the classification system chosen and
the objective being pursued does not have to be a close one. In fact, there does not need to be an actual empirical link between means and end, merely a rational belief on
the part of the legislature that there is a link between means and end. Here, a bar association (acting as a government body) certainly could rationally have believed that
lawyers who take mandatory legal education will be more up to date, and thus less likely to make mistakes through outmoded training. The fact that doctors might need
refresher courses even more than lawyers will be ignored by the court, because reform may take place “one step at a time” (i.e., the government does not commit an equal
protection violation every time it addresses one aspect of a problem without solving all related aspects).

What is the rational relation test?


It states that for a state law to be valid, there need only be a set of facts imaginable that would make the law a reasonable method of achieving a legitimate governmental
purpose. All economic and social legislation is subject to this test, as are classifications based on wealth, age, and mental retardation. In addition, if the burden on a
fundamental right is not "undue" (so that the law is not subject to strict scrutiny), the rational relation test will apply.

NOTE:
It is difficult for a law to fail this test; however, watch for impermissible state motives, especially any indication that the state has acted with "animus" toward a group or
a person. E Ch.10-I(B)(8)(a), 10-II(E); CH §9.2.
You already know that the rational relation test is very difficult to fail, and that if a statute isn't subject to strict or intermediate scrutiny, this is probably the kiss of death
for an equal protection claim. However, there are a few situations in which laws subject to the rational relation test are struck down. When is this most likely to happen?
When either the law rests on grounds wholly irrelevant to achieving the state's objective or the objective itself is invalid.
For instance, say a state charges a higher tax on out-of-state businesses than on in-state businesses. If the state's motive is to protect local businesses from competition,
the interest is invalid, so the tax will fail the rational relation test. Metropolitan Life v. Ward, 470 U.S. 869 (1985); CH p. 464.
Or, let's say the state amends its constitution to prohibit the state, or any city in it, from giving gay individuals protection from job discrimination. The state's "bare desire
to harm a politically unpopular group" (such as gay individuals) can't be a legitimate governmental interest. So the amendment must be struck down, even though it's only
subject to the rational relation test. Romer v. Evans, 517 U.S. 620 (1996); E Ch.10-II(E)(1)(b)(ii).

RELATED ISSUE:
The Court relied on substantive due process to invalidate a Texas statute that penalized homosexual, but not heterosexual, sodomy, concluding that the right to privacy
protected by the Due Process Clause of the Fourteenth Amendment included a right to adult consensual homosexual activity. Justice Sandra Day O'Connor's concurrence
argued that the Texas statute should have been invalidated under the Equal Protection Clause because it singled out homosexuals for government disfavor, failing rational
basis scrutiny.

A state regulation denies employment in state-owned transportation facilities to individuals undergoing methadone treatment. The regulation is attacked on equal
protection grounds. What level of scrutiny will be used in testing the validity of the regulation?
The rational relation test. As such, the regulation need only be rationally related to a legitimate state interest to be valid.
The rational relation test is used for equal protection problems that don't involve a "suspect" class (e.g., race or alienage), don't unduly burden a fundamental right, and
don't involve a "quasi-suspect" class (e.g., illegitimacy or gender). Here, the classification involves those in methadone treatment–not a suspect or quasi-suspect class. And
public employment is not a fundamental right. Because the state could have any number of valid reasons for the discriminatory classification (i.e., safety), the classifications
will be upheld. New York City Transit Authority v. Beazer, 440 U.S. 568 (1979); E Ch.10-II(A); CH pp. 686-687.

The State of Golden Pond becomes concerned with the large number of injuries associated with aerobics classes. It enacts a statute preventing aerobic classes from being
taught by anyone except a licensed physical therapist. The statute has a "savings" clause exempting all those aerobics teachers already teaching on the date the statute is
enacted. Would this clause withstand an equal protection attack?
Yes. When a state action creates a classification involving economic or social legislation (and not otherwise involving a suspect or semisuspect class or fundamental right),
the classification's validity is judged by the rational relation test–that is, the statute is valid if there is a set of facts imaginable under which the action would be rationally
related to a legitimate state goal. The classification here is between present and future aerobics instructors, and because there could be a rational basis for the distinction
(e.g., present instructors have a reliance interest in being able to continue to practice their profession, but future ones don't), the statute is valid. E Ch.10-II(B).

The State of Geyser produces a great deal of oil from state-owned lands. It credits the profits from the sales of the oil to its residents, at a rate of $50 for each year the
person has resided in the state. Would this practice withstand an equal protection attack?
No. Equal protection problems arise when the state creates a classification. When the legislation is social or economic, the rational relation test determines its validity–if
there is a set of facts conceivable under which the legislation is rationally related to a permissible state goal, it will be valid. The problem here is that the goal isn't valid;
it's not permissible for a state to award benefits based on how long a person has resided in the state. Zobel v. Williams, 457 U.S. 55 (1982). Thus, the credit program will
fail. E Ch.10-II(E)(2)(b); CH p. 683.

The legislature of the State of Greenery is very sensitive to environmental issues. Pursuant to this concern, it requires that all infant goods used in the state–including
diapers and baby bottles–must be recyclable. Burp 'N Change Baby Co., located in Greenery, makes disposable diapers, as well as baby formula th at comes in a non-
recyclable plastic six-pack. Burp 'N Change challenges the recycling requirement on equal protection grounds, showing that a neighboring state that has the same ban has
not shown any decrease in the number of diapers and bottles thrown away, nor a measurable decrease in total trash. What result?
The statute will be upheld. Equal protection problems arise when a state makes a classification. Here, the classification is manufacturers of recyclables versus
manufacturers of non-recyclables. Because this isn't a suspect or quasi-suspect classification, the rational relation test applies-the statute is valid if there is a set of facts
imaginable under which the classification would be rationally related to a legitimate state goal. Here, the legislature could feel that banning disposable diapers and bottles
would help preserve the environment, a legitimate state goal. It doesn't matter whether the recycling will actually reduce litter, as the legislature needn't prove a statute
will have the desired effect. Instead, a law only fails the rational relation test when either it rests on grounds wholly irrelevant to achieving the state's objective, or the
objective itself is invalid. E Ch.10-II(G).

After their TV show goes off the air, Ricky and Lucy Ricardo open up their own nightclub, where Lucy finally gets a chance to be in "the show." She bombs, and so does the
club. Lucy's antics start driving Ricky crazy, and he wants to divorce her, but he can't afford the $75 divorce fee charged by the state. He challenges the fee on equal
protection grounds. What result?
The fee will be struck down. Although poverty is not a suspect or quasi-suspect classification, when a statute involves a classification based on exercise of a fundamental
right, it will be subject to strict scrutiny. The fundamental right of privacy includes marriage rights, which in turn includes the right to dissolve a marriage. The restriction
of the right to divorce on the basis of who can pay for it, coupled with the fact that the state holds a monopoly as to the means of dissolving a marriage, means the state is
unduly burdening a fundamental right. As a result, the fee is impermissible. Boddie v. Connecticut, 401 U.S. 371 (1971); E Ch.10-X(E)(8).As this hypothetical indicates,
watch out for the possibility of heightened scrutiny whenever poverty prevents someone from exercising a fundamental right!
ANALYZING EQUAL PROTECTION ISSUES
Keep in mind that this is a very bare-bones guide and doesn't include every nuance of equal protection by any stretch of the imagination; however, it does give you a basic
roadmap for examining equal protection. Ask these questions:
1. Is the action involved a state action?
If so, go on to #3. If not, go on to #2.
2. Is the action involved a federal action?
If so, and it involves an area of unique federal power (e.g., power over immigration or war), it's subject to the rational relation test. Otherwise, federal actions
are treated like state actions for equal protection purposes, but you should keep in mind that the Equal Protection Clause doesn't apply to the federal
government; the Due Process Clause of the Fifth Amendment, which does bind the federal government, is said to include an equal protection guarantee. For
federal actions, go on to #3.
If not (i.e., the action is by neither the state nor the federal government, but by a private individual), there's no equal protection problem, because private
individuals aren't bound by the equal protection guarantee.
3. Does the government action create a classification (e.g., men vs. women, resident vs. non-residents, legitimate vs. illegitimate children)? If so, go on to #4. If
not, there's no equal protection problem; check for substantive due process.
4. Is the governmental discrimination intentional?
Discriminatory effect in and of itself is insufficient to trigger strict or intermediate scrutiny; the statute must be facially discriminatory or intentionally
unequally administered or have a discriminatory motive. A government act that only has a discriminatory effect is subject to the rational relation test,
regardless of the basis of the classification. Go on to #5.
5. What's the basis of the classification?
a. Is the classification based on race?
If so, the classification will be subjected to strict scrutiny: It must be necessary to promote a compelling state interest. This is now true even if
the statute is affirmative action (see Richmond v. J. A. Croson Co., 488 U.S. 469 (1989)), and even if the affirmative action is being done by Congress
rather than by a state (see Adarand Constructors, Inc. v. Pena,515 U.S. 200 (1995)). Strict scrutiny will also be applied where the government is
making a race-conscious decision to further operational needs, as for prison administration. Johnson v. California, 543 U.S. 499 (2005); E Ch.10-
IV(L).

NOTE:
A race-based affirmative action measure might nonetheless be valid if it is very narrowly tailored and is designed to remedy the government
entity's own past discrimination (not just discrimination in general).
If the classification is not based on race, go on to (b).
b. Is the classification based on alienage?
If so, go on to (c). If not, go on to (e).
c. Who's creating the (alienage-based) classification?
If the federal government, it's subject to the rational relation test; it need only be rationally related to a legitimate governmental interest. If the
state, go on to (d).
d. Does the (alienage-based) classification involve a "function at the heart of representative government"?
If so, it's subject to the rational relation test; it need only be rationally related to a legitimate governmental interest. If not, it's subject to strict
scrutiny.
e. Does the classification unduly burden a fundamental right (i.e., the right to vote; the right to have access to the courts; the right to travel; and
(possibly) the right to be a political candidate)?
If so, it's subject to strict scrutiny: It must be necessary to promote a compelling state interest. If not, go on to (f).
f. Is the classification based on legitimacy or gender?
If so, it's subject to intermediate scrutiny: It must be substantially related to an important state interest. If not, go on to (g).
g. All other classifications are subject to the rational relation test-including virtually all classifications arising in "economic" and "social-welfare"
legislation, and all classifications based on or involving age, mental retardation, poverty, welfare, housing, education, government employment,
homosexuality, and access to the judicial process when no fundamental right is involved.

6. What are the three levels of scrutiny courts use in analyzing equal protection problems?
7. Strict scrutiny, intermediate scrutiny, and the rational relation test.
8. As a general matter, strict scrutiny requires that the classification in question must be necessary to promote a compelling governmental interest. Classifications
subject to strict scrutiny are called "suspect classes." There are three classifications that are, in general, subject to strict scrutiny–those based on race, those
based on alienage, and those that unduly burden, discriminate, or penalize the exercise of a fundamental right.
9. Intermediate scrutiny requires that the classification in question be substantially related to an important governmental interest. Classifications subject to
intermediate scrutiny are called "quasi-suspect." There are two classifications that qualify for this treatment–gender and illegitimacy. (Gender classifications
must be supported by an "exceedingly persuasive justification.")
10. Everything else is considered "nonsuspect" and is subject to the rational relation test–the classification need only be rationally related to a legitimate
governmental interest. All economic and social welfare classifications fit into this category, including some that you might be tempted to consider suspect or
quasi-suspect, like poverty, welfare, housing, education, government employment, and age. E Ch.10-I(B)(8).
11. Is there a constitutional requirement that states outlaw discrimination?
12. No. They are only forbidden from authorizing or encouraging it
13.
What is the strict scrutiny test?

14. It requires that, for a law to be valid, the law must be necessary to promote a compelling government interest. A statute is only necessary if there are no less
discriminatory alternatives available to attain the goal. In the equal protection context, this test applies to state actions that create either a suspect
classification and discriminate against a suspect "class," or a classification that unduly burdens, discriminates against, or penalizes the exercise of a
fundamental right. E Ch.10-I(A)(3).
15. NOTE:
16. It is very difficult for a law to pass the strict scrutiny test.

17. NOTE:
18. Explicitly guaranteed rights (like those in the Bill of Rights) are usually analyzed by themselves, not as an equal protection issue (e.g., free speech, freedom of
religion).

19. NOTE:
20. The strict scrutiny test is also used for substantive due process problems involving a governmental action that impinges on a fundamental right. E Ch.10-
III(D)-(E); CH pp. 671-674.

With numerous websites urging college students to apply for food stamps as a means of dealing with the high cost of tuition and books, Congress amended the
program to disqualify any person enrolled as a full-time student. Members of Congress overwhelmingly support the amendment and used the floor to criticize the
abuse of the program by middle-class young people whose parents probably still claim them as dependents. Congress notes that such persons were never meant to
benefit from the plan and that scarce resources must be conserved for the truly needy, whose numbers have risen during the recent economic downturn.

21. If a federal court upholds the amendment, it is probably because:


22. a. The law is rationally related to the legitimate interest of conserving scarce resources in a governmental relief program.
23. Remember we’re looking for reasons that a court might not invalidate the amendment. Ordinarily, unless a law classifies on the basis of a suspect
characteristic, such as race, courts will apply a deferential version of rational basis review, cases like Morenonotwithstanding. That means that B is not the
correct answer, because it does not recite the appropriate standard of review here. Nor is C correct. While the Fourteenth Amendment does apply only to
states, the Court has found an equal protection component in the Fifth Amendment’s Due Process Clause, which does bind the federal government. While
true, D is not the best answer; as the text above indicates, sometimes courts will invalidate laws even where no fundamental right or suspect classification is
involved. Afurnishes the best reason a court would uphold the law: Conserving scarce resources for their intended purpose is a legitimate interest, and
barring from eligibility those who Congress believes are abusing aid programs is a rational means of effecting that interest. Note too, on the facts present,
that that belief appears to be a sincere one. It does not appear that Congress is out to “get” college students the way that, in Moreno, some members of
Congress allegedly targeted “hippies” for exclusion. In other words, something more than the bare desire to target the politically unpopular is at work here.
24. Ames City, a city in the State of Ames, historically compiled jury pools using the state’s list of registered drivers. Recently, however, it began to use voter
registration records as a basis for selecting jurors. As a result, the number of African American jurors fell by over 80 percent. Which of the following would
allow for an inference of racial discrimination?
Rationale for Correct Answer
While discriminatory intent is required to make out a prima facie case of racial discrimination under the Fourteenth Amendment, plaintiffs may compile evidence that
would allow a fact finder to infer discriminatory intent even where it does not appear on the face of a law or other official action. Among the factors mentioned
in Davis and Arlington Heights were disparate impact (probative of intent, but not sufficient on its own); the historical background of decisions, including evidence of past
discrimination; and legislative history. Those factors are present in A, B, and C, which means that the best answer is D. The presence of those conditions could allow a fact
finder to infer discriminatory intent from the switch.

Ames City mandates that boys and girls in kindergarten through eighth grade be taught in separate classrooms. Data suggest that children of that age benefit from single-
sex classrooms. If Ames City’s policy is challenged in federal court, which of the following would be most helpful to the city’s argument that its policy is constitutional?
b. Data showing a significant link between educational performance and lifetime income as well as data showing increases in educational performance in single-sex
classes through the eighth grade.

This question was intended to test the Court’s distinction between actual and pretextual purposes, as well as the Court’s distinction between real differences and those
rooted in stereotypes. The facts indicate that Ames City has a single-sex classroom policy for children through the eighth grade. Of the proffered choices, the least helpful
would be D, because it suggests that the real reason for the separation has nothing to do with the educational achievement of children and is rooted, at best, in time-
honored custom. A is not much better, because it is rooted in generalizations that approach the stereotypical. While C points to an important governmental purpose —
preparing school-age children for success as adults — there’s nothing to indicate that the separation is substantially related to that important purpose. That leaves B,
which is the best answer: If the city can demonstrate both that it is in pursuit of that important purpose — adult success — and that the separation is related to that
purpose, it would go far to justify its policy. If, in other words, the state can show not only that increased academic performance yields benefits down the road, but also
that separating children by sex through the eighth grade increases academic performance, it has a stronger claim that its classification does not violate the Equal
Protection Clause.
A city fire ordinance prohibits the operation of fraternity or sorority houses in residential areas of the city, on the stated ground that the occupants of such houses may
have difficulty evacuating the building in the event of a fire. The city allows other types of group housing in residential areas, including regular student dormitories,
boarding houses, senior citizen housing, and nursing homes. Is the city’s discrimination against fraternity and sorority houses rationally related to a legitimate
governmental purpose?
Perhaps not. Unless the city can demonstrate that there is something about fraternity and sorority houses that exposes their occupants to special risk in time of fire, the
discrimination is literally irrational. To meet its burden, the city would have to explain why, in terms of fire safety, it allows student dormitories, boarding houses, and
senior citizen and nursing homes, while prohibiting fraternity and sorority houses. All would seem to be equally safe and equally risky in the event of a fire. In terms of
equal protection, in the absence of any relevant distinction between the permitted uses and the prohibited use, the classification would not rationally advance the city’s
avowed interest in fire safety.
If the ordinance had not identified fire safety as the purpose of the ban, the city could more easily defend the measure against an equal protection attack by arguing that
the discrimination against fraternity and sorority houses is rationally related to another legitimate governmental purpose—i.e., preserving residential tranquility. Since
the occupants of fraternity and sorority houses are often particularly loud and boisterous, the ordinance’s distinction between them and other types of group housing
would rationally further the goal of maintaining peace and quiet in residential areas. However, the fact that the ordinance specifically identified fire safety as the purpose
of the ban precludes any plausible inference that the reason for the discrimination was that of residential tranquility.
A state law requires that hospital emergency room personnel have been born in the United States. This law creates a suspect classification on the basis of national origin.
It was challenged by Carlos, a naturalized U.S. citizen who was born in Mexico and who, because of the law, was refused a job in a local emergency room. The state has
defended the law on the ground that it furthers a compelling interest in health and safety by assuring that emergency room personnel speak and understand English well
enough to communicate perfectly with patients and medical staff. If the court accepts this interest as being compelling, will the law survive Carlos’s equal protection
challenge?

Under the “narrowly tailored” prong of the strict scrutiny test, this law must “fit” its alleged health and safety purpose better than any feasible alternative. The law is
overinclusive in relation to its goal, for it applies to persons of foreign origin who speak and understand English perfectly. The law is also underinclusive, since it
apparently allows anyone born in the United States to work in an emergency room even if that person’s English is poor. If the state was really concerned about English
fluency in hospitals, it could have achieved its goal as well or better by requiring all prospective employees to pass an English proficiency exam. The facts that the law is
so poorly tailored to its alleged goal and that a less discriminatory alternative is available strongly suggest that the asserted health goal is a sham and that the law’s real
purpose is simply to burden people of foreign origin on the basis of prejudice or dislike. This law would, therefore, be found to violate the Equal Protection Clause.
A state law requires female workers to pay higher annual premiums to the state retirement fund than men, on the theory that, because they live longer, women will draw
retirement benefits for a longer period of time. The state has justified this discrimination on the basis that it needs to maintain the fiscal integrity of its pension plan. Is
this gender discrimination valid?
The goal of preserving the fiscal soundness of the state pension fund is no doubt an important state goal. Yet the means chosen are not substantially related to that
objective, since the state could achieve its ends without engaging in gender discrimination. The state could maintain the fiscal integrity of its plan by charging men and
women the same premium, with the total premiums collected being the same as they would have been under a gender-discriminatory plan.
If the state wishes to vary premiums based on an individual’s projected life span, it may not use gender as a proxy for longevity. Instead, the state must look beyond gender
to other factors that influence life span, such as smoking, drinking, diet, exercise habits, and the like. See City of Los Angeles v. Manhart, 435 U.S. 702 (1978) (invalidating
similar pension scheme under Title VII of the 1964 Civil Rights Act).

Strict scrutiny in racial classification cases


What are the elements of a suspect class?
The class must be:
1. Determined by unalterable characteristics (immutable);
2. Historically subjected to unequal treatment; and
3. Politically powerless.

MNEMONIC:
PUNCH (Powerless; UNalterable Characteristics; Historical mistreatment)

NOTE:
The only two classes that are considered "suspect" are race (or national origin) and alienage. If you read the elements of a suspect class literally, you'd have to wonder
why aliens are considered determined by "unalterable characteristics," because aliens can become "non-aliens" by being naturalized. Oh well. That's just the way it is. E
Ch.10-III(D); CH pp. 671-673.

The State of Enterprise has a law prohibiting interracial marriage. Lieutenant Uhura, who is African American, and Captain Kirk, who is Caucasian, want to marry. They
challenge the statute on equal protection grounds. The state argues that because the statute treats all races equally, it's valid. What result?
The State of Enterprise loses. In Loving v. Virginia, 388 U.S. 1 (1967), the Court invalidated a similar statute, stating that any racial classification, even one that treats the
races equally, should be subject to strict scrutiny. The Court observed that the law prohibited only interracial marriages involving white persons; non-whites could marry
each other. Its purpose was therefore to protect white supremacy, which is a violation of equal protection. E Ch.10-III(E)(4).

Under what circumstances will a zoning ordinance be considered an improper racial classification?
Only when it can be proven that the purpose and effect of the ordinance is to exclude members of a racial minority from a residential area. Arlington Heights v. Metropolitan
Housing Dev., 429 U.S. 252 (1977); E Ch.10-III(C)(5)-(6)

The State of Suspicion enacts a statute requiring that the race of candidates for public office be listed on ballots. Alf, a Melmackian running for sheriff, challenges this
requirement on equal protection grounds. Because the statute treats all candidates alike, regardless of their race, will the statute be valid?
No. The statute is discriminatory, and it violates equal protection.
To violate equal protection, the government must purposefully discriminate–that is, discriminatory impact alone isn't sufficient. A statute can be purposefully
discriminatory on its face, in its administration, or due to a discriminatory motive or intent. The statute here has a discriminatory intent, in that listing race on a ballot
would certainly support, and perhaps encourage, racial prejudice. As purposeful discrimination based on race, the statute must be necessary to support a compelling
interest to be valid. Because there is no legitimate interest in fostering prejudice, the statute is unconstitutional. Anderson v. Martin, 375 U.S. 399 (1964); E Ch.10-III(C),
(E)(5); CH p. 700.

COMPARE:
Recording the race of those seeking a divorce would be valid if records are kept for statistical purposes, as there would be no discriminatory intent (or impact) involved
in keeping such records.
For the last 100 years, the City of Alphaquad has used an at-large election system to elect its five-member governing body, called "the Federation." The city's population is
52% human and 48% Vulcan; however, the at-large electoral system has prevented any Vulcan from ever being elected to the Federation. Although there's no
discriminatory purpose evidenced by the system itself, and it wasn't adopted for that purpose, the city has been unresponsive to the needs of its Vulcan community. Would
the at-large system survive an equal protection challenge?

No. To violate equal protection, discrimination must be purposeful-that is, discriminatory impact, standing alone, doesn't make a governmental action invalid. A statute
can be purposefully discriminatory on its face, in its administration, or due to a discriminatory motive or intent. The issue in these facts is that what you've got is a
significant impact–no Vulcan has ever been elected to the Federation–which isn't enough, by itself, to make the at-large system discriminatory. However, discriminatory
impact can be evidence of an improper motive, and this, coupled with the City's unresponsiveness to Vulcan needs, would be sufficient to evidence purposefulness. A
purposefully discriminatory governmental action based on race must be necessary to promote a compelling governmental interest to be valid. Because discriminating
against minorities is not a legitimate governmental interest, the at-large system is unconstitutional. Rogers v. Lodge, 458 U.S. 613 (1982); E Ch.10-III(C)(8)(b); CH p. 900.

A State of Cootie statute provides that no retail smoke shop can operate in a wooden building without a license. The state licensing authority consistently refuses to grant
such licenses to Cuban Americans, while granting them freely to others. Ricky Ricardo, a Cuban American who's been denied a smoke shop license, challenges the statute
on equal protection grounds. What result?
The statute is invalid.
Equal protection problems arise when the state government creates a discriminatory classification. The issue here is the intentional nature of the discrimination, as the
statute on its face is not discriminatory. However, a statute may be discriminatory in two other ways: It may be unequally administered, or it may have a discriminatory
motivation. Here, the statute is administered on a racial basis–Cuban Americans versus non-Cuban Americans. Thus, the statute will be subject to strict scrutiny, and
because there's no compelling reason for it, it will fail. Yick Wo v. Hopkins, 118 U.S. 356 (1886); E Ch.10-III(C)(2); CH p. 715.

NOTE:
Because the statute is not discriminatory on its face, Ricardo would have to prove a discriminatory impact; once he does so, the state would bear the burden of proving
some non-discriminatory, constitutionally permissible explanation for the disproportionate impact on Cuban Americans. E Ch.10-III(C); CH pp. 717-719.

RELATED ISSUE:
Where a statute is discriminatory on its face, the claimant need not show a discriminatory impact. E Ch.10-III(C)(3); CH §9.3.3.1.

The State of Factoria has a statute requiring that all police officers take an oral test to ensure that they can communicate adequately in English. Although the test appears
racially neutral, a disproportionate percentage of African Americans fail it. The test is administered fairly, and the legislature did not intend to discriminate. Will the test
be subject to strict scrutiny?
No; de facto discrimination–where there is a disparate effect, but the statute is facially neutral, administered fairly, and the legislature did not intend to discriminate–is
not subject to strict scrutiny, even if a "suspect class" is affected. Instead, the rational relation test will be used: The statute will be upheld if it is rationally related to a
legitimate state interest. Because it is at least "legitimate" for government to ensure that its police officers are able to communicate in English, the statute will survive a
constitutional attack. Washington v. Davis, 426 U.S. 229 (1976); E Ch.10-III(C)(4); CH p. 710.
The City of Brotherhood has an ordinance requiring that contractors on city construction projects set aside 30% of their subcontracts for minority-owned businesses. The
ordinance was enacted after a city council study showed that only ½ of 1% of city contracts were awarded to minority-owned businesses, despite a 50+% minority
population. (There's no evidence that this paucity of minority contracts is due to past discrimination by the city.) Will the ordinance withstand an equal protection attack?
No. Where a state action creates a race-based classification, the strict scrutiny test will determine the action's validity, even if the action is designed to remedy past
discrimination. Richmond v. J. A. Croson, 488 U.S. 469 (1989). Thus, the action must be necessary to promote a compelling state interest. A government entity does have a
compelling interest in remedying its own past discrimination (whereas there is no compelling interest in the traditional form of discrimination against minorities). The
problem here is that there's no evidence of Brotherhood's own past discrimination, so there's no compelling interest.
The relevant statistic would be the number of qualified minority contractors, not the percentage of contracts awarded to minorities, which is insufficient evidence of past
racial discrimination to justify a race-based remedy. (For instance, if there were an inordinately small number of qualified minority-owned contractors, the ½ of 1% figure
would not indicate discrimination.) Richmond v. J.A. Croson,488 U.S. 169 (1989).
Even given that figure, the program here would not be considered necessary, as it's insufficiently narrowly tailored to withstand strict scrutiny. Note that rigid racial
"quotas," like the one here, are virtually never upheld. Richmond v. J.A. Croson, 488 U.S. 169 (1989); E Ch.10-IV(G).
Stringer Ball (an African American) challenges a state prison policy segregating new inmates for 60 days according to their race because he wants to be cellmates with his
best friend, Jimmy McNutty (a Caucasian). The state defends the policy as race neutral and necessary to prevent violence from race gangs. Who will win?
Not clear. The one thing that is clear is that strict scrutiny will apply, as it will for all racial classifications imposed by the government. In Johnson v. California, 543 U.S. 499
(2005), the Court remanded a prison segregation case to determine whether the segregation could survive strict scrutiny. The Court observed that prison segregation
could increase racial hostility, but also noted that prisons are dangerous places, and policies such as racial segregation may survive a strict scrutiny analysis as necessary
to promote the compelling interest of safety of officers and prisoners. E Ch.10-IV(L); CNR p. 413.

State statutes involving legal aliens are subject to strict scrutiny (except for "right to govern" situations). What about illegal aliens?
Discrimination against illegal aliens is not subject to strict scrutiny. However, where there's no congressional policy favoring a state rule discriminating against illegal
aliens, the state rule must meet "intermediate" scrutiny–that is, it must be substantially related to the achievement of an important state goal. (Note that a state could give
preferential treatment to citizens and legal resident aliens in allocating government benefits.) Plyler v. Doe, 457 U.S. 202 (1982).

EXAMPLE:
A state cannot deny a free, public education to the children of illegal aliens, because:
1. The children have no control over their status; and
2. The burden of a lack of education is enormous and lasting.

NOTE:
There haven't been significant cases on adult illegal aliens other than Plyler v. Doe; it's not clear how significant it was that the class of people at issue were not those who
had intentionally decided to illegally enter the country, but rather their children, who had done nothing wrong. E Ch.10-VI(E); CH §9.5.5.

The state of Minnetonka has enacted an adoption statute that specifies the criteria to be used when public agencies make adoption placements. (No private adoptions are
permitted in the state.) According to the statute, the “racial compatibility” between the child and the adoptive parents is the most important factor to be considered, though
many other factors are also to be considered. Peter and Jill, a white couple in their 40’s, wished to adopt a child. They learned that a large number of African American
children were being raised in state-sponsored orphanages, because no homes could be found for them. The couple applied to a state adoption agency, requesting to adopt
any African American child under the age of four, who was currently living in an orphanage. The adoption agency replied, “Because applicable statutes require us to give
racial matching heaviest weight in making adoption placements, and because you are seeking a relatively young child who may well be placeable with a racially-compatible,
i.e., African American family in the future, we cannot grant your request.” Peter and Jill have sued in federal court for a ruling that the statute providing for race-matching
violates their equal protection rights. They have presented evidence that African American children are staying longer in orphanages than white children who are
otherwise similar, and that this lengthier stay is due in part to the heavy statutory weight placed on race-matching.
(a) By what standard should the court test the constitutionality of the race-matching statute as it applies to Peter and Jill? _________________
(b) Should the court agree with Peter and Jill’s contention that their equal protection rights have been violated? _________________
(a) The court should apply strict scrutiny, and ask whether the statute is necessary to achieve a compelling governmental objective.

(b) Yes, probably. The Supreme Court has generally given stict scrutiny to any race-based classification scheme, even if it does not appear to be enacted for the purpose
of disadvantaging African Americans. In a somewhat analogous situation involving child custody, the Court held that child custody decisions could not be made by taking
into account race-matching or social prejudices; see Palmore v. Sidoti. So unless the mandatory consideration of race here is shown to be a necessary method of achieving
a compelling state objective, it will be struck down. The state undoubtedly has a compelling interest in guaranteeing that a child be raised by a family that will be compatible.
All other factors being equal, a racial match probably will make it easier for the child to develop a strong sense of identity. However, it is hard to see where the mandatory
first-preference for a racial match is a “necessary” means of achieving state objectives, especially in light of evidence that it may keep children in institutional care longer.
Therefore, the odds are that the court will find that the “necessary means” part of the equation has not been satisfied, and will strike down the mandatory first preference.
(But a scheme merely requiring that race be considered as one factor among many, without making it the most important factor, probably would survive even strict
scrutiny.)

Carolene High School, a public high school, has for years maintained special remedial reading classes. Students can take these classes in lieu of the regular English class, if
they are shown to have learning disabilities or otherwise shown to be behind on their reading skills. Students of African American ancestry make up 12% of the student
body of the high school. Forty-five percent of the students in the remedial reading program are African American. Because of reduced federal aid to education and a local
recession, the school board has reluctantly concluded that it must reduce expenditures. Therefore, it has cancelled the remedial reading program, which is about 15%
more expensive per student than the regular English program. The school board has not cut funding for the honors English prog ram, which similarly costs about 15%
more per student than regular English. About 9% of the students in the honors English program are African American. All of the evidence is that the school board made
the decision it did on the honest belief that the honors program accomplishes more per dollar and strengthens the school system more, than does the remedial reading
program. Carl, an African American student enrolled in the remedial reading program, has now challenged the termination on the grounds that it violates his right to equal
protection.
(a) What standard should the court use in deciding Carl’s challenge? _________________
(b) Will Carl’s challenge succeed? _________________

Answer
(a) The court should use the mere-rationality standard, not the strict scrutiny standard.
(b) No, probably. Race is a “suspect category,” and classifications based on race are therefore ordinarily subject to strict scrutiny. However, strict scrutiny will only be
applied where the court finds that there was a governmental intent to discriminate against the disfavored group. The mere fact that a law has a less favorable effect on a
racial minority than it has on the majority is not sufficient to trigger strict scrutiny. Disparate effect can be used as circumstantial evidence of an intent to discriminate,
but such evidence is not dispositive. Here, the facts tell us that an intent to disfavor African American students was not present on the part of the school board. Therefore,
strict scrutiny will not be applied, and instead the “rational relation” standard will be used. Under this standard, the classification will be upheld so long as it is rationally
related to the achievement of a legitimate state objective. Here, cutting an expensive program is certainly rationally related to saving money for the school system, and
saving money is certainly a legitimate state of objective in a time of economic hardship.

Smithville is a small town located near the Canadian border. A significant minority of the children attending Smithville High School are American citizens of French-
Canadian descent. The school board has just canceled the honors French program. The decision was made by the school principal, an American of Anglo-Saxon descent,
who told school board members, “If we keep running an extensive French program, we’ll just attract more Americans of French-Canadian ancestry to the town, and soon
they’ll be a majority.” Since the school board was completely dominated by Anglo-Saxon Americans, they concurred with the decision. Jacques, an American student at the
school of French-Canadian descent, has sued the school district, arguing that the cancellation has violated his constitutional rights.
(a) What standard of review should the court use to judge the school district’s cancellation decision? _________________
(b) Will Jacques’ challenge succeed? _________________
(a) Strict scrutiny. Classifications based on national origin are suspect. Similarly, classifications based on ethnic ancestry are suspect. So whether the French-Canadians
are being discriminated against because they or their ancestors came from another country, or because they are part of a different ethnic group that speaks a different
language, they are clearly a suspect class. Since all the evidence indicates that the decision was made for the purpose of disadvantaging members of this group (i.e., the
disadvantage was not merely an inadvertent effect of action taken for some other reason), the requisites for strict scrutiny have been satisfied.
(b) Yes. Once strict scrutiny is applied, the challenged classification will only be upheld if it is necessary to achieve a compelling governmental objective. Preventing a
particular group of citizens from becoming a majority in a town or school system is certainly not a compelling state objective (and probably not even a “legitimate”
objective). Therefore, even if the challenged classification is a “necessary” means of achieving that objective, the classification must fall.

Strict scrutiny and affirmative action

When the government creates a classification based on race, what are the two compelling interests that may allow the classification to survive strict scrutiny?
1. Remedying past discrimination; and
2. Enhancing diversity.
CH §9.3.5.2; E Ch.10-IV(A)(1), (G)(4)(d).

Are affirmative action programs subject to strict scrutiny?


Yes, regardless of whether the program is a federal program or a state/local program. See Adarand Constructors, Inc. v. Pena, 515 U.S. 200 (1995) (holding that
congressionally authorized programs must satisfy strict scrutiny). E Ch.10-IV(H).
Even though such programs are strictly scrutinized, they may be upheld if they meet the following requirements:
1. They must be narrowly tailored (i.e., not overbroad; they should be based on pertinent racial percentages in the relevant population);
2. They must not involve a rigid racial quota, but rather use race as a "plus"; and
3. There must be an adequate showing of specific past discrimination by the entity doing the affirmative action (e.g., city) in question; a mere history of national
racism or a statistical demonstration won't suffice); or they must enhance diversity in a way that a colorblind program would not. Richmond v. J. A. Croson, 488
U.S. 469 (1989); E Ch.10-IV(G).

NOTE:
Where state and local governments are concerned, remedying an entity's own past improper discrimination is a compelling interest; remedying the general effects of
society's discrimination is not a compelling interest.

RELATED ISSUE:
Enhancing diversity is a compelling state interest that allows race to be considered as a factor in college and university admissions programs. Strict racial quotas are
impermissible, University of California Regents v. Bakke, 438 U.S. 265 (1978), as are mechanical, non-"holistic" approaches resembling quotas, Grutter v. Bollinger, 539 U.S.
306 (2003) and Gratz v. Bollinger, 539 U.S. 244 (2003); E Ch.10-IV(F).
The planet Centauri Prime has a population that is 90% Centauri and 10% human, with a history of discrimination against humans. The Bab 5 school board on Centauri
Prime has a policy of laying off Centauri teachers with greater seniority than human teachers if doing so is necessary to maintain racial balance among the faculty. Londo
Molari, a Centauri teacher, is laid off before John Sheridan, a human teacher with less seniority. Londo challenges the policy on equal protection grounds. Centauri Prime
has a constitution and history of cases identical to ours. What result?
The layoff policy violates equal protection, because it involves a rigid racial quota.
The classification here is based on race-layoffs are determined by the teacher's race. Race is a suspect classification, so even though the policy is a form of "affirmative
action," it must satisfy strict scrutiny–it must be necessary to promote a compelling interest. While promoting racial diversity in public schools is probably a compelling
interest, quota systems like the one here are not considered necessary to promote that goal. (Of course, race could be a factor in determining who gets laid off; it just cannot
be the determining factor.) Wygant v. Jackson Bd. of Education, 476 U.S. 267 (1986); E Ch.10-IV(I)(3)(b); CH pp. 744-745.

RELATED ISSUE:
Say it was the federal government that created the layoff policy, not a local government. Under Adarand Construction, Inc. v. Pena,515 U.S. 200 (1995), strict scrutiny would
still have to be used, and the Court would probably decide that this was still a rigid quota system that wasn't a necessary means of combating the effects of past
discrimination.

The Encounters Construction Co., a Caucasian-owned construction company, submits the lowest bid for a subcontract to construct telescopes for a UFO-watching tower in
rural Nevada. However, the general contractor accepts a bid from E.T. Ltd., a minority-owned firm qualifying under congressionally authorized regulations as a
Disadvantaged Business Enterprise. The prime contractor awards the contract to E.T., and for doing so receives a financial incentive from the federal government.
Encounters Construction brings suit. What type of scrutiny will this federal regulation have to meet?

Strict scrutiny. Congressionally authorized, race-conscious, affirmative action programs are subject to strict scrutiny, just as such programs are scrutinized when enacted
by state or local governments. The regulation at issue here can be upheld only if it was necessary to achieve a compelling state interest. Adarand Construction, Inc. v.
Pena, 515 U.S. 200 (1995). The court must consider: (1) whether the governmental interest being served is compelling; (2) whether the racial classification is narrowly
tailored to serve that interest (i.e., whether race-neutral means might have been effective to achieve the interest); and (3) whether the remedy is acceptably short-lived
(i.e., whether it avoids lasting longer than the discriminatory effects it is designed to eliminate). E Ch.10-IV(H). Here, the provision will probably be struck down, because
it's rare that direct government financial incentives rendered on a race-conscious basis will be found to be a "necessary" means of wiping out past discrimination.
The State of Mission Law School accepts 350 of its 3,500 applicants. One of its goals in admitting those students is to build a student body of diverse backgrounds and
interests. Mission Law School has a particular commitment to racial diversity, and works to include students from groups that have faced historical discrimination, namely
African Americans, Hispanics, and Native Americans. The school's goal is to have a critical mass of unrepresented minority students. To promote this goal of diversity, the
Law School's admissions staff put together a system in which any applicant is admitted if he or she receives 100 points. The staff adds an extra 20 points to the files of all
non-white minority applicants. Bobbie Bratz, a white student with a higher grade point average and LSAT score than some of the accepted minority students, challenges
the program as a violation of equal protection. Will her challenge succeed?
Yes. The Law School program does not survive strict scrutiny. All racial classifications are subject to strict scrutiny, which means that they are constitutional only if they
are narrowly tailored to further compelling governmental interests. In Grutter v. Bollinger, 539 U.S. 306 (2003), the Court recognized diversity as a compelling government
interest that could justify the use of racial classifications. In Gratz v. Bollinger, 539 U.S. 244 (2003), however, an undergraduate program similar to the hypothetical because
it awarded 20 points for racial classifications was invalidated because there was no individualized selection process and no meaningful individualized review.
The State of Mission Law School accepts 350 of its 3,500 applicants. One of its goals in admitting those students is to build a student body of diverse backgrounds and
interests. Mission Law School has a particular commitment to racial diversity, and works to include students from groups who have faced historical discrimination, namely
African Americans, Hispanics, and Native Americans. The school's goal is to have a critical mass of unrepresented minority students. To promote this goal of diversity, the
Law School's admissions staff reviews individual files carefully with diversity in mind. In this review, while using race as a "plus factor," the admissions staff also considers
other possible bases for diversity, such as geographical location and life experience. Bobbie Rudder, a white student with a higher grade point average and LSAT score
than some of the accepted minority students, challenges the program as a violation of equal protection. Will her challenge succeed?
No. The Law School program survives strict scrutiny. All racial classifications are subject to strict scrutiny, which means that they are constitutional only if they are
narrowly tailored to further compelling governmental interests. In Grutter v. Bollinger, 539 U.S. 306 (2003), the Court recognized diversity as a compelling government
interest, in the educational setting, that could justify the use of racial classifications. The Court then ruled that the University of Michigan Law School's program, similar to
the hypothetical here, was narrowly tailored because the program did not establish a quota, and used a flexible file review that considered each applicant as an individual.
Public School District has no history of racial discrimination. The student population is 41% Caucasian and 59% non-Caucasian. The district allows incoming ninth graders
to list their preferences for what school they want to attend. Public School District's high schools are oversubscribed, however, so all students cannot receive their top
choice. If too many students list the same school as their top choice, a series of "tiebreakers" determines who can attend what school. The first tiebreaker is whether the
student has a sibling at that school. The second tiebreaker depends on the racial composition of the school and the race of the student. If an oversubscribed school is not
within ten points of the district's Caucasian and non-Caucasian population, the tiebreaker selects students of the race that will reset the racial balance in the school. Because
Public School 101 is 52% Caucasian, Wanda White lost the tiebreaker to an African American student who gained admission to Public School 101. Is the tiebreaker
constitutional?
No. In Parents Involved in Community Schools v. Seattle School District, 127 S. Ct. 2738 (2007), in the Roberts Court's first consideration of affirmative action, the Court
invalidated a similar program in Seattle by a 5-4 vote. In a plurality opinion by Chief Justice Roberts, four members of the Court concluded that any use of race in the
context of school admissions is unconstitutional. The plurality found that the school district had no compelling interest for the program. First, there was no history of past
discrimination. Second, the plurality wrote that the government's compelling interest is in the broad diversity of the student body, not the racial diversity targeted by
Seattle.
A concurrence by Justice Kennedy stated that the government may still consider race to achieve diversity and avoid resegregation. Kennedy rejected the Seattle plan
because he believed it engaged in "individual typing by race"; instead, Seattle could have conducted a more individualized review of the applications or used race-neutral
means to achieve its goals.
The dissenters argued that the plurality had misinterpreted Brown and Grutter. E Ch.10-IV(F).

What race-conscious measures does Justice Kennedy's concurring opinion in the Parents Involved cases permit for public school districts that seek to avoid schools
segregated by race?
According to Justice Kennedy, a school district may use the following race-conscious methods:
1. Strategic site selection of new schools, based on awareness of neighborhood demographics;
2. Drawing attendance zones with awareness of neighborhood demographics;
3. Allocating resources for special programs;
4. Recruiting students and faculty in a targeted fashion; and
5. Tracking enrollments, performance, and other statistics by race.

MNEMONIC:
DARTS (Drawing attendance zones; Allocating resources; Recruiting students; Tracking enrollments; strategic Site selection).

N.B.:
According to Justice Kennedy, the government may not "treat each student in different fashion solely on the basis of a systematic, individual typing by race." Parents
Involved in Community Schools v. Seattle School District, 127 S. Ct. 2738 (2007); E Ch.10-IV(F)(4)(c)(i).

The faculty of Baxter State College adopted an affirmative action plan for faculty hiring after finding that racial minorities were severely underrepresented on the college
staff. Under the plan, faculty are to be hired on an alternating basis so that every other faculty member hired must be a black or Latino, until the percentages of blacks
and Latinos on the Baxter faculty reach the percentages these groups occupy in the state’s population. The faculty adopted the plan after a U.S. Justice Department study
found that the nation’s colleges and universities had a long history of racial discrimination in faculty recruitment. Mary was turned down for a job in Baxter’s history
department because she is white. She has sued the college, claiming a violation of the Equal Protection Clause. Can Baxter establish that its action was warranted by a
compelling interest in remedying past discrimination?

Answer
The college probably cannot prove that it had a compelling interest in adopting the affirmative action plan for remedial purposes. As to the first requirement, even if Baxter
was guilty of past racial discrimination in faculty hiring, it is doubtful that state law gives the college faculty, as opposed to the college’s board of trustees or other governing
body, the legal authority to adopt such a hiring plan.
The second requirement might not be met for several reasons. The fact that there are few minority faculty at Baxter does not by itself establish a constitutional violation;
proof of discriminatory intent would also be needed. In short, Baxter must make findings establishing a prima facie case that it had violated the Equal Protection
Clause before adopting the plan. Instead, the faculty seems to have relied solely on a federal study that probably did not address the specific conduct of Baxter State College.
Even if many or most of the nation’s colleges engaged in illegal discrimination, this does not establish that Baxter did so. On the other hand, if the faculty had first gathered
evidence of Baxter’s past unconstitutional discrimination in faculty hiring, the second requirement would presumably be met.
The city of Monroe enacted an ordinance providing that 10% of all procurements of office supplies in the city for the coming year must be purchased from office supply
companies majority-owned by either African Americans or Hispanics, the two largest ethnic groups in the city. African Americans and Hispanics together make up a
substantial minority (approximately 40%) of the city’s population. The City Council did not conduct legislative hearings or investigations prior to enacting the bill. However,
minutes of the Council’s deliberations show that Council members enacted the bill primarily because they felt that minority business people, especially in the office supply
industry, had had less general economic opportunity than white-owned firms, and that the 10% set-aside was the best way to increase the opportunities for minority
owned companies. At the moment the bill was enacted, firms owned by African Americans or Hispanics amounted to approximately 10% of the total office supply
companies in the city, but these companies did only about 4% of the total office supply business in the city, since the average minority-owned firm was much smaller than
the average white-owned firm.
Anglo Office Supply, a white-owned office supply firm, has challenged the Monroe set-aside on equal protection grounds.
(a) What standard should the court use in evaluating the constitutional sufficiency of the set-aside? _________________
(b) Will Anglo’s attack succeed? _________________
(a) Strict scrutiny.
(b) Yes. The facts here are fairly similar to those of Richmond v. Croson. That case makes it clear that any race-based affirmative action scheme will be strictly scrutinized,
and will thus be struck down unless it is necessary to achieve a compelling governmental objective. Here, the scheme has at least two shortcomings, each of which is
probably separately fatal. First, the city is not trying to remedy past official discrimination, or even explicitly trying to remedy past unofficial discrimination — the facts
tell us merely that there has been an inequality of “economic opportunity,” which is not the same as racial discrimination. In the absence of a legislative finding of
discrimination in the office supply industry in the city of Monroe itself, any attempt to give a racial preference would probably be struck down regardless of how that
preference was carried out. Secondly, the plan here is essentially a quota — it allocates a particular fixed percentage of city contracts for minority-owned firms. Rarely if
ever will quotas be found to be a “necessary” means of redressing even clear past discrimination — certainly a city needs to consider, and probably try, less restrictive
measures first, such as voluntary goals, an outreach program to solicit more bids from minority-owned firms, or some other means that is less hard-and-fast than an
absolute set-aside.
A statute in the state of Ames confers upon the board of education of any city in the state the right to exercise control of assignments of students to public schools in the
city. Bigtown, Ames’ largest city, exhibits a large degree of racial segregation in housing. Years ago, a federal court found that this racially-segregated housing pattern came
about solely through private housing choices, and was not significantly encouraged by any intentional governmental action. Last year, the popularly-elected Board of
Education of Bigtown instituted a system under which elementary school students were assigned to (and bused to) schools that were not necessarily near the student’s
home, for the express purpose of reducing racial imbalances in the city’s elementary schools. (Assume for purposes of this question that the assignment and busing plan
did not violate any student’s constitutional rights.) In response to the widely-publicized Bigtown plan, anti-busing forces in Ames convinced the state’s voters to approve
a voter initiative that amended the state Constitution so as to forbid any city Board of Education in the state from enacting a mandatory busing plan. (Assume that this
initiative was a valid means of amending the state constitution.)
Some minority children in Bigtown have sued the state, contending that the initiative is an intentional race-based classification, one that modifies the state’s political
processes so that local governments’ use of busing to overcome racial isolation are made much more difficult to enact than government plans to use busing for all non-
racially-oriented purposes. Therefore, the plaintiffs say, the initiative must be strictly scrutinized, and struck down as a violation of their right to equal protection, unless
shown to be necessary to achieve a compelling governmental purpose. Are the plaintiffs correct about the level of review to be given to the initiative? _________________

No. The 2014 decision in Schuette v. BAMN establishes that voters are generally free to repeal a race-conscious affirmative action plan , and to change the political
process in a way that intentionally makes it harder for a comparable race-conscious affirmative action plan to be instituted in the future. Schuette itself upheld a voter
initiative to change a state’s constitution so as to forbid a particular type of race-conscious affirmative action (in that case, a preference for admission to state
universities), even though the initiative had the purpose and effect of making race-conscious government action much harder to institute than other sorts of comparable
government action. (For instance, Schuette allowed the state to say, in effect, that residents lobbying for a race-conscious admissions preference must get the state
constitution amended, whereas residents lobbying for a non-race-conscious admissions preference in favor of, say, alumni, merely have to persuade the university
officials to grant it.) Schuette will control the outcome here: since the facts tell us that the problem in question (racial isolation in schools) came about through private
housing decisions rather than by any government action, the voters are free to roll back government’s attempt to use race-conscious means to deal with the problem,
even though those race-conscious means (pupil assignments and busing) would not themselves have violated anyone’s constitutional rights.

Intermediate scrutiny

What standard must a statute meet to withstand intermediate scrutiny?


The statute must be substantially related to an important governmental interest.
Intermediate scrutiny is employed where a classification system involves a "quasi-suspect" class, like gender or illegitimacy, and the government intentionally
discriminates on the basis of that quasi-suspect class (discriminatory impact alone is insufficient). E Ch.10-I(B)(8)(c).

NOTE:
"Substantially related," unlike the "necessary" requirement of strict scrutiny, does not require that the least discriminatory alternative be chosen by the government.

NOTE:
In the case of gender-based classifications (but not yet illegitimacy), the Court has added another formulation-the gender classification must be supported by "exceedingly
persuasive justification." U.S. v. Virginia, 518 U.S. 515 (1996). This seems to require that both the purpose and the means-purpose link be very persuasive, and seems to
make the intermediate standard a bit tougher to satisfy than previously. For instance, only "actual state purposes," not hypothetical purposes that are not shown to have
motivated the state, can count. Id.

NOTE:
Some classifications involving illegal aliens may also be subject to intermediate scrutiny. The only case indicating this is Plyler v. Doe, 457 U.S. 202 (1982), which gave
intermediate scrutiny to rules depriving illegal aliens' children of public education, but did not address the review to be given to classifications involving adult aliens. So
it's not clear whether other classifications involving illegal aliens would also be subject to intermediate scrutiny.

In the context of classifications based on gender, what governmental interests are considered "important"?
When governments defend their gender-based classifications against equal protection attack, most of the interests they assert have been found to be "important."
Two common governmental interests that are found to be "important" are (1) remedying past discrimination against women, and (2) preventing illegitimate teen
pregnancies.

NOTE:
Two governmental interests that do not meet the importance requirement are (1) administrative convenience, and (2) providing women a choice of coed educational
environments while denying that option to men. E Ch.10-V(D)(1).

Could a state statute discriminating against men violate equal protection?


Yes. As with statutes discriminating against women, statutes discriminating against men will be found to violate equal protection unless the law substantially relates to an
important government objective (the "intermediate scrutiny" test). However, keep in mind that the state has an important interest in compensating women for past
discrimination and in preventing illegitimate teen pregnancies. E Ch.10-V(A)(2); CH p. 762.

EXAMPLES:
A state law provides that only husbands and not wives may be ordered to pay alimony. The law is invalid, because the aim is to help all spouses in need–thus, it should
apply to men and women alike. Orr v. Orr, 440 U.S. 268 (1979). However, the state may define statutory rape so that only men may commit it, as there is an important state
interest in preventing teen pregnancies. Michael M. v. Superior Court, 450 U.S. 464 (1981).

The State of Military prides itself on its long military tradition. The state has for many years maintained one (and only one) state-sponsored military academy, the Military
Military Academy (MMA), and it has always limited that academy to men. The MMA has a program emphasizing extreme physical rigor and extensive hazing, and the state
believes that this type of program would not appeal to the vast majority of women. Is the academy's single-sex limitation constitutional?
No. On these facts, the Court in U.S. v. Virginia, 518 U.S. 515 (1996), held that restricting attendance at Virginia Military Institute violated the equal protection rights of
women. The Court announced a new standard for gender-based classifications-they must be supported by "exceedingly persuasive justification," and will be subjected to
"skeptical scrutiny." The majority said that as long as there are some women who would be interested in, and qualified for, this type of rigorous and hazing-based program,
the fact that most women aren't is irrelevant.

NOTE:
The Court also expressed its skepticism about any gender-based classifications that are based on "overbroad generalizations" or stereotypes about men and women. So
watch out for stereotypes; they should not be able to survive intermediate scrutiny. E Ch.10-V(E).

NOTE:
As the result of U.S. v. Virginia, one big change is that in gender-based cases, only objectives that are shown to have actually (not just hypothetically) motivated the
government may be considered as satisfying the "important governmental objective" requirement.

A state law provides that veterans will be given preference in public employment opportunities. This results in far greater public employment for men than for women.
Will the law be upheld when challenged on equal protection grounds?
Yes. The discrimination here is de facto–the law was not intended to discriminate against women, it simply has a disproportionate impact on women. Because the
discrimination is not intentional, the law will not receive "intermediate" scrutiny (requiring that the law be substantially related to important government interests), as is
usual with gender discrimination. Instead, it will only have to meet the rational relation test, and need only be rationally related to a legitimate state interest. Under this
test, the law will be upheld. Personnel Administrator v. Feeney, 442 U.S. 256 (1979); E Ch.10-V(G).
ANALYZING CLASSIFICATIONS BASED ON ILLEGITIMACY
Illegitimacy, like gender, is a "quasi-suspect" class, subjecting it to intermediate scrutiny.
One important point to look for is the purpose behind a statute tied to illegitimacy; thus, courts won't uphold discriminatory laws intended to punish illegitimate children.
For instance, a state law prohibiting illegitimate children from pressing wrongful death actions concerning their parents' death, while allowing wrongful death claims by
legitimate children, would be invalid. Levy v. Louisiana, 391 U.S. 68 (1968); E Ch.10-VII(B); CH pp. 778-779

A State of Bonneville statute prohibits illegitimate children from maintaining wrongful death actions for the deaths of their parents, but allows legitimate children to
maintain such actions. Captain Hook's illegitimate son, Fish, wants to file a wrongful death action against Al Ligator for eating his father, but the statute prevents him from
doing so. Fish challenges the statute on equal protection grounds. What result?
The statute will be struck down. Equal protection problems arise when the state creates a discriminatory classification. When the classification is based on illegitimacy, as
here, the statute is subject to intermediate scrutiny–the law must be substantially related to an important government goal. Here, the state's interest is discouraging people
from having illegitimate children. The problem is that the statute punishes the illegitimate children themselves, who cannot control their status. Statutes like this are struck
down because of this unfair punishment. Levy v. Louisiana, 391 U.S. 68 (1968); E Ch.10-VII(B); CH pp. 778-7
Joe was denied admission to the State School of Nursing under a law providing that only women may attend the school. Joe challenged the state’s refusal to admit him as
violating the Equal Protection Clause. The state defended its admissions policy on the ground that it serves the important objective of remedying past societal
discrimination against women in the workplace by offering professional training that will assure women a job. Will the court accept the state’s argument?
Remedying past societal discrimination against women is an objective that may justify a gender classification. However, for this compensatory goal to suffice, several
requirements must be met, including proof that this was the actual objective of the law. In the absence of any evidence to this effect in the state statute or its legislative
history, a court would be skeptical of the state’s benign goal. This skepticism would be compounded if Joe shows that the nursing school is part of a state university that
has been limited to women since it was created a century ago. Moreover, the fact that nursing is one of the few professional fields from which women have not been
excluded would cast further doubt on the state’s alleged remedial goal. While women may need special assistance in gaining access to fields such as law, medicine, or
engineering, there is no similar need in the field of nursing. It is thus doubtful a court would accept the state’s proffered remedial goal as being the real objective of its
discriminatory admissions plan. See Mississippi Univ. for Women v. Hogan, 458 U.S. 718 (1982).

A state law makes it a crime for a male to have sexual intercourse with a female who is under the age of 18 unless the couple is married. Only the male partner is
criminally punishable for such conduct. The state defends this gender classification on the ground that it is substantially related to the important goal of preventing
illegitimate teenage pregnancies. According to the state, female minors would be less likely to file a complaint if they could then be prosecuted. Assuming that the court
accepts the state’s claimed objective as being both important and genuine, what should the state have to show to meet the “substantially related” requirement?
The state should have to persuasively demonstrate that its gender-discriminatory law is more effective in achieving the state’s goal than would be a gender-neutral law
under which both partners are potentially subject to criminal prosecution. While it is no doubt true that there would be fewer complaints filed by female minors if they
were subject to punishment, the state’s goal was not to maximize the number of complaints filed, but to reduce the number of teenage pregnancies. A gender-neutral law
might have a much stronger deterrent effect than a law that threatens only males with punishment. The state should have the burden of proving that its gender-
discriminatory approach actually has a greater deterrent effect than would a gender-neutral law.
This example is taken from Michael M. v. Superior Court, 450 U.S. 464 (1981), where the Court upheld California’s statutory rape law by applying what amounted to rational
basis review. The plurality opinion by Justice Rehnquist did not require the state to prove that its gender-discriminatory law “is any more effective than a gender-neutral
law would be in deterring minor females from engaging in sexual intercourse.” Id. at 496 (Brennan, J., dissenting). Michael M. would probably be decided differently under
the standard of review applied by the Court today.
Under the Immigration and Nationality Act (INA), a child born overseas to unmarried parents, only one of whom is a U.S. citizen, will be deemed to be an American
citizen at birth only if certain statutory requirements are satisfied. These requirements make it harder for a child to qualify as a citizen if its citizen parent is the father
rather than the mother. In particular, for the child to be eligible for American citizenship, a citizen father (but not a citizen mother) must (a) agree in writing to support
the child until it reaches age 18, and (b) legitimate the child or formally acknowledge the child while it is under 18. Joseph, a United States citizen, is the father of Tuan
Ahn Nguyen, who was born in Vietnam to a woman who was not a U.S. citizen. At age six, Tuan moved to Texas, where he lived with his father until reaching majority. His
father, however, never took any action to legitimate Tuan or to formally acknowledge paternity. Tuan, who is now 28, has been ordered deported to Vietnam by the
Immigration and Naturalization Service based on his conviction of two crimes of moral turpitude. He claims that he is not deportable because he is a citizen of the U.S. by
virtue of Joseph’s citizenship. He further claims that to the extent that the above provisions of the INA impose additional burdens on those born to U.S. citizen fathers as
opposed to those born to U.S. citizen mothers, the INA violates the equal protection guarantee of the Fifth Amendment. The government defends these provisions as
discouraging fraudulent claims by those claiming to be children of U.S. citizen fathers, and as helping to ensure that citizen fathers become aware of their foreign-born
children at an early age, thereby fostering the parent-child relationship and the child’s ties to the United States. Are the INA provisions invalid as constituting forbidden
gender discrimination?

These provisions facially discriminate on the basis of gender, for they make it more difficult for citizen fathers to transmit American citizenship to their children than for
citizen mothers to do so. The government may argue that intermediate scrutiny should not apply because the Court traditionally shows deference to the political branches
in cases involving immigration or naturalization. However, this case involves the acquisition of citizenship at birth and, thus, does not concern either immigration (the
terms on which aliens may be admitted to or remain in the United States) or naturalization (the process by which persons may obtain citizenship after birth). Thus, the
mid-level standard of review should not be relaxed. Nevertheless, a federal court might have some reluctance to grant the remedy requested by Tuan, given that he seeks
what is in essence a judicial conferral of citizenship that would conflict with the judgment of the Immigration and Naturalization Service. Hence, while mid-level scrutiny
surely applies to this gender discrimination, the actual standards might not be applied with the same intensity as one would expect in other contexts.
Let’s begin with a “strict” application of mid-level scrutiny. Under that standard, the government must show that these gender-based classifications are substantially
related to an important governmental interest—i.e., that a gender-neutral law would not achieve its goals as well as a gender discriminatory law. As to the goal of
discouraging fraudulent claims, because another section of the INA already requires proof of a blood relationship by “clear and convincing evidence,” the support and
acknowledgment provisions are entirely gratuitous and unnecessary in terms of this goal. As to the goal of encouraging fathers to develop a caring and supportive
relationship with their child, the statute rests on the stereotype that fathers are less likely than mothers to care for children born out of wedlock. Even if this generalization
may be true in most cases, it is not true for all fathers or all mothers. The government’s goal would be equally well served by a gender-neutral law requiring that during
the child’s early years, a citizen parent (whether father or mother) have maintained at least minimal contact with the child, established a relationship with the child,
supported the child, or been the child’s caretaker. Thus, the INA probably violates the Fifth Amendment by imposing support and acknowledgment requirements solely
on citizen fathers. This was the position taken by the four-person dissent in Tuan Ahn Nguyen v. Immigration and Naturalization Service, 533 U.S. 53, 74 (2001) (O’Connor,
J., dissenting). It was, however, a dissent.
The majority in Tuan Ahn Nguyen, although invoking the language of mid-level scrutiny, employed a somewhat less strict version of the test, perhaps partly influenced by
their articulated concern with what might be called a judicial conferral of citizenship and with the perceived need for deference to the INS in the overall context of
immigration and naturalization. More generally, the majority saw the “discriminatory” burdens imposed by the INA as “minimal” and as being sufficiently related to the
government’s interests in preventing fraud and promoting a familial relationship between a citizen father and his offspring. The majority dismissed the availability of
nondiscriminatory alternatives—e.g., such neutral alternatives as DNA testing—as being “hollow,” given that the mother was present at birth, hence establishing her
maternity beyond doubt. In short, given the clarity of a citizen mother’s maternity, the citizen father could be treated differently, and Congress, in devising that different
treatment, was not required to act with scientific precision. See 533 U.S. at 71-73. Perhaps the best way to read the majority opinion in Tuan Ahn Nguyen is as an example
of what sometimes occurs when a finely tuned doctrine collides head-on with policy concerns that lead some Justices to see a need for a less finely tuned approach. It is
doubtful that the decision will lower the general threshold of mid-level scrutiny in gender discrimination cases.

The prosecution in a paternity case used all of its peremptory challenges to excuse from the jury ten men who had previously been defendants in paternity actions. Does
the prosecutor’s action violate the Equal Protection Clause?
In J.E.B. v. Alabama, 511 U.S. 127 (1994), the state in a paternity suit used all but one of its peremptory challenges to remove men from the jury out of fear that male jurors
would sympathize with the defendant. The Court held that such use of gender as a proxy for bias was unconstitutional; instead of employing “stereotypical and pejorative
notions about a particular gender,” id. at 143, the state was required to “look beyond the surface” to discover possible bias based on gender-neutral factors. Id. at 139 n.11.
However, the Court in J.E.B. noted that “strikes based on characteristics that are disproportionately associated with one gender could be appropriate, absent a showing of
pretext.” Id. at 143 & n.16.
In our example, the prosecutor has not employed a stereotype about men in general or presumed that men will be biased toward the defendant. Instead, the prosecutor
excused only those potential jurors who had been sued for paternity. While this group by definition consists only of men, it was not a juror’s sex that led to exclusion, but
rather the fact that he had been sued for paternity. Such conduct by the prosecutor does not constitute gender discrimination if the jurors were excused in spite of, not
because of, their sex. See §6.2.2. In other words, if the prosecutor had a good-faith concern about the jurors’ prior involvement in paternity cases rather than about their
being males, this would rebut any prima facie case of gender discrimination by defeating the purpose element. To be sure, the prosecutor relied on the stereotype that
those sued for paternity will be biased toward the defendant in a paternity suit; but since this presumption rests on a trait or characteristic—prior involvement in a
paternity suit—that does not call for heightened scrutiny, reliance on the presumption is constitutionally unobjectionable. The prosecutor’s action would, therefore, not
violate the Equal Protection Clause because it probably does not involve gender discrimination.

State law provides that a father need not pay support on behalf of an illegitimate child unless paternity is established in an action filed prior to the child’s fifth birthday; if
paternity is so established, the father’s support obligation shall not exceed 50 percent of the amount that would be owed to a legitimate child. Is this statute
constitutional?
Insofar as the statute limits the support that must be paid on behalf of an illegitimate child to 50 percent of what would be paid to a legitimate child, the statute is surely
unconstitutional. It is difficult to imagine any reason for the 50 percent limitation other than a desire to punish the mother and her child for the child’s having been born
out of wedlock.
The five-year statute of limitations for filing a paternity action is also problematic. The state may argue that this rule bars stale or fraudulent paternity claims brought at a
time when the putative father can no longer defend himself. The Court has recognized the importance of this state interest, but has held that to meet the “substantially
related” test, the state must allow a reasonable time for those interested in the well-being of such children to sue on their behalf. Moreover, the time limit may be shorter
than necessary to protect the state’s interest. Given the fact that scientific evidence is now often able to prove and disprove claims of paternity, the harm to the putative
father of allowing suit after the child’s fifth birthday is likely to be minimal. Thus, the five-year limitations period is probably also unconstitutional. See Clark v. Jeter, 486
U.S. 456 (1988) (invalidating six-year limitation on paternity suits as condition for receiving child support).
The Oneanta state legislature mandated that any employer with five or more employees grant a paid six-month maternity leave to any woman who gave birth while on
the company’s payroll. The legislature imposed no requirements regarding paid or unpaid family leave for men who had just had a child. In deciding to enact the measure,
the legislators relied principally on evidence that various rising social problems, such as drugs and crime, were caused in part by the rise in working women and a
consequent failure of mothers to bond with their infants. Frank, the father of a newborn infant who wished to have a paid paternity leave, has challenged the statute on
the grounds that it violates his equal protection rights.
(a) What standard should the court apply in evaluating Frank’s challenge to the maternity-leave statute? _________________
(b) Will Frank’s attack succeed? _________________

(a) Mid-level scrutiny, i.e., whether the statute is “substantially related” to the achievement of an “important” governmental objective. (U.S. v. Virginia says that
gender-based classifications will now need an “exceedingly persuasive justification,” and will be “skeptically scrutinized,” but the case seems not to officially reject mid-
level scrutiny, merely to indicate that mid-level scrutiny will now be applied in a rigorous way.)
(b) Yes, probably. Government classifications involving suspect or semi-suspect claims are evaluated by the same standard regardless of whether the purpose is to
discriminate against the traditionally disfavored class or to redress past discrimination. In other words, there is no easier standard for “affirmative action.” Thus the
Supreme Court applies the same mid-level review to all gender-based classes, whether the classification is old-fashioned discrimination against women or affirmative
action designed to improve the lot of women. In both cases, the gender-based classification will be upheld only if it is substantially related to the achievement of important
governmental objectives.
Here, encouragement of bonding between parent and newborn child is probably an “important” governmental objective. However, a court would probably conclude that
there is no “substantial relation” between a mother-only parental-leave program and achievement of this objective. It is highly likely that whatever social problems are
caused by not having either parent at home during a child’s infancy, those problems can be redressed as well or almost as well by having the father be at home. Therefore,
a scheme that entitled either parent, but not both parents, to take a paid maternity leave would accomplish the legislative purposes as well or almost as well as the mother-
only scheme. Consequently, the court will probably, thought not certainly, strike down the statute.
In general, courts are likely to strike down any statute that reflects stereotypes about the “proper place of women.” See, e.g., U.S. v. Virginia, finding Virginia’s belief that
VMI’s intense military training program is unsuitable for women to be an unconstitutional generalization about “the way women are.” The scheme here — which implies
that a mother’s place, but not a father’s place, is to be home with the newborn infant — reflects similar stereotypical thinking.

Substantive due process analysis of fundamental rights


Understand the substantive due process analysis of fundamental rights.
When does a substantive due process problem arise?
When a governmental action intrudes on a right. If the right is both personal and "fundamental," the statute must meet the "strict scrutiny" test. If it's not both personal
and fundamental, but rather, economic or social, it need only meet the "rational relation" test of validity. E Ch.9-III(C); CH pp. 792-794.
ANALYZING SUBSTANTIVE DUE PROCESS PROBLEMS
When legislation impairs a right, ask: Is the right a "fundamental personal" right? If yes, go to #1; if no, go to #2.
1. Yes, the right is a "fundamental personal" right. These are the First Amendment rights (e.g., speech, press, religion, assembly, petition), interstate travel,
voting, privacy (e.g., marriage, contraception, procreation, raising children, family interest), death, and fairness in the criminal process (e.g., right to counsel
on first appeal).
Here, the legislation must meet the "compelling state interest" test–it must be necessary to promote a compelling governmental interest.
2. No, the right is not a "fundamental personal" right. (Instead, the statute is merely regulating social or economic interests of lesser importance.)
Here, the statute is only subject to the "rational relation" test–if there is a set of facts imaginable that would make the law a reasonable means to achieve a
legitimate governmental purpose, the law is valid.
Such laws include public health and safety measures and all kinds of business regulations, including trade practices, wage and hour regulations, price controls,
and bans on discrimination against union (or non-union) personnel. E Ch.9-III(C), 9-IV.

Name the rights that are "fundamental" for substantive due process purposes.
1. The Right to Marry;
2. The Right to Custody of One's Children;
3. The Right to Keep the Family Together;
4. The Right to Control Upbringing of Children;
5. The Right to Procreate;
6. The Right to Purchase and Use Contraception; and
7. A Protected Liberty Interest for Competent Adults to Refuse Unwanted Medical Care.
E Ch.9-III(C), 9-IV, Ch.10, Ch.14; CNR §11.6; CH §§10.2-10.5

RELATED ISSUE:
Although abortion was recognized as a fundamental right in Roe v. Wade, the government is now held to the undue burden standard of Casey; a "finding of an undue burden
standard is a shorthand for the conclusion that a state regulation has the purpose or effect of placing a substantial obstacle in the path of a woman seeking an abortion of
a nonviable fetus." CH pp. 829-830.

N.B.:
The Court has ruled that the right to privacy protects private consensual adult sexual activity, including homosexuality, without identifying this privacy as a fundamental
right. The Court invalidated a homosexual sodomy statute in Lawrence v. Texas, 539 U.S. 558 (2003), using rational basis review. CH p. 846.

NOTE:
Some textbooks treat the punitive damages cases and the Caperton decision about a judge's refusal to recuse from a case involving a campaign contributor as a new type
of economic substantive due process.
In recent years, in the state of North Rockland, there has been an increase in the number of marginally-funded, educationally-inadequate private schools, as well as a rise
in the number of parents who have been teaching their children at home rather than sending them to the public schools. The North Rockland legislature has, therefore,
just passed a statute providing that every child between the age of 6 and 16 must be educated in the state’s public schools. Parents who do not comply face criminal
punishment. The statute does not allow any exceptions for even educationally-sound private schools or educationally-sound home instruction. Paula, the parent of a seven-
year-old son, wishes to educate him at home. Paula was until last year a first-grade teacher in a state public school, and all concerned agree that she is well qualified to
teach her child at home provided that she does so full time, which she expects to do. Paula has sued to overturn the statute, as applied to her and her son, on substantive
due process grounds.
(a) What standard of review should the court use in deciding Pau-la’s suit? _________________
(b) Will the statute be found constitutional as applied to Paula and her son? _________________
(a) Strict scrutiny, probably. That is, the statute will probably be struck down unless shown to be necessary to achieve a compelling state interest. The question is really
whether the right to make core decisions about how one’s children shall be educated is a “fundamental” right. The Supreme Court has not addressed this issue directly in
recent years. However, several cases (discussed further in part (b) below) suggest that the Court would hold that the right to direct the education of one’s children is
indeed “fundamental,” in which case strict scrutiny must be applied to any governmental regulation that substantially impairs that right.
(b) No, probably. In Pierce v. Society of Sisters (a 1925 case), the Court struck down a state statute requiring children to attend public schools. The decision seems to have
been on substantive-due-process-like grounds, and seems to have applied essentially strict scrutiny. Similarly, in more recent years, the Court has held that parents have
a fundamental right to decide who may visit the child; Troxel v. Granville.
Assuming that the right to choose how one’s children are to be educated is in some sense fundamental, it is very unlikely that the statute here could survive strict scrutiny.
Ensuring a good education for children is certainly an important state objective, and probably a “compelling” one. However, it is highly unlikely that foreclosing all options
other than public schools is a “necessary” means of attaining that objective. For instance, allowing a program of home instruction by one who is clearly a qualified
elementary school teacher certainly seems to be an adequate way to assure a good education. And the state could adopt a monitoring program to make sure that home
study programs satisfy a minimum quality standard. So the statute will probably flunk strict scrutiny, if strict scrutiny is applied.
The town of Tinsel originally did not have any written regulation concerning the beard or hair styles of fire fighters. At the time Jordan was hired onto the fire fighting
force, he had a beard and hair that was neatly combed but of shoulder length. After Jordan had been on the force two years, Tinsel enacted a regulation providing that no
male firefighter could wear a beard or hair extending below his neck. Jordan has challenged this regulation as violating his substantive due process rights. Tinsel has
defended the regulation on the grounds that: (i) uniformity of hairstyle is necessary to generate a feeling of esprit de corps among firefighters; and (ii) facial hair and long
hair are more likely to catch fire even if the person dons the usual safety equipment.
(a) What standard should the court use in deciding whether Jordan’s substantive due process rights have been violated? _________________
(b) Is the regulation constitutional? _________________
(a) The “rational relation” standard, not strict scrutiny. The choice of standard depends on whether the right of a firefighter to wear a beard, etc., is found to be
“fundamental.” In a case almost completely on point, Kelley v. Johnson, the Supreme Court held that a policeman’s right to wear his hair as he wished was not “fundamental,”
and that the hair-length regulation there should be judged on a rational relation standard.
(b) Yes, probably. Assuming that the rational relation standard is used, the regulation here almost certainly passes muster. The town certainly has a legitimate interest
in preserving esprit de corps and promoting safety. The contribution of short hair to fulfillment of these objectives may be questionable, but it is certainly “rational.” (For
instance, the town could reasonably have believed that long-haired male firefighters would not fit in as well.) Certainly the comparable hair-length regulation in Kelley was
found to be rationally related to a legitimate governmental objective, and was thus upheld, so the same result is likely here.

Equal protection review of fundamental rights

Ames City passed an ordinance prohibiting non-property-owning taxpayers from voting in favor of or against the issuance of municipal bonds out of the general revenue
to fund the building, maintenance, or expansion of any public utility. Tom, a resident who rents property in Ames City, sued in federal district court, claiming that this
was a violation of the Equal Protection Clause. A reviewing court would likely:
invalidate the ordinance, because the exclusion of non-property owners from voting is not necessary to any compelling governmental interest.
The facts in this question are similar to those in City of Phoenix v. Kolodziejski, 399 U.S. 204 (1970) (invalidating law permitting only property-owning taxpayers to vote
in elections regarding the issuance of certain bonds), and Cipriano v. Houma, 395 U.S. 701 (1969) (same). But even without knowing how those cases came out, you could
have narrowed down your choices. First, A is out because, as we’ve seen, even though voting is not explicitly guaranteed, the Court has recognized it as a fundamental
right and applies strict scrutiny to state attempts to deny it to groups of people. Because strict scrutiny is the standard of review, B is not the correct answer; it employs a
rational basis standard. D, on the other hand, goes too far — states can restrict the franchise to residents, for example. That leaves C, the correct answer: Even assuming
that restricting the franchise to those most affected by the outcomes of votes, in the words of Kramer, property owners and non–property owners alike are affected by
decisions to issue bonds to expand or repair utilities; the burdens and benefits are not restricted to property owners. Therefore, restricting the franchise to that group is
not necessary to achieve that compelling interest.

Ames City just passed an ordinance requiring that one show proof of real property ownership to vote on property tax increases or decreases. Peter, who owns no
property in Ames City, sues, claiming that the plan violates the Equal Protection Clause. A reviewing court would likely:

Invalidate the ordinance, because it is not necessary to achieve any compelling governmental interest.

It might have occurred to you to ask what the difference is between Salyer and Ball on the one hand, and Kramer on the other. Could a town limit votes on tax increases
to property owners on the theory that they are more affected? While that is one reading of Salyer and Ball, it is probably too broad a reading. The difference seems to be
that the cases in which a selective denial was upheld concerned water districts whose members were charged on the basis of water consumed. On the other hand, the
decision to issue bonds or fund schools impacts all citizens, even if it impacts some — who may pay higher taxes — more than others. Going through the answers,
then, A is not true for the reasons just given. The Court rejected those arguments in Kramer. B is incorrect as well; the denial in Kramer was limited to school board
elections. C is incorrect on the facts because non–real property owners were not denied all effective voice; they were just denied votes on the question of tax hikes. That
leaves D, the best answer: Even if there is some compelling reason to restrict the franchise to those who are most affected by a particular measure, the exclusion of non–
property owners is not necessary to achieve that, which is the result the Court reached in Kramer.

Voters in the State of Ames recently amended their Constitution, by statewide majority vote, to apportion seats in the lower house of the Ames General Assembly by
population but apportion seats in the upper house by geography. Voters who opposed the referendum sue, claiming that the new apportionment scheme violates the
Equal Protection Clause. A reviewing judge should

Invalidate the scheme, because it dilutes the votes of citizens electing members of the upper house of the Ames General Assembly.
The “one person, one vote” rule applies even if the citizens themselves decide they prefer a different basis for representation, as Colorado found out in a companion case
to Sims. Lucas v. Forty-fourth General Assembly, 377 U.S. 713 (1964). Colorado voters had approved a referendum apportioning seats in the lower house of the General
Assembly by population and seats in the upper house by geography. The Court held that a majority of state voters had no authority to override the requirements of the
Equal Protection Clause. 377 U.S. at 736-737. Even without knowing about the Lucas case, however, you could have reasoned your way through the choices to the correct
answer. For example, B states the wrong standard of review. Vote dilution is apparently a per se violation of the Constitution. Moreover, because it is a constitutional
mandate, states cannot override it, no matter how large the majority or supermajority. Therefore, C is not the right answer either. While it is true that many of the early
apportionment cases arose out of state legislatures’ refusal to reapportion following the census, the issue is not so much how the malapportionment came about, but
rather whether it exists at all. So D isn’t the best answer either. That leaves A, the correct answer. By apportioning seats in the upper house according to geography, the
voters have ensured that some parts of the state will be overrepresented while the more populous parts of the state will be underrepresented in the Ames General
Assembly. This would violate Reynolds v. Sims’s “one person, one vote” rule and is therefore unconstitutional.

Frank is convicted of murder in an Ames state court. Frank appeals his conviction to the Ames Court of Criminal Appeals, which affirms it. Convinced that his conviction
was erroneous, Frank seeks an appeal to the Ames Supreme Court. The court, however, requires a $250 filing fee to process a Petition for Leave to Appeal. Frank is
indigent and cannot afford the fee; there is no exception for indigency, however. Frank sues in federal court, claiming that the filing fee requirement violates the
Fourteenth Amendment. A reviewing court should:

Uphold the filing fee requirement, because Frank has no constitutional right to review by the state supreme court.

The filing fee is a bit of a red herring. The fee is required for the filing of a Petition for Leave to Appeal with the state supreme court. Because one is not constitutionally
entitled to a second appeal, the filing fee is valid. C is the best answer. B is incorrect because, as we have seen, wealth is not a protected class. A is also incorrect: There is
no right to more than one appeal. D is not so much incorrect as irrelevant. Sure, states have an interest in recouping the costs of judicial administration, but it does not
follow that there are no constitutional restrictions on what or whom they may charge.

Which of the following statutes is vulnerable to a constitutional challenge under the Fourteenth Amendment by an indigent plaintiff?
I. A state statute requiring a $50 fee in order to appeal an administrative eviction decision in a public housing project.
II. A state statute imposing a nonwaivable $100 fee for a marriage license.
III. A state statute imposing a $25 filing fee to file a civil suit in state court.

IV.

V. c. II only
VI. Civil litigants generally receive relief only when an independent constitutional interest is at stake. Thus, of the three choices, the fee in neither I nor III
would satisfy that test. Thus, neither A, B, nor D is correct. But in II, the state’s conditioning the ability to marry — a fundamental right — on payment of a
nonwaivable fee would seem vulnerable for the same reasons that the Court invalidated the fee for divorce in Boddie. Of the three choices, then, C is the
best.

VII. The State of Ames recently engaged in redistricting. As a result of the Ames General Assembly’s efforts, one irregularly shaped district, District 1, contains
over 30 percent of the state’s African American population, making it the only district in the state in which a majority of voters are composed of a racial
minority. (Caucasians comprise a majority in Ames.) The redistricting was challenged by African American residents of District 1, who claim that the
redistricting is unconstitutional. Which of the following facts would be least helpful to their challenge?

VIII. Evidence that most African Americans in Ames vote Democrat, and that the Republican-controlled legislature generally sought to pack Democratic voters
into districts that already tended to elect Democratic candidates.
IX. The Court has said that in racial gerrymandering cases, intent is everything. Intent, however, can be demonstrated by factors such as demographics, the odd
shape of the district, or evidence of legislative purpose in the redistricting. But these are not necessary required, according to the Court. In determining
which of the above is least helpful, then, A would be helpful to plaintiffs. Direct evidence of an intent to create districts based on the race of voters would be
useful. So would C: An irregular shape, one that cuts across geographical boundaries, could also be helpful to plaintiffs. D too would be helpful. It strongly
suggests that the legislature might have been “packing” minority voters into a single district, depriving them of the opportunity to elect candidates in other
districts. That leaves B, which would be least helpful. If the state can show that race and party affiliation correlate closely, then the plaintiffs have to prove
that the political objectives could have been obtained through other means that would have also resulted in a greater racial balance. That would, of course,
increase the burden on plaintiffs seeking to invalidate or overturn the legislature’s creation of District 1.
Johnny Appleseed roams from state to state, taking short-term jobs in orchards during his travels. He never stays in a state longer than one year; however, he currently
lives in the State of Bighorn, where he's lived for six months, and he plans to stay indefinitely. Bighorn has a one-year wait before a new resident will be entitled to certain
state benefits and services. What kinds of benefits and services may validly require this one-year residency period?
Keeping in mind that the right to interstate travel is fundamental, a one-year residency requirement would not pose an invalid burden on one seeking these services:
1. Reduced ("in-state") tuition at state universities and colleges; and
2. Divorce.
E Ch.10-X(F); CH §10.7.

RELATED ISSUE:
One year would be too long a delay to obtain these benefits:
1. Welfare;
2. Medical care; or
3. Voting (50-day residency is close to the upper constitutional limit).

The State of Minds has a state law that requires a person to be an in-state resident for at least one year to vote in state elections. The statute is attacked on equal protection
grounds. Is it valid?
No; it is an undue burden on the right to interstate travel and the right to vote.
The statute creates a classification involving the right to exercise two fundamental rights–the right to travel interstate and the right to vote. Thus, the classification must
be necessary to a compelling state interest. States have a valid interest in restricting voting to their own citizens to prevent voting fraud and promote a knowledgeable
electorate. However, the test of validity must consider less burdensome alternatives. That's the problem here; the duration of the residency requirement. One year is too
long. Dunn v. Blumsetein, 405 U.S. 330 (1972).

RELATED ISSUE:
The Supreme Court has held that a 50-day residency requirement is valid (and may be close to the upper limit). Marston v. Lewis,410 U.S. 679 (1973); E Ch.10-X(C)(3)(c)(1);
CH pp. 876-877.

The State of Bitterroot denies convicted felons the opportunity to vote. Does this violate equal protection?
No. The Fourteenth Amendment specifically allows states to disenfranchise felons. Richardson v. Ramirez, 418 U.S. 24 (1974); E Ch.10-X(C)(4)(b); CH pp. 879-880.
The State of Celebration requires that for a political party to get its candidates' names on the ballot, it must submit a nomination petition signed by at least 5% of the
number of votes cast in the last election. Otherwise, it will have to conduct a "write-in" campaign. A new political party, the Beer & Pizza Party, challenges the requirement
claiming that it unduly burdens its members' freedom of association and therefore is an equal protection violation. What result?
The requirement is valid. As a general rule, when a state action creates a classification that unduly burdens a fundamental right, the action is subject to strict scrutiny.
That's the case here; the classification penalizes the exercise of the freedom of association. Under strict scrutiny, the statute is only valid if it is necessary to promote a
compelling governmental interest. Although most statutes fail this test, this one doesn't. Preserving the integrity of the electoral process by keeping ballots from being
unmanageable or confusing is a compelling interest. Requiring a significant measure of community support to get on the ballot would therefore be valid. Jenness v.
Fortson, 403 U.S. 431 (1971); E Ch.10-X(D)(4)(d); CH pp. 903-904.

The State of Osage intends to hold an election to vote for officers of a water storage district. It intends to restrict the right to vote to landowners in the district and to weigh
the votes by acreage owned. Is this a violation of the "one person, one vote" requirement?
No; the right to vote can be limited to a select group of people when the governmental unit for which elections are being held has a limited purpose that disproportionately
affects only one group. Here, the landowners in the district receive all the benefits and bear all the burdens of the election (it's for officers of their water district, which
assures their water supply). Thus, the restriction on voting is valid. Salyer Land Co. v. Tulare Lake Basin Water Storage District, 410 U.S. 719 (1973); E Ch.10-X(C)(3)(b)(iii);
CH pp. 875-876.

COMPARE:
Requiring land ownership to vote for school board members violates equal protection, as the classification is "irrelevant" to the state's objectives. Kramer v. Union Free
School District, 395 U.S. 621 (1969); E Ch.10-X(C)(3)(b)(ii).

RELATED ISSUE:
Requiring property ownership by candidates also violates equal protection. Turner v. Fouche, 396 U.S. 346 (1970); CH p. 907.

Roger is 18 years of age and the father of a 3-year-old girl who was born out of wedlock. Two years ago Roger was ordered to pay child support in the amount of $200
per month. Because he has been unemployed, Roger has not paid child support and owes an arrearage of almost $5,000. Roger recently applied for a marriage license.
The license was denied under a state law that prohibits anyone from marrying who has not met his or her child-support obligations. Roger sued, alleging that the license
denial violates his rights under the Equal Protection Clause. How should the court rule?
The right to marry is a fundamental constitutional liberty. Loving v. Virginia, 388 U.S. 1 (1967). In this case, the state has created a classification that allows some people
to marry, while denying that right to others. The classifying trait—payment of child support—is not one that calls for heightened scrutiny. However, because the law
substantially interferes with a person’s fundamental constitutional right to marry, the law is subject to strict scrutiny.
The state might argue that its discrimination against those who fail to support their children serves the compelling interest of assuring that minor children are cared for.
Even if this is a sufficiently compelling interest to justify interference with the right to marry, the state must show that the marriage ban is narrowly tailored to that goal
in the sense that it is neither overinclusive nor underinclusive. Here, the ban is overinclusive because it applies even when preventing a marriage will in no way further
the state’s child-support goal. Whether or not Roger gets married, it is unlikely he can pay child support. The state thus gains nothing by impairing Roger’s constitutional
rights. The law, therefore, violates the Equal Protection Clause because it creates a classification that burdens the fundamental right to marry without being narrowly
tailored to the state’s interest in seeing that children are adequately supported. See Zablocki v. Redhail, 434 U.S. 374 (1978).

State X has refused to issue a marriage license to a same-sex couple on the grounds that State X law limits marriage to a union between a man and a woman. Is the State X
law unconstitutional as a matter of equal protection?
Yes. In Obergefell v. Hodges, 135 S. Ct. 2584 (2015), the Supreme Court held that the fundamental right to marry extends to same-sex couples as part of the liberty protected
by the Fourteenth Amendment. Since the discrimination here bears on the exercise of that fundamental right, it will also be subject to strict scrutiny under the Equal
Protection Clause. In fact, the Court in Obergefell relied on both the Due Process Clause and the Equal Protection Clause in striking down state laws virtually identical to
the State X law at issue here. In the Court’s words:It is now clear that the challenged laws burden the liberty of same-sex couples, and it must be further acknowledged that
they abridge central precepts of equality. Here the marriage laws enforced by the respondents are in essence unequal: same-sex couples are denied all the benefits afforded
to opposite-sex couples and are barred from exercising a fundamental right. Especially against a long history of disapproval of their relationships, this denial to same-sex
couples of the right to marry works a grave and continuing harm. The imposition of this disability on gays and lesbians serves to disrespect and subordinate them. And
the Equal Protection Clause, like the Due Process Clause, prohibits this unjustified infringement of the fundamental right to marry.Id. at 2604. Hence, in the absence of a
compelling justification for this discrimination, the State X law will be struck down.

The state has passed a law providing that the state attorney general shall be appointed by the governor rather than elected by the people. May state citizens successfully
challenge the statute on the ground that it violates the Fourteenth Amendment Due Process Clause by impairing their fundamental liberty interest in voting?
The suit should be dismissed. There is no such fundamental liberty under the Due Process Clause. The Fourteenth Amendment protects only the right to vote on an equal
basis with other citizens. This right is not violated when all citizens are deprived of the right to choose the attorney general.

A town of 60,000 people is run by a three-member town council. The town is divided into districts A, B, and C, each of which elects one member to the council. District A
consists of 40,000 people, district B of 15,000 people, and district C of 5,000 people. See Figure 7-1. Are some people’s votes diluted in comparison with the votes of others?
Yes. The 5,000 people who live in district C have the same representation and the same influence on the council as the 40,000 people in district A. When the people in
district C cast ballots for a council member, they each in effect have a 1/5,000th say in the outcome of the election, while the people of district A each have a 1/40,000th
say. Because the votes of those who live in district C carry eight times the weight of those who live in district A, the votes of those living in district A are diluted in
comparison with the votes of those living in district C. To avoid individual vote dilution, each of the three districts would need to contain the same number of people—
i.e., 20,000 (60,000/3 = 20,000).

The state of Pacifica enacted a statute providing that no person who is not a U.S. citizen could hold title to beach-front land located in the state. Pacifica’s west coast consists
entirely of ocean-front property. The statute was enacted in part because state residents were annoyed that Japanese nationals and citizens of other Pacific rim countries
were paying high prices for Pacifica beach-front property, making it harder for U.S. citizens living in Pacifica to compete for ownership of that property. Yukio, a Japanese
citizen who resides permanently in the United States, and who wishes to buy ocean-front property in Pacifica, has sued to have the statute overturned on the grounds that
it violates her equal protection rights.
(a) What standard of review should the court give to the Pacifica statute? _________________
(b) Will Yukio’s attack succeed? _________________

(a) Strict scrutiny.


(b) Yes, probably. Discrimination against aliens (at least aliens who are legally in this country) is to be strictly scrutinized, because aliens as a class are politically
powerless and frequently discriminated against. Therefore, any statute that discriminates against aliens on its face, or whose purpose is to disadvantage aliens, will be
struck down unless it is necessary to achieve a compelling governmental interest. Here, the state’s interest in keeping land prices low for American citizens is almost
certainly not “compelling.” In any event, discrimination against aliens is not a “necessary” way to achieve that objective, since there are other less-discriminatory options
available (e.g., price controls that apply to everybody).

Substantive due process and the fundamental right to privacy and autonomy
What is the source of the right of privacy?
The right of privacy is a "penumbral" right arising from various rights explicitly found in the Constitution, including: the First Amendment (association), Third Amendment
(home), Fourth Amendment (person and property), Fifth Amendment (privilege against self-incrimination), and Ninth Amendment (enumerated rights do not deny
existence of others not enumerated). The right of privacy is also considered a form of "liberty" protected by the Fourteenth Amendment's Due Process Clause. Griswold v.
Connecticut, 381 U.S. 479 (1965); E Ch.9-IV(C)(2); CH §10.3.2.
What types of activities are subject to the right of privacy?
Marriage, procreation and abortion, contraception, living with one's relatives, private consensual sexual activity, and the "right to die."
Some things that are not included within the right to privacy are the right of parents to send their children to discriminatory private schools, the right to government-
funded abortions (even if the abortion is medically necessary), the right of non-custodial fathers of illegitimate children to block adoption of the children, and the right to
engage in adultery. E Ch.9-IV(A)-(R).
What type of state regulation is permissible on abortions during pregnancy?
As the result of Planned Parenthood v. Casey, 505 U.S. 833 (1992), abortion is no longer a "fundamental right," and regulations on it will no longer be "strictly scrutinized."
Generally, the state may now regulate to promote important health objectives, as long as there is no undue burden on the right to abort. Some state regulation is therefore
clearly allowed.
Here's a brief summary of present law:
1. The state cannot ban all previability abortions. In fact, the state cannot even forbid all previability abortions except those necessary to save the life or health
of the mother–even where the abortion isn't needed to protect the mother, the state must allow the previability abortion (but subject to regulation of
procedures, methods, etc.).
2. The state probably can forbid all postviability abortions not necessary to protect the mother's life or health.
3. The state may impose elaborate informed consent rules (e.g., the state may require 24 hours advance consent).
4. The state may require that an unemancipated woman under age 18 obtain parental consent. (But if the state does require parental consent, it must give the
girl an opportunity to persuade a judge that an abortion is in her best interests. This is a "judicial bypass.")
5. The state cannot require that the woman's husband be given notice or a veto power over the abortion.
6. The state may forbid the use of public facilities or public staff to perform abortions. (Also, the state may refuse to pay for abortions for the indigent.)
Juliet, age 15, is horrified to discover that she's pregnant. She seeks a previability abortion; however, the state requires that both parents of a pregnant girl under
18 must be notified before she can have the abortion. Juliet definitely doesn't want her parents to find out about her pregnancy. She challenges the constitutionality
of the parental notification requirement. What result?

7. The requirement is unconstitutional, but only because it doesn't provide a "judicial bypass" means of avoiding the notification requirement. The state may
regulate previability abortions to promote important health objectives, as long as there is no undue burden on the right to abort. A parental notification
statute for pregnant women under age 18 only meets this standard if it provides for a judicial bypass. A judicial bypass must give the pregnant girl the
opportunity to prove to a judge either that she's mature enough to give informed consent (i.e., "emancipated"), or that the abortion is in her best interest.
Because the statute here doesn't do so, it's not constitutional. E Ch.9-IV(I)(2); CH §10.3.3.5.
8. The State of Straightlace prohibits the sale of condoms to adults except through state-licensed pharmacists. It also prohibits sale to those under age 18 without
the prior approval of a physician. Casanova, age 17, challenges this statute on grounds of substantive due process. What result?
9. The statute will be struck down.
10. A substantive due process problem arises when a law impairs a right. If the right is personal and fundamental, the law is subject to strict scrutiny. Here, the
law impairs the right to use contraception. Contraception is one element of the personal, fundamental right of privacy. Thus, the statute would have to be
necessary to promote a compelling governmental interest and would fail on this basis. Carey v. Population Services International, 431 U.S. 678 (1977).

11. RELATED ISSUE:


12. The statute could also be challenged on equal protection grounds, as it involves a classification–those 18 and over versus those under 18. Because the
classification involves the exercise of a fundamental right (the right to privacy), it would be subject to strict scrutiny, and it would likely fail on th at basis. E
Ch.9-IV(M); CH pp. 817-818.

13. Montgomery Burns has alienated every person he's ever met. Each of them, acting alone, decides to do away with him. Within the space of a few hours he is
poisoned, stabbed, electrocuted, shot, and run over. He falls into an irreversible coma and is kept alive only because he is artificially fed and hydrated. Burns'
wife requests that life support be discontinued on the grounds that Burns frequently told her, "If I ever become a vegetable, pull the plug." All other family
members agree that this is what Burns would have wanted, but no document (e.g., a "living will") says so. State law says that the plug can be pulled only when
there's "clear and convincing" evidence that the patient would have wanted this in the present situation. A state judge concludes that no such evidence is
present, and denies Mrs. Burns' request. She claims that the denial violates Mr. Burns' substantive due process "right to die." Is Mrs. Burns correct?
14. No. In Cruzan v. Missouri Dep't of Health, 497 U.S. 261 (1990), the Court said that (1) a competent adult has a Fourteenth Amendment liberty interest in not
being forced to undergo unwanted medical procedures, but (2) where the patient is not competent, the state may insist on clear and convincing evidence that
termination of life support is what the patient would have wanted had he known he would be in this situation.
15. In Cruzan, the oral testimony of the patient's family members that the patient would have wanted the plug pulled was found not to meet this "clear and
convincing" standard; therefore, the same result would presumably be found here. Consequently, the state has not violated Mr. Burns' "right to die," because
of the lack of sufficiently convincing proof of his wishes.
16. However, if Burns had signed a living will saying that he wanted the plug pulled if he ever became permanently comatose, the state would violate his
substantive due process "right to die," that is, to refuse medical treatment, by refusing to pull the plug. E Ch.9-IV(P).

17. The City of Miapasta has a statute forbidding families from living together with their cousins and grandchildren. Because the Supreme Court takes a liberal
view toward zoning regulations, will it uphold this statute?
18. No. A law that forbids people related to one another from living together is considered a substantive due process violation, because it involves the exercise of
a fundamental right (the right to privacy). Moore v. City of East Cleveland, 431 U.S. 494 (1977); E Ch.9-IV(N)(2).

19. NOTE:
20. The standard of review applied to this issue would certainly be higher than "rational relationship," but it's unclear whether strict scrutiny or intermediate
scrutiny is appropriate. (There's no clear answer, but the Court has, in recent years, found that a person's decision concerning how she conducts her family
life is equivalent to a "fundamental right," and thus receives strict scrutiny.) E Ch.9-IV(N)(1).

21. NOTE:
22. Unrelated persons have no fundamental right to live with each other. Belle Terre v. Boraas, 416 U.S. 1 (1974); E Ch.9-IV(N)(2)(e).

23. The State of Britannia becomes concerned that children from Scottish families are not learning English in schools in Scottish neighborhoods where teachers
frequently teach classes in Scottish. The state enacts a statute prohibiting elementary school classes from being taught in any language but English. The statute
applies to private and parochial schools as well as public schools. Would this statute survive a substantive due process attack?
24. No. The right to educate one's children as one desires is considered part of the right to privacy. Thus, a state would need a compelling interest to regulate the
right. Although the state may validly set reasonable educational standards, it cannot require education only in English. Meyer v. Nebraska, 262 U.S. 390 (1923);
E Ch.9-IV(B)(1); CH p. 798.

25. A Sun Valley state statute forbids the father of an illegitimate child from vetoing the child's adoption by another man, unless the father has obtained a court
order legitimating the child. Julius Caesar tries to stop his illegitimate daughter Sallad's adoption by her stepfather, Marc Anthony. Caesar has never
legitimated Sallad and never even lived with her. When he steps in to veto the adoption, the statute prevents him from doing so. He challenges the statute on
substantive due process grounds. What result?
26. The statute will be upheld. The government has a legitimate interest in family life and child rearing. Where a father has never lived with a child or sought
custody of him, the statute would preserve existing families and would therefore be in the child's best interests. Quilloin v. Walcott, 434 U.S. 246 (1978).

27. RELATED ISSUE:


28. The statute would also withstand an equal protection argument (the classification being unmarried fathers versus married ones, who could block adoption).
The distinction is valid on grounds that custody and responsibility over the child justifies the unequal treatment. E Ch.9-IV(N)(5); CH pp. 568 n.65, 803 n.40.

29. RELATED ISSUE:


30. A state law may create an irrebuttable presumption that a married woman's husband is the father of her child, thus denying the biological father parental
rights, including visitation. Michael H. v. Gerald D., 491 U.S. 110 (1989); CH pp. 804-805. The plurality opinion in Michael H. concluded that there is no
substantive due process tradition of protecting a biological father when the mother is married to another man.

31. Socrates, a resident of the State of Wrangler, engages in homosexual sodomy in between his teaching and philosophizing sessions. A Wrangler statute forbids
consensual homosexual sodomy between adults. Socrates is arrested for violating the statute in the privacy of his bedroom. He challenges the statute on
substantive due process grounds, claiming the right to engage in sodomy is "fundamental." What result?
32. The statute will not stand. Lawrence v. Texas, 539 U.S. 558 (2003), granted substantive due process protection to a person's sexual conduct apart from
procreation or family life, and held that states may not criminalize private homosexual conduct between consenting adults. Lawrence overturned Bowers v.
Hardwick, 478 U.S. 186 (1986), which emphasized that the only liberties regarded as fundamental are those that are either implicit in the concept of ordered
liberty or deeply rooted in the nation's history and tradition-and held that homosexual intimacy did not qualify as either. Although the Court's language
in Lawrence suggested broad protection for consensual homosexual intimacy, the opinion did not classify the interest in pursuing such conduct a fundamental
right; instead, it applied rational-basis review, finding that the statute involved furthered no legitimate state interest. E Ch.9-IV(O).
33. The State of Ames General Assembly recently passed an abortion statute. Among other provisions, the statute contains an informed consent provision
requiring a woman seeking a pre-viability abortion to receive information regarding the procedure 24 hours in advance of the procedure. The literature
describes the procedure, fetal growth, and mentions alternatives to abortion, including adoption. A nurse or doctor distributes the material and then
answers questions. No abortion can be performed — except in an emergency — for 24 hours after distribution of the material. In addition, a woman
seeking an abortion, in addition to receiving literature, must submit to an ultrasound. The age and development of the fetus are recorded by the doctor
(with the mother’s personal identifying information omitted) and the ultrasound is shown to the mother. Presenting the statute to the Assembly, its chief
sponsor said, “This will reduce the number of abortions performed in this state. What mother would knowingly kill her baby once she sees its picture there
on the ultrasound machine sucking its thumb and moving around in her womb?” Would a court likely uphold the Ames statute if it is challenged in a
lawsuit?
34. No, because the purpose was to place an obstacle in the path of someone seeking a pre-viability abortion
35. Recall that after Casey, the standard of review changed to the undue burden test. Because C describes part of the strict scrutiny standard of review, it is
incorrect. The new standard of review asks whether the regulation is a permissible means of achieving a legitimate interest, one of which is preserving the
state’s interest in potential life. While D restates this part of the Casey opinion, it does not go on to ask whether the means used here to express that
preference are permissible. To be a permissible means of achieving a legitimate end, the regulation may not be an undue burden, defined by the Casey joint
opinion as having the purpose or effect of placing a substantial obstacle in the path of a woman seeking a pre-viability abortion. While A is correct — not all
burdens will be “undue” — it does not address the question whether the ultrasound requirement is such a burden. Therefore, it is not the best answer. That
leaves B, which, on these facts, is the best answer. Casey says that undue burdens can result from either purposes or effects. The facts strongly suggest that
the purpose in proposing the ultrasound requirement was to pressure a woman not to have an abortion by showing her pictures of the fetus. The sponsor
says nothing about wanting the woman to make an informed choice; his remarks seem aimed only at preventing her from going through with the
procedure.

36. Which of the following restrictions on abortion would be least likely to pass constitutional muster if challenged?
37. a. A five-day period of reflection before an abortion could be performe
38. Rephrased in terms of Casey’s standard of review, the question is which of the choices would likely be found to be an “undue burden.” If you read the
foregoing material carefully, you will recall that two of the choices actually passed muster under Roe’s strict scrutiny regime. Maher and Harris upheld
funding restrictions on abortion, making clear that infringement of the right and refusal to subsidize it were conceptually distinct. Casey itself said that
simply making abortions more expensive did not, ipso facto, impose an undue burden on the right. Therefore, B would likely be upheld. The same goes
for D; a number of pre-Casey decisions made clear that as long as a judicial bypass was available, the requirement of parental consent for minors was
constitutionally permissible. The record keeping requirements described in C are similar to those upheld in Casey, so they are also unobjectionable. That
leaves A, which is the best answer. Although the 24-hour waiting period was upheld in Casey, the Court hinted that it might not be as solicitous toward
waiting periods in excess of 24 hours. Certainly the state would have a considerable burden to bear explaining why a period of five days was necessary to
promote “reflection.” Further, because Casey held that any regulation with the purpose or effect of placing a substantial obstacle in the path of a woman
seeking to terminate a nonviable fetus, even if the legislature’s heart was pure in believing that five days was necessary to promote reflection, the effect of
such a lengthy waiting period would likely be found to be a substantial obstacle.

The State of Ames has banned all employment of children under 17 years of age. “Employment” is defined broadly as any job, task, service, or undertaking for
which the child receives compensation in cash or in kind. Which of the following parties would have the greatest chance of successfully challenging the statute?
39. Parents of a 15-year-old who require their son to do housework and yard work in exchange for a weekly cash allowance.
40. Review time! This question pulls together a few threads of this chapter. We know that, after Lochner, economic regulations, like the Ames law here, will not
generally be subject to strict scrutiny, but will receive the more deferential rational basis review. The law does indeed infringe on the “liberty of contract” of
employers and employees, but that fact alone is not enough to warrant judicial intervention. Therefore, A and C are incorrect. But if you then
concluded D must be the correct answer, you were a little hasty. There is one potential plaintiff among the choices who might have a good chance of
winning — the parents! If, as Meyer, Pierce, and Troxelmaintain, parents have a fundamental right to raise their children relatively free from governmental
interference, then a law that is broad enough to forbid you from requiring your children to work in exchange for an allowance (or simply as part of the
family responsibilities) would seem to infringe upon that freedom. At the very least, the government would have to justify its sweeping definition of
employment, or its lack of an exception for entities such as family businesses. The case wouldn’t necessarily be a slam-dunk, but of the available
choices, B is the best answer.
41.
Following a scandal that received substantial attention in the press, Ames revised its probate code to prohibit inheritance by adult adoptees if the testator
had disinherited his natural children. Julius, a wealthy Ames citizen, had a natural son from whom he had been estranged for many years; he had no other
heirs. To prevent his substantial estate from passing to his son, Julius adopted Octavius, an adult with whom Julius was quite close. Julius then made a will
naming Octavius as his heir and disinheriting his natural son. After the will was executed, Ames’s law was passed. What is Julius’s strongest argument
against the constitutionality of the Ames inheritance law?
42. The law is not narrowly tailored to any compelling governmental interest
43. This question also serves as a mini-review. If you’ll recall, economic and social matters that don’t rise to the level of fundamental rights are deemed
legitimate objects of regulation, and are subject to a rational basis test. Regulation often interferes with one’s ability to dispose of property at will, so A is
incorrect. Moreover, the Court no longer, as in the days of economic substantive due process, passes on the reasonableness or unreasonableness of
economic regulation, meaning that B is not a good choice. D too is incorrect; numerous laws interfering with liberty of contract exist and are regularly
upheld by courts. That leaves C, which is the best answer. Note that this law does not simply regulate the disposition of property, but also has the effect of
interfering with the effects of an adoption and the disposal of property among family members. In essence, the state is relegating adopted children, in some
situations, to a second-class position. The law permits the disinheritance of some natural children for the benefit of other natural children, but forbids it
where the beneficiary is adopted. Such interference would likely trigger strict scrutiny.
44. Citing a study alleging that a high percentage of durable powers of attorney (DPAs) are either forged or obtained through fraud or undue influence, the Ames
General Assembly has prohibited the delegation of medical decisionmaking authority to surrogates. Wendy, the wife and designated surrogate of Bill, who is
in a persistent vegetative state, would like to exercise her authority and remove her husband’s ventilator. Wendy sues, claiming that the law violates the Due
Process Clause. A reviewing court would likely:
45. Strike down the law because it is not narrowly tailored
46. We know from Cruzan and Glucksberg that the Court seems to regard the right of a competent individual to refuse unwanted medical treatment as
fundamental. The Court stressed in Glucksberg and Vacco, however, that there was no right to commit suicide, assisted or not. Therefore, A is not the correct
answer — there is no fundamental right to “die.” State infringements of fundamental rights, moreover, are not deferentially reviewed under a rational basis
standard, but rather are subject to strict scrutiny. B, therefore, is not the correct answer because it states the wrong standard of review. D is not the right
answer either, because though the statute is an exercise of the state’s police power, that says nothing about whether it is a constitutional exercise of that
power. That leaves C, which is the best answer. The right to refuse unwanted medical care is a fundamental right, according to the Court. Delegating that
decisionmaking power to a surrogate guarantees that one’s wishes will be respected if one becomes incapacitated. While the prevention of fraud and
preservation of life are no doubt compelling state interests, those interests could be achieved short of barring all durable powers of attorney; additional
witness requirements, for example, could help ensure that the document was genuine and that no undue influence was applied. Depriving otherwise
competent individuals of all means to ensure their wishes will be carried out in the future would sweep too broadly to survive strict scrutiny.
47. Alice is 16 years old and was recently married. Shortly after her marriage, the state terminated her $550-per-month welfare grant under a law that allows
those who are under 21 to receive welfare benefits only if they are unmarried. Does this law violate Alice’s rights under the Fourteenth Amendment Due
Process Clause?
48. Alice will argue that the law interferes with her liberty interest in marriage. Since the right to marry is a protected and fundamental liberty interest, the first
two steps of the basic due process analysis are satisfied. Alice will have a harder time meeting the undue burden requirement. The state has not directly
interfered with the marriage decision by prohibiting her from getting married. On the other hand, it has indirectly punished her for deciding to marry by
terminating her subsistence benefits as a result. In Califano v. Jobst,434 U.S. 47 (1977), the Court held that the termination of Social Security benefits to a child
who marries does not impinge on the liberty interest in marriage; there, however, the benefits were relatively low and were not intended to meet subsistence
needs. Alice will claim that because of the critical nature of the benefits at stake, her case is distinguishable from Jobst and that the state has, therefore,
impinged on her fundamental liberty to marry.
49. If the Court agrees, the law will receive strict scrutiny. Under step 4, the state may claim a compelling interest in encouraging children on welfare to work or
educate themselves before starting their own families, thus reducing the chances that they will continue to be dependent on welfare. Alice would respond
that the mere saving of money is not a compelling interest; if it were, the government could justify curtailing our most basic liberty interests such as freedom
of speech and assembly on the ground that their exercise is too costly. Even if the interest in saving money were compelling, Alice will argue under step 5 that
the state has less burdensome ways to achieve its goals, such as simply requiring children who are on welfare to work or attend school whether or not they
are married. Thus, if the court applies strict scrutiny, the law might well be struck down on the basis that it was not necessary for the state to interfere with
Alice’s right to marry to achieve its goal of reducing welfare costs. If, on the other hand, the court were to apply the rational basis test, the law would be
upheld, since a rational legislature could reasonably conclude that this law was a good way to reduce long-term dependence on welfare.
50. A city ordinance provides that no household may be occupied by more than two persons under the age of ten. As a result of the ordinance, the Sutro family,
which includes three children under the age of ten, is being evicted from the apartment where they have lived for the past 12 years. Can the Sutros avoid
being evicted on the ground that the ordinance violates their rights under the Fourteenth Amendment Due Process Clause?
51. The fundamental liberty interest recognized in Moore protects the right of persons who are related by blood or marriage to live together in a common
household. The city’s ordinance directly and substantially interferes with this right by barring the Sutros from preserving the integrity of their family unit.
The only way they could avoid being evicted is by sending one of the children to live somewhere else or by giving the child up for adoption. The ordinance
clearly impinges on the family’s fundamental liberty interest. It is no answer that the Sutros could live together as a family in some other city. This option was
available to the family in Moore but did not suffice to avoid the application of strict scrutiny there.Since the first three steps of the due process analysis have
been satisfied, the ordinance will be struck down unless the city can show that it furthers a compelling governmental interest and that it is the least
burdensome means of doing so. The city may argue that it has an interest in preserving the peace and tranquility of residential areas and that households
with many young children often disturb others, particularly senior citizens. It is far from clear that this interest is sufficiently compelling to justify a law that
strikes so close to the heart of the family. Small children have always been part of the American landscape, and the noise that they make is an unavoidable
fact of life.Even if a court were to find this interest to be compelling, there are less burdensome ways of achieving the city’s goals, such as by enforcing noise
control ordinances, allowing landlords to limit the occupancy of certain buildings to senior citizens, or perhaps even zoning parts of the city exclusively for
adults.

52. Beverly owns a house with ten bedrooms. She lives there with eight adult boarders who are not related by blood, marriage, or adoption. Over the years, the
group has developed strong social, economic, and psychological commitments to each other. They share expenses, rotate chores, and eat evening meals
together. In addition, they often attend movies, go bowling, and take vacations together. A city zoning ordinance provides that no household may contain
more than five people who are unrelated by blood, marriage, or adoption. The city has sued Beverly for an injunction, claiming that she is in violation of the
ordinance. May Beverly oppose enforcement on the ground that the ordinance violates her rights under the Fourteenth Amendment Due Process Clause?
53. Beverly cannot invoke the fundamental liberty interest in protecting family living arrangements, since this right applies only to groups that are related by
blood, adoption, or marriage. However, she may be able to invoke the privacy-based right of intimate association. In contrast to the students in Belle Terre, her
household shares many attributes normally associated with a typical family. Beverly can argue that the group is relatively small and congenial, that she is
very selective in whom she takes in as boarders, and that the group’s purpose transcends mere convenience. She and her tenants have formed bonds that are
intimate and enduring, and similar to those developed in traditional family households.
54. If the court finds that Beverly and her tenants are protected by the right of intimate association, the city’s effort to break up her household impinges on this
right so as to trigger strict scrutiny. Even if the city has compelling interests in limiting density, controlling noise, and reducing the number of cars in
residential areas, there are other ways to achieve these goals without burdening the right of intimate association. See City of Santa Barbara v. Adamson, 27
Cal. 3d 123, 610 P.2d 436 (1980) (state constitution barred use of zoning ordinance to force dissolution of household of 12 unrelated persons who shared
many attributes of family).

55. A state law provides that before an abortion may be performed, a woman must have two in-person consultations with a physician, at least 48 hours apart,
and another 48 hours must elapse between the second consultation and the abortion. At each consultation, the physician must seek to persuade the woman
not to have an abortion. Does this law violate a woman’s rights under the Due Process Clause?
56. The law may well be valid. Under the undue burden standard, the state may enact laws whose explicit purpose is to persuade pregnant women to choose
childbirth over abortion. While a challenger might claim that the state’s goal here is to prevent a woman from exercising free choice, the state would
successfully reply that it is seeking to persuade rather than hinder and that it is also trying to assure that a woman’s decision to have an abortion is deliberate
and well informed.Since the purpose of the law does not violate the undue burden test, the measure will be found invalid only if its effect is to place a
substantial obstacle in a woman’s path by preventing her from having an abortion. The consultation and waiting-period requirements will surely increase the
cost and difficulty of having an abortion, for a woman must make three separate trips to a clinic, hospital, or doctor’s office. Yet increased cost or increased
difficulty alone does not constitute an undue burden. Without proof that the effect of this law is to actually prevent abortions, the measure would be upheld
under the undue burden test.

57. Robert is suffering from a form of cancer for which there is no proven cure. For the past few months, he has been using a new drug for treating cancer that
was developed by a French company but that has not been approved by the U.S. Food and Drug Administration (FDA). Robert has been indicted under a
state law that makes it a crime to use any medication that has not been approved by the FDA. May Robert defend the action on the ground that the state, by
barring him from using the drug, is violating his Fourteenth Amendment due process right to privacy and personal autonomy?
58. If the court is willing to recognize that the fundamental right of privacy extends to decisions concerning what medical treatment a person wishes to receive,
the court must still decide whether the state has adequate grounds for interfering with this right. The state will claim that it has a compelling interest in
protecting public health and safety and that until a drug is approved by the FDA, there is no way to be sure that the drug will not cause serious harm or even
death to those who use it. If the court uses a variation on the basic strict scrutiny model and asks only whether the state has a compelling interest (see §2.4.4),
Robert will probably lose, since the state’s interest in public health will probably be found to outweigh an individual’s liberty interest in choosing his or her
own medical treatment.On the other hand, if the court applies the basic strict scrutiny due process test and insists (under the fifth step) that the state use the
least burdensome means of achieving a compelling interest, the law may be unconstitutional as applied to Robert. Since he suffers from an otherwise fatal
disease, to apply the law to him is arguably more burdensome than necessary, for it does little, if anything, to advance the state’s goals. A less burdensome
approach would be for the state to make sure that Robert is aware of the risks entailed, but once his decision is fully informed, it should prevail over the
state’s interest in protecting public health and safety.
59. The state of Aha has enacted a statute that proscribes what the statute refers to as “unnatural sexual acts.” The acts described include oral sex. The statute
applies to conduct between married persons as well as to conduct between unmarried persons, but contains an exemption for con duct that takes place in a
dwelling that is the residence of one or both parties. Joe and Martha Danzig, a married couple, were vacationing at the Happy Times Motel, when a state police
officer burst into the room acting on a tip (reasonable-seeming but erroneous) that the couple was using drugs. The officer happened to see Joe and Martha
engaging in oral sex at that moment, and arrested them.
60. (a) If Joe and Martha want to challenge the charges on constitutional grounds, what is their best argument? _________________
61. (b) Will this argument succeed? _________________
62.
(a) That the charges violate the defendants’ substantive due process right to “privacy.”
63. (b) Yes, probably. The Supreme Court has held that every individual has a “zone of privacy,” and that government action which invades that zone will be
found to violate the individual’s substantive due process rights unless the government action is necessary to achieve a compelling governmental objective
(strict scrutiny). The Supreme Court has never explicitly held that the sexual acts of a married couple, taken in private, fall within the zone of privacy for all
purposes. However, the Court has held (in Griswold v. Connecticut) that the right to privacy is violated when the state interferes with a couple’s attempts to
use birth control. Later cases suggest that state interferences with a married couple’s sexual intimacy would similarly violate the right to privacy, assuming
that the conduct took place in private (even if it took place in a hotel room or other non-residential but non-public setting). See, e.g., Lawrence v. Texas, the
homosexual-sodomy case, where the majority opinion refers to “an emerging awareness that liberty gives substantial protection to adult persons in deciding
how to conduct their private lives in matters pertaining to sex.” In summary, since Joe and Martha are married and were performing the activity in private,
their conduct probably fell within the protected zone of privacy. (It is a bit less clear that their “right of privacy” argument would prevail if they had been
unmarried, or engaged in an adulterous relationship.)
64. A number of states have enacted regulations bearing on specific aspects of abortion. Consider the following:
65. (a) The state of Aloha provides that no abortion may be performed within the state if at the moment of the procedure the fetus is more than three months
old.
66. (b) The state of Brie provides that no abortion may be performed on a married woman unless she signs an affidavit that she has notified her spouse. However,
the prohibition does not apply if the married woman instead signs an affidavit that she and her husband are not living together, or that her husband is not the
father of the child, or that she has not given notice because she fears that he will abuse her if he finds out that she is planning an abortion. The statute is
challenged by P, a woman who is two months pregnant, wants an abortion, and does not want to notify her spouse (with whom P lives, whom she believes to
be the father of the child, and who is not likely to abuse her if she tells him she wants an abortion).
67. (c) The state of Chaucer provides that “No physician shall perform or induce an abortion in this state unless, at the time of the abortion procedure, the
physician has patient-admitting privileges at a hospital located within 20 miles of the facility in which the procedure is performed.” The preamble to the
statute says that it was enacted for the purpose of ensuring that if a woman should encounter medical complications arising out of the abortion procedure,
she can be admitted to a hospital for emergency treatment without delay. The statute replaces a previously-in-force provision (the “old statute”) that required
any physician providing an abortion to have made arrangements in advance with a physician having admitting privileges at a local hospital; the earlier statute
was enacted for the same purpose of ensuring prompt hospital treatment in case of complications from an abortion. Prior to enacting the new statute, the
legislature did not conduct any hearings or otherwise consider evidence about the number of women who, under the old statute, had suffered abortion-
related medical complications requiring hospitalization. The statute is challenged by P, a physician who wishes to perform abortions in Chaucer, and who
does not have admitting privileges at any local hospital. At a trial, P produces evidence (not rebutted by the state) that during the prior 10 years when the old
statute was in force, no woman was admitted — or sought to be admitted — to a hospital for emergency treatment of post-abortion complications on the day
on which she received the abortion. P also shows that most hospitals in Chaucer will not grant admitting privileges to a physician unless the physician
demonstrates that he or she will send a regular flow of patients to that hospital, a showing that neither P nor most abortion providers in the state will be able
to make precisely because abortions so rarely leads to medical complications that require hospitalization.
68. State whether each of these provisions is constitutional. (Assume that the suits in (a) and (c) are attacks on the face of the statute, and that the attack in (b) is
“as applied.”) _____________________________
69. (a) Unconstitutional. Under Whole Woman’s Health v. Hellerstedt, the states may not place “undue burdens” on the right of abortion. A state regulation will
constitute an undue burden if the regulation has the purpose or effect of placing a “substantial obstacle” in the path of a woman seeking an abortion of a non-
viable fetus . Id. Since a four- or five-month-old fetus is certainly not viable even under present medical advances, the blanket prohibition on abortions during
this time frame would almost certainly be found to be a “substantial obstacle” and thus an “undue burden.” (But a state probably could bar all
abortions after the fetus becomes viable.)
70. (b) Unconstitutional, probably. Planned Parenthood v. Casey struck down a spousal notification provision. The provision here, while it may contain slightly
more escape hatches, would likely be found to be a substantial obstacle to the plaintiff (who is not covered by any of the escape hatches). Notice that the
attack here is “as applied” rather than “on the face” of the statute. That is, the suit argues that the statute directly violates the rights of the plaintiff, rather than
asking the Court to prevent the statute from being enforced against anyone because it violates the rights of persons not before the Court. The present Court
would likely find that the statute acts as an undue burden upon P, even if that Court wouldn’t allow a facial attack on the spousal notification provision. (The
pre-Hellerstedtopinion in Gonzales v. Carhart indicates that the Court will allow facial attacks only if the plaintiffs prove that the statute would be
unconstitutional “in a large fraction of relevant cases”; the plaintiffs in a facial attack might not be able to make this “large fraction” showing, given the various
escape hatches.)
71. (c) Unconstitutional. An abortion regulation will be unconstitutional if either its purpose or effect is to place a “substantial obstacle” in the path of a
woman seeking an abortion of a pre-viable fetus. Or, under a slightly different formulation of the standard, the regulation will be unconstitutional if it imposes
an “undue burden” on pre-viability abortions. Whole Woman’s Health v. Hellerstedt. Not all burdens on abortion are unconstitutional; only those burdens that
are “undue” violate women’s substantive due process rights. A reviewing court will classify the burden as “undue” if the burdens imposed by the
restriction outweigh the benefits it confers. And at least where the benefits claimed by the legislature are not medically certain , the reviewing court is not
to give “uncritical deference” to the legislature’s judgment about medical benefits, but is instead to make its own factual review of the existence and size of
those benefits.
72. The statute here, and the factual setting behind it, are almost identical to one of the two Texas struck down by the Supreme Court in Hellerstedt. There, the
majority concluded that a requirement that abortion physicians have local-hospital admitting privileges conferred no medical benefit at all compared with
the prior statute (which merely required the provider to have a working arrangement with another physician who did have such admitting privileges). This
was so in part because even under the old statute, extremely few if any post-abortion emergency hospital admissions occurred, especially on the actual day
of the abortion (which was the only time the provider’s ability to arrange an emergency admission would be even theoretically applicable). On the “burden”
side, the Court noted that abortion providers can rarely obtain admitting privileges, because the very safety of the abortion process means that a provider
usually can’t generate the frequent hospital admissions that hospitals typically require as a condition of granting admitting privileges; therefore, requiring
providers to obtain unobtainable admitting privileges places a sizable burden on the availability of abortions. When this burden was balanced against what
the Court found to be non-existent medical benefits, the former clearly outweighed the latter, making the burden in Hellerstedt “undue.” Here, there is nothing
about the facts of the Chaucer statute that is materially different from the admitting-privileges requirement struck down in Hellerstedt, so it is virtually certain
that the present Court would strike down the statute here as well.

Procedural due process


73. Procedural due process is required when one is deprived of life, liberty, or property. Is "property" confined to land or chattels?
74. No; it also covers an interest already acquired in specific benefits. For instance, a high school student has a property interest in his education; welfare
recipients have a property interest in continued benefits; drivers have a property interest in retaining their driver's licenses. E Ch.9-V(C)-(I); CH §7.3.2.

75. N.B.:
76. A person's mere "expectation" of maintaining benefits (or a job) isn't sufficient to create a property right; the person must have a "legitimate claim of
entitlement" to continued benefits, and this in turn requires a government-created expectancy pursuant to applicable law (e.g., tenure for public school
teachers). E Ch.9-V(E); CH pp. 561-563.

ANALYZING PROCEDURAL DUE PROCESS PROBLEMS


1. Has a government action impaired a liberty or property interest?
If not, procedural due process is not an issue. If so, go on to #2.
2. What procedural requirements must be satisfied?
The two principal tools are notice and a hearing. However, they can range from a full, adversarial prior hearing to a promise of a prompt postdeprivation
evidentiary hearing to a notification of charges and an opportunity to respond. The requirements are determined by weighing the importance of the property
or liberty interest in question and the risk an erroneous deprivation in a particular procedure would create (considering the probable value of any additional
safeguards) versus the importance to the government of the function in question, and the administrative and fiscal expense of a particular safeguard.

RATIONALE FOR PROCEDURAL DUE PROCESS:


To prevent inaccurate decisions. Where there is little risk of error, the individual will be entitled to correspondingly less procedural protection. E Ch.9-V(J)(2).

NOTE:
Although procedural due process also applies to the deprivation of life, that won't be discussed in these cards, as deprivation of life without procedural due process is an
overwhelmingly obvious problem you can undoubtedly spot on your own.

How did you do?

Next

Flag for later

77. What factors determine the procedural safeguards necessary in depriving someone of a liberty or property interest?
78. Hide Answer
79. Courts employ a balancing test–the importance of the property or liberty interest in question and the risk an erroneous deprivation in a particular procedure
would create (considering the probable value of any additional safeguards) are weighed against the importance to the government of the function in question
and the administrative and fiscal expense of a particular safeguard. Mathews v. Eldridge,424 U.S. 319 (1976); E Ch.9-V(J)(2); CH §7.4.2.

PROCEDURAL DUE PROCESS REQUIREMENTS OF COMMON PROBLEM AREAS


80. Welfare benefit termination: Notice + pretermination evidentiary hearing. Goldberg v. Kelly, 397 U.S. 254 (1970); E Ch.9-V(C)(1); CH p. 584.
81. Disability benefit termination: Notice + promise of posttermination evidentiary hearing. Mathews v. Eldridge, 424 U.S. 319 (1976); E Ch.9-V(J)(2)(b); CH pp.
584-585.
82. Public employment termination: If for exercising constitutional right (e.g., speech)-prior hearing. Perry v. Sindermann, 408 U.S. 593(1972); E Ch.9-V(E)(2);
CH p. 587. If not, depends on whether employee can only be terminated for cause. If employee can only be removed for "cause"–notice with opportunity to
respond + posttermination evidentiary hearing. Arnett v. Kennedy, 416 U.S. 134 (1974). If employee needn't be removed for cause (i.e., serves at the will of
public employer)–there is no property right so no procedural due process problem.
83. Public school suspension for disciplinary reasons: If student's presence in school is dangerous or disruptive–removal immediately followed by notice of
charges + opportunity to explain. If not dangerous or disruptive, and suspension is for substantial length of time (e.g., ten days)–notice + explanation of
charges + opportunity to explain must all precede removal. Goss v. Lopez, 419 U.S. 565 (1975). If not dangerous or disruptive and suspension is brief (e.g., two
or three days)–ex parte review by school official suffices. Bethel School District v. Fraser, 478 U.S. 675 (1976); E Ch.9-V(I), (J)(7)(a).
84. Public school suspension for academic reasons: Prior notice + opportunity to respond (no hearing is required). Board of Curators v. Horowitz, 435 U.S. 78
(1978); E Ch.9-V(J)(7)(b); CH p. 591.
85. Prejudgment garnishment of wages by creditors: Notice + prior hearing. Sniadach v. Family Finance, 395 U.S. 337 (1969); CH p. 599.
86. Prejudgment seizure of property (on installment sales contract) by creditors: No prior notice or hearing as long as:
1. Creditor applies to judge (not clerk) for prejudgment seizure;
2. Application includes affidavit based on personal knowledge of specific facts (not conclusions) justifying seizure;
3. Creditor posts sufficient surety bond;
4. Provision made for prompt postseizure hearing, where creditor must prove probable cause for seizure. Mitchell v. W.T. Grant, 416 U.S. 600
(1974); CH p. 600.
87. Driver's license suspension: If probable cause to believe conditions of license have been violated–prompt postsuspension hearing. NR p. 667.
88. Involuntary commitment to mental institution–Adult: Clear and convincing evidence of need to commit; notice and hearing except in exigent
circumstances. Addington v. Texas, 441 U.S. 418 (1979). Child: Precommitment inquiry, but no prior adversary hearing if inquiry is sufficient. Parham v.
J.R., 442 U.S. 584 (1979); CH pp. 591-592.
89. Termination of parental status: Prior hearing where parental unfitness must be proven by "clear and convincing evidence." Santosky v. Kramer, 455 U.S. 745
(1982); CH p. 588.
90. Detention of "enemy combatants" who are citizens: U.S. citizens detained during war are entitled to due process, including at a minimum the right to counsel
and the right to go before a neutral decision maker. Hamdi v. Rumsfeld, 542 U.S. 507 (2004); E Ch.9-V(J)(5).
John Merrick, the "elephant man," receives disability benefits from the state. The state now wants to terminate those benefits, because it believes he is no longer
disabled within the meaning of the disability–benefits statute. Does the state have to give him an evidentiary hearing (at which he can show he is in fact still disabled)
before the termination goes into effect?

91. No. Procedural due process rights attach only when one is deprived of a property or liberty right. Deprivation of one's disability benefits is a right that does
qualify for protection. Determining the amount of protection necessary requires balancing the importance of the property or liberty interest in question and
the risk an erroneous deprivation in a particular procedure would create (considering the probable value of any additional safeguards) against the importance
to the government of the function in question, and the administrative and fiscal expense of a particular safeguard. The individual's interest in disability
benefits is not considered "vital" (unlike welfare payments), as they are not based on need; there is little risk of error, as the determination is based on medical
evidence, which can be presented in document form without requiring oral testimony; and the cost of full hearings would severely burden the government.
Thus, prior notice, the opportunity for a written response, and a posttermination evidentiary hearing are together sufficient to satisfy procedural due process
requirements. Mathews v. Eldridge, 424 U.S. 319 (1976); E Ch.9-V(J)(2); CH pp. 584-585.
92. Old Mother Hubbard receives a notice in the mail informing her that her welfare benefits have just been cut off. This is the first that Mother has heard about
the possibility of losing her benefits. Mother claims that this action violates her due process rights. Does it?
93. Yes. Procedural due process rights only attach when one is deprived of a property or liberty right. Deprivation of one's welfare benefits is a right that qualifies
for protection. Determining the amount of protection necessary requires balancing the importance of the property or liberty interest in question and the risk
an erroneous deprivation in a particular procedure would create (considering the probable value of any additional safeguards) against the importance to the
government of the function in question and the administrative and fiscal expense of a particular safeguard. Mathews v. Eldridge, 424 U.S. 319 (1976). The
Court has held that a welfare recipient is entitled to notice and an evidentiary hearing prior to the termination of such benefits. Goldberg v. Kelly, 397 U.S. 254
(1970); E Ch.9-V(C)(1); CH p. 584.

94. Libby Zbiblenik is the executive in charge of Meaningless Paper Shuffling at City Hall. She has an employment contract mandating that she can only be fired
for "cause," including any kind of misfeasance or incompetence. Libby comes to work one day and finds a sinister envelope in her inbox. The envelope contains,
along with two McDonald's gift certificates, a letter telling Libby she is to be dismissed two weeks hence, due to the poor quality of her work. The note further
tells her she can request a post-termination hearing as to the cause of her dismissal. Has Libby been denied due process?
95. No. Procedural due process rights attach only when one is deprived of a property or liberty right. Deprivation of public employment, when one can only be
terminated for cause, is a property right that qualifies for protection. Determining the amount of protection necessary requires balancing the importance of
the property or liberty interest in question and the risk an erroneous deprivation in a particular procedure would create (considering the probable value of
any additional safeguards), against the importance to the government of the function in question and the administrative and fiscal expense of a particular
safeguard. Mathews v. Eldridge, 424 U.S. 319 (1976).
96. In the case of a job requiring good cause for dismissal, due process requires merely notice beforehand and the opportunity for a post-termination evidentiary
hearing (at which she can get back pay and reinstatement if she's successful). An evidentiary hearing beforehand is not part of the required procedural
package, because of the ability of back pay and reinstatement to make the job holder whole. Arnett v. Kennedy, 416 U.S. 134 (1974); CH p. 587.

97. RELATED ISSUE:


98. Say instead that Libby was fired after speaking out against the mayor at a city council meeting. Then she would be entitled to a hearing before she was fired,
because a prior hearing is necessary if the dismissal is based on the exercise of a constitutional right. NR p. 667.

99. RELATED ISSUE:


100. Say that Libby didn't have the "for cause" clause in her contract and could be fired at will. Assuming she was fired for something other than exercising a
constitutional right (e.g., incompetence), she would not have a property right in continued employment, so she would not be entitled to procedural due
process at all.

Lala Bamba, an alien from Colombia, has been convicted of murder in a state court. She was not permitted to obtain counsel. When she challenges her
conviction on grounds that the lack of counsel violated her due process rights under the federal Constitution, the government claims that aliens aren't
protected by the Due Process Clause. Who's correct?
Lala Bamba; aliens (and corporations) are considered "persons" for purposes of due process. CH p. 767.
Lizzie Borden is a mentally unbalanced ten-year-old. Will a prior adversary hearing be necessary before she can be committed to a mental institution?
101. No. First, procedural due process rights only attach when one is deprived of a property or liberty right. Deprivation of one's freedom is a liberty right that
clearly would qualify for protection. Second, determining which procedural safeguards are called for requires balancing (1) the strength of the property or
liberty interest in question, the risk of erroneous deprivation of the interest through applicable procedures, and the probable value of the additional or
substitute procedural safeguards, against (2) the government's interest in the function involved, and the fiscal and administrative expense that the additional
procedural safeguards would entail. Mathews v. Eldridge, 424 U.S. 319 (1976); E Ch.9-V(J)(2)(b).
102. Miss Brooks teaches at Shiny Apple State College. She is serving her fourth consecutive one-year contract. State law grants tenure after the completion of
seven consecutive one-year contracts. Brooks is told she won't be rehired for a fifth year and is given no reasons why (which is proper under state rules).
Could Brooks possibly establish a property right in her continued employment?
103. Yes; and, in doing so, she could claim her procedural due process rights were violated by her termination without a hearing.Procedural due process rights
only attach when one is deprived of a property or liberty right. A legitimate claim to continued public employment requires a government-created expectancy,
pursuant to applicable law (e.g., tenure). However, tenure can be de facto–that is, college rules, regulations, and practices could create a tenure system in
fact. Perry v. Sindermann, 408 U.S. 593 (1972). Say, for instance, that the college president orally assured Brooks that she'd be rehired, and she reasonably
relied on this promise. This would probably be enough to create a property right, such that due process rights would attach to her dismissal. E Ch.9-V(E)(2);
CH p. 561.
In a last-minute effort to close a yawning budget deficit, the State of Ames doubles the tax on cigarettes sold in the state. Convenience store and gas station
owners sue, protesting the lack of both notification and hearing prior to the enactment of the tax. The state, they allege, violated the Due Process Clause. A
reviewing court should:
Dismiss the suit, because the imposition of the tax was not an adjudicative determinati
Recall that due process protections operate at the retail, not the wholesale, level. As the Court has put it, legislative determinations of policy do not trigger
procedural due process restrictions; adjudicative determinations do. Decisions to impose or raise taxes on a large group of people by legislatures must be
controlled at the ballot box. Therefore, B is the correct answer. A is not correct because the taxpayers do, of course, have property interests in their money,
but they must protect them in this instance by voting. C is correct about the core requirements of due process, but incorrect insofar as it suggests they apply
to these facts. D suggests that property is inviolate, and this is contrary to the Due Process Clause, which anticipates governmental deprivation of life,
liberty, or property if due process is observed.
Which of the following would likely be a cognizable liberty interest to which due process requirements attach?
The key to a liberty interest, according to the Court, is the alteration of one’s legal status. In each case above, the individual is subject to an alteration of his
or her status that would likely trigger due process protections. Thus, D is the correct answer. The subject in Awas permitted to drive prior to an official
determination that he was “unsafe,” after which he could no longer drive. That is not unlike the drinker in Constantineau. The Court in Hamdi held that due
process required that American citizens classified as enemy combatants be able to challenge their classification so that they might escape confinement.
Thus, B also describes a recognized liberty interest. And finally, like the student subject to school discipline in Goss v. Lopez, in C the student’s being subject
to corporal punishment involves a loss of liberty that would require some kind of notice and opportunity to be heard.
John, who shares the name of a known terrorist, finds that he is on the federal no-fly list, which complicates his efforts to travel by air. The no-fly list, which
is maintained by the Federal Aviation Administration pursuant to a provision in a congressional statute, contains the names of people who are suspected of
terrorism and therefore are not permitted to board flights in the United States. John sues, arguing that because he flies for business purposes, his ability to
travel is very important to him; thus, he should have received notice and been granted a hearing prior to being placed on the no-fly list by the federal
government. His placement on the list, he maintains, violates the Due Process Clause. A reviewing court should:
Dismiss his suit if, despite the importance of flying to John and a chance that additional procedures could keep some people from being erroneously placed on the list, it
would be expensive and administratively burdensome for the government to provide a pre-deprivation hearing.
Rationale for Correct Answer
While the creation of the list was a legislative adjudication, John’s inclusion on that list would be characterized as the result of an adjudicative determination that he
belonged on it. So A would not be correct. Also, it is not true that pre-deprivation hearings are required merely where a particular interest is subjectively important to an
individual; that was the old Goldberg v. Kelly rule abandoned in Matthews v. Eldridge. Therefore, B is an incorrect statement of the law. C is not correct because cost and
administrative burden are considered along with the importance of the interest, the risk of erroneous deprivation, and the probable value of additional safeguards in
preventing errors, after Matthews v. Eldridge. If the government could show that it was very costly and administratively burdensome to provide hearings to everyone, it
would likely win, despite the hardship and inconvenience to John caused by the lack of pre-deprivation hearings. Thus, of the four, D is the best answer.

A state X law prohibits doctors from performing more than one abortion per month; any doctor who violates this statute is barred from the further practice of medicine.
Dr. Mary Jones was recently notified by state X that her medical license has been revoked because she performed two abortions last March. How might Dr. Jones
challenge the validity of the state’s action?

Dr. Jones might first argue that the law is substantively invalid because it interferes with her liberty interest in practicing her profession or because it burdens her patients’
liberty interest in the abortion decision. If either of these arguments is successful, the law will be struck down as a matter of substantive due process, and Dr. Jones will be
allowed to keep her license.
However, if these challenges to the substantive validity of the law fail, Dr. Jones might argue that her procedural due process rights were violated because the state, in
applying this otherwise valid law to her, did not use a fair decision-making process. She would assert that state X should not be allowed to revoke her license without first
giving her notice and an opportunity to be heard. If Dr. Jones were to prevail on this procedural due process argument, the state could not revoke her license until after it
granted her a hearing. This would allow Dr. Jones to show that the state was mistaken in believing she had violated the law or that it was unfair under the circumstances
to enforce this law against her.
As a lawyer representing Dr. Jones, you would obviously prefer to win on substantive as opposed to procedural due process grounds. A substantive due process victory
will preclude the state from revoking your client’s license even if she violated the statute. By contrast, a procedural due process argument will simply afford your client a
chance to be heard and to show that the state was mistaken concerning her conduct. Nevertheless, a procedural due process challenge is still of value, for if your client is
given an opportunity to present her side of the story, she may be able to persuade the government to change its mind.

Smith was convicted of burglary and sentenced to ten years in prison. A state law authorizes mandatory sentence reduction on the basis of prisoners’ good behavior.
Under this law, prisoners may earn good-time credits, which can be revoked only for “flagrant or serious misconduct.” Prison officials revoked Smith’s good-time credits
on the ground that he had smuggled a knife into his cell; the revoked credits would have allowed Smith to leave prison six months early. Did Smith have a procedural due
process right to notice and an opportunity to be heard before the state revoked his good-time credits?
Smith has no constitutionally based liberty interest in receiving credit for good behavior. The Constitution does not compel prison officials to reward good behavior in
this manner. Yet Smith may be able to show that he has a nonconstitutional liberty interest in a shortened prison sentence, for state law seems to allow good-time credits
to be revoked only for specified reasons—i.e., for “flagrant or serious misconduct.” If Smith can show that these grounds are exclusive in the sense that they constitute a
mandatory limitation on the discretion of prison officials, he will have a state-created liberty interest protected by the Due Process Clause. See Wolff v. McDonnell, 418
U.S. 539 (1974) (finding protected state-created liberty interest based on similar facts). In that event, Smith’s due process rights would have been violated by the state’s
revocation of his good-time credits without notice and an opportunity to be heard.
Chuck was hired as a summer law clerk in the State Attorney General’s Office. He expected to work during June, July, and August, but on July 2, Chuck was fired. When he
asked why, his boss told him that he could not divulge the reason. Was Chuck entitled to notice of the reasons for his firing and an opportunity to be heard?
To establish that he had a property interest in the summer clerkship, Chuck would have to show that there was some basis on which he could legitimately and objectively
expect not to be fired without cause. If there was no statute, regulation, written office policy, or formal contract to this effect, Chuck might be able to rely on correspondence
or on advertisements announcing the position. If, for example, there was a letter offering him a three-month clerkship or the Attorney General’s Office announced the
position as being for a three-month period, a court might find an implicit assurance that the job would not be terminated within that period unless good cause existed.
This would be enough to create a property interest in the position.
If Chuck cannot establish a property interest in the position, he is unlikely to trigger procedural due process protection on the basis that a liberty interest was impaired.
While he does have a liberty interest in pursuing employment, he remains free to seek other work. Nor was his liberty interest in his good name directly impaired, since
no reasons were given for his termination. See Board of Regents of State Colleges v. Roth, 408 U.S. 564 (1972).

Delores was found eligible to receive workers’ compensation benefits based on her having suffered a work-related injury. A state statute provides that those found
eligible to receive workers’ compensation benefits are also eligible to have their employer or the employer’s workers’ compensation insurance carrier pay for all of the
worker’s “reasonable and necessary” medical treatments. If a worker’s doctor submits a bill, the insurer may challenge the expense and withhold payment until a
“utilization review” hearing is held to determine whether the expense was “reasonable and necessary.” Delores has challenged this procedure claiming that by
withholding medical payments without a pre-deprivation notice and opportunity to be heard, the state has violated her right to procedural due process. How should the
court rule on this constitutional claim?
The state has not violated Delores’s procedural due process rights. Although she has been found eligible to receive workers’ compensation benefits, eligibility for medical
payments exists only if a particular expense was “reasonable and necessary.” While Delores clearly has a property interest in her claim for medical payments since her
claim cannot be denied without cause, she does not have a property interest in any medical payment until it has first been determined that the treatment in question was
reasonable and necessary. Only then does a property interest in the payment attach. Therefore, the right to hearing before she is deprived of her property does not require
that medical payments be paid before a hearing has occurred. See American Manufacturers Mutual Ins. Co. v. Sullivan, 526 U.S. 40 (1999).
If, on the other hand, Delores had been found eligible to receive payment for a series of medical treatments that were deemed to be reasonable and necessary, she would
probably be entitled to notice and hearing before the carrier could withhold further payments for those treatments. In this case, Delores would have a property interest in
the payments themselves, not just in her claim for payment. See id. at 63 (Breyer, J., and Souter, J., concurri
The town of Corinth advertised an opening for a position as secretary to the City Clerk. The ad described the job briefly, and said nothing about the criteria that would be
used to fill it. Jane Brown applied for the job. She was given a typing test, and then an interview with George Crako, the City Clerk. Crako chose somebody else for the job.
Brown asked for a statement of why she didn’t get the job, but the Clerk’s office refused to respond.
Brown later heard from the grapevine that Crako had told someone else in the department that he thought Brown was probably the best at performing the technical tasks,
but that he had declined to hire her because he had heard she was gay. Brown, who was not gay, realized that Crako was probably thinking of another member of the
community, Jane Browne, who was widely known to be gay. Brown asked for a hearing at which she could present what she called “information which would cause the
Clerk to reverse his decision,” but town officials again refused. Brown has now sued, arguing that the procedures used to fill the opening deprived her of her Fourteenth
Amendment due process rights. Should the court find in Brown’s favor? _________________

No. The Fourteenth Amendment’s Due Process Clause only prevents the government from depriving a person of “ life , liberty , or property without due process of law.”
Unless Brown can show that she had a “liberty” or “property” interest that was impaired, she will not even get to the point of being allowed to show that fair procedures
were not followed. In other words, the Due Process Clause does not bar the government from procedural irregularities per se, only procedural irregularities in
connection with the taking of life, liberty or property. It is very clear that a person applying for an initial government position has no liberty or property interest in the
position, so the government may turn the applicant down for totally arbitrary or irrational reasons (so long as the reasons are not themselves violations of
independently-guaranteed constitutional rights, such as refusing to hire a person because she is, say, black). So the fact that the Clerk’s decision was completely “wrong,”
in the sense that he had the wrong person in mind, is completely irrelevant. We never get to the question of what type of process (e.g., right to a statement of reasons, or
right to a hearing) would have been due.
Tennant, a single mother who was receiving Welfare, resided in a public housing project owned by the city of Pretoria. She had lived in the building for over 10 years,
pursuant to a series of two-year leases. The rules of the housing project, posted on a bulletin board in the complex, stated that “customarily, residents who are well behaved
and current on all of their obligations will be offered the opportunity to renew their leases upon their expiration.” There is no statute or other body of state or local law
bearing on whether one in Tennant’s position is entitled, as a contractual matter, to a renewal. At the end of Tennant’s current two-year term, she was not offered the
opportunity to renew her lease. Instead, the apartment was given to a woman who turned out to be the niece of Pretoria’s Buildings Commissioner, who would not have
been given an apartment had the ordinary informal allocation procedures that had previously been followed in the housing project been followed here. Tennant was at no
time given an explanation for why she wasn’t permitted to renew, or a hearing regarding the decision.
(a) If Tennant wishes to challenge the city’s handling of her tenancy on constitutional grounds, what is the strongest argument she can make? _________________
(b) Will this argument succeed? _________________
(a) That it violated her right not to have her “property” taken without due process, and that this right included the right to a hearing.
(b) Yes, probably. The core issue is whether Tennant had a “property” interest in her apartment. The Court has held that eveninformal practices or customs may be
sufficient to create a legitimate claim of entitlement to a benefit. See, e.g., Perry v. Sindermann (in which a college was found to have created a de facto tenure program
that created in P, a college professor, the “understanding” that he would be entitled to tenure). Here, the statement regarding the customary right of renewal was probably
enough to create in Tennant such a legitimate claim of entitlement to renew, in view of the lack of any other law bearing on whether Tennant did indeed have such a right.
(If there were a statute or body of case law holding that as a matter of state law there is no right to renew despite a seeming indication to the contrary in a housing project’s
rules and regulations, then this body of state law would be dispositive, and Tennant would not have any “property” interest.)
If Tennant indeed had a legitimate entitlement to being allowed to renew provided that she was in good standing, then she was presumably entitled to at least a hearing
before this property interest could be taken away. (The precise procedures that would have to followed are not clear, but for a right as important as the right to continue
to live in subsidized housing, it is likely that some sort of hearing and statement of reasons, however informal, would be required.)

N&P
Congress, pursuant to its power to establish and regulate copyrights, has decided that there is far too much counterfeiting of copyrighted musical recordings. Therefore,
Congress has passed a statute making it a felony, punishable by up to five years in prison, to give a “bootlegged” (i.e., not authorized by the copyright owner) CD or MP3
recording to any other person, even if it is the donor’s neighbor or relative, and even though no compensation is charged. Dennis, charged with a violation of this statute,
asserts that it is unconstitutional because it is beyond the scope of Congress’ authority. Should the Court agree with Dennis’ assertion, and why? _________________
No, because the statute is valid under the “Necessary and Proper” Clause. McCulloch v. Maryland, 17 U.S. 316 (1819), establishes that when Congress is acting in
pursuit of a constitutionally-specified objective, the means chosen merely has to be rationally related to the objective, not “necessary” to the objective’s attainment.
Here, Congress is exercising its enumerated power to regulate copyrights. Congress could rationally have believed that even non-profit-motivated transfers of copyright-
violating recordings contribute to the general decline of copyright pro-tection, and that felony punishment for such transfers is a reasonable way of combatting the
problem. The Court will show great deference to Congress’ choice of the means to attain constitutionally-enumerated objectives, so the statute here will certainly be
sustained.

11th amendment

when you're analyzing a question in which a state is being sued in federal court, there's a potential Eleventh Amendment problem (in that the Eleventh Amendment
prohibits federal courts from exercising jurisdiction over certain kinds of suits against states). In fact patterns like these, ask the following questions:
1. Who is the plaintiff?
Eleventh Amendment applies to: All private plaintiffs.
Eleventh Amendment doesn't apply to: Another state or the federal government as plaintiff (even when it's suing to protect private persons).
2. Is the state a defendant? (The Eleventh Amendment applies only to cases against states.)
Yes: State government, agencies of the state.
No: Political subdivisions, municipal corporations, counties, school boards, or state officers sued in their personal capacity (unless the action requests the
officer be ordered to pay funds from state treasury for wrongful acts of the state or return property in the state's possession, in which case the state is
the real party in interest).
3. What relief is sought?
Eleventh Amendment applies to: Damages, past debt, retroactive relief of any type.
Eleventh Amendment doesn't apply to: Injunctive or declarative relief or suits against state officials if suit seeks to force them to conform their conduct to
federal (not state) law.
4. Has the state waived immunity?
Yes: Express (in legislation itself) or implied (willfully engaging in federally regulated activities that are not essential state functions).
No: Participation in a federal program (e.g., accepting federal funds in the absence of an explicit congressional requirement that doing so constitutes a waiver).
5. Does a congressional grant of jurisdiction apply? Federal statutes based on congressional power to enforce the Fourteenth Amendment (e.g., civil rights laws)
are not subject to the Eleventh Amendment bar.

Вам также может понравиться